heart system

187
7 1 117 Lab Values Notes Calculator . Item: 6 of 44 Mark Previous Next Feedback, Suspend End Block A 27-year-old man has a history of repeated attacks of dyspnea upon exertion. He does not use tobacco, alcohol, or illicit drugs. The patient's blood pressure is '130/80 mm Hg and pulse is 78/min. On examination, there is no peripheral edema or jugular venous distension. His lungs are clear to auscultation. There is a IIINI ejection-type systolic murmur along the lower left sternal border that decreases in intensity when the patient moves from the standing to the squatting position. Which of the following is true regarding the inheritance of this disease? O Autosomal dominant Autosomal recessive O C. Multifactorial O D. X-linked dominant E_ X-linked recessive 1 2 3 4 5 7 a 9 10 11 12 13 14 - 15 16 17 19 22 23 24 25 26 27 28 I 29 30 1 32 33 I 34 36 38 I 39 40 41 42

Upload: anonymous-n80qei6

Post on 15-Feb-2016

64 views

Category:

Documents


3 download

DESCRIPTION

information on heart, its diseases, its foundation on how heart beats. dynamic of flow and circulation

TRANSCRIPT

•71 117 Lab Values Notes Calculator .

Item: 6 of 44 Mark Previous Next

Feedback, Suspend End Block

A 27-year-old man has a history of repeated attacks of dyspnea upon exertion. He does not use tobacco, alcohol, or illicit drugs. The patient's blood pressure is '130/80 mm Hg and pulse is 78/min. On examination, there is no peripheral edema or jugular venous distension. His lungs are clear to auscultation. There is a IIINI ejection-type systolic murmur along the lower left sternal border that decreases in intensity when the patient moves from the standing to the squatting position. Which of the following is true regarding the inheritance of this disease?

O Autosomal dominant

• Autosomal recessive

O C. Multifactorial

O D. X-linked dominant

E_ X-linked recessive

1 2

3

4

5

7 • a

• 9

• 10

• 11

• 12

• 13

▪ 14 • -

• 15

▪ 16 • • 17

• 19

22

• 23

• 24

• 25

▪ 26

27

28 I

▪ 29 • 30

• 1

• 32 ■

• 33

I 34

• 36

• 38

I 39

• 40

41

42

117 Notes Calculator .

Item: 2 of 44 Lab Values.

r Mark Previous Next

Feedback, Suspend End Block

A 32-year-old man comes to the physician because of 6 months of progressive weakness and exertional dyspnea. His past medical history is significant for a knife injury, with a stab wound to the right thigh 10 months ago. He saw a doctor, had suturing, and has a healed scar. He has been arrested several times for robbery. He regularly drinks alcohol and occasionally smokes crack. His younger brother died of cystic fibrosis. He is afebrile. and his blood pressure is 160160 mm Hg and pulse is 100imin. His right leg feels warmer and appears more flushed compared to his left leg. Carotid upstroke is brisk. The point of maximal impulse is displaced to the left. and a soft systolic murmur is heard over the cardiac apex that does not change with the Valsalva maneuver. Vihich of the following is the most likely cause of this patient's symptoms?

• A_ Decreased cardiac output

• 8_ Hypertrophic cardiomyopathy

• C. Increased cardiac preload

• D_ Papillary muscle dysfunction

• E Pulmonary hypertension

1 2

3

5

7 • a

• 9

• 10

• 11

• 12

• 13

▪ 14 • • 15

▪ 16 • • 17

• 19

22

• 23

• 24

• 25

▪ 26

27

28 I

▪ 29

• 30

• 1

• 32 ■

• 33

▪ 34-

• 36

• 37

I 39

• 40

41

42

1

2

5 6 7

I

• 9 • 10

• 11 • 12

• 13

• 14 •

• 15

• 16 •

• 17

• 19

• ARM 21 —1 22

• 23

• 24

• 25

• 26

27 28 I

• 29

• 30

• 1

• 32 ■

• 33

• 34-

• 36

• 37

• 39

• 40

41

42

Item: 3 of 44

r Mark 117

Previous

Next

Lab Values. Notes . Calculator.

A 35-year-old woman who has recently emigrated from Asia presents to the emergency room with acute

onset of dyspnea_ She denies any cough, chest pain. or fever. She has a history of rheumatic heart disease

as a teen. On examination. she has an irregular pulse of 97/min. blood pressure of 125/75 mm of Hg and

temperature of 37.2°C (989°F) The first heart sound is loud and a mid-diastolic rumble is heard at the

apex_ Crackles are present in both lung fields_ ECG shows an irregularly, irregular heart rhythm and the

absence of 'F" waves.. Which of the following is the most likely cause of this patient's abnormal heart rhythm?

C. A. Left atrial dilatation

B. Right atrial dilatation

• C_ Left ventricular dilatation

O ID. Left ventricular hypertrophy

0 E. Pulmonary hypertension

Feedback, Suspend

End Block

1>- Next

rNlark -441

Previous

Item: 4 of 44

Suspend Feedback, End Block .

A 64-year-old white female presents for evaluation of two weeks of decreased appetite and nausea_ She also notes occasional palpitations, which have been especially prominent over the past two days. Her medical history is significant for an anterior wall myocardial infarction one year ago and secondary congestive heart failure with left ventricular systolic dysfunction_ Her current medications include aspirin.. digoxin, furosemide, enalapril and metoprolol_ On physical examination, her blood pressure is 120/80 mm Hg, pulse is 106/min and respirations are 15.1min_ The remainder of her exam is unremarkable_ Chest x-ray shows an enlarged cardiac silhouette and normal lung fields. On laboratory testing, her digoxin level is twice the upper limit of normal_ You order an EKG_ Which of the following arrhythmias is most specific for digitalis toxicity?

1 2

3

5

7 • a

9 • 10 • 11

• 12 • 13 ▪ 14 • • 15 ▪ 16 • • 17

• 19

22

• 23 • 24 • 25 ▪ 26

27 28 I

▪ 29 • 30

• 32 ■

• 33 ▪ 34-

I 37

I 39 • 40

41 42

• A_ Atrial flutter

• 13_ Atrial fibrillation

C. C. rylobitz type II second-degree AV block

O D. Atrial tachycardia with AV block

• E_ Multifocal atrial tachycardia

117 Notes . Calculator.

Item: 5 of 44 Lab Values.

r Mark Previous Next

Feedback, Suspend End Block

A 45-year-old mildly overweight man admitted to the hospital 5 days ago with anterior wall myocardial infarction develops sudden onset of left-sided chest pain_ He underwent cardiac catheterization on admission that showed complete occlusion of the left anterior descending artery. However. no revascularization was performed due to late presentation_ He has had no dyspnea, extremity swelling. or palpitations since admission_ His past medical history is significant for diabetes mellitus type 2 and hyperlipidemia_ He appears agitated and restless_ Two minutes later, he is unresponsive_ His pulse is not palpable and the electrocardiogram shows sinus tachycardia of 30lmin. Which of the following is the most likely diagnosis?

• A. Acute pericarditis

• B_ Aortic dissection

• C. Interventricular septum rupture

• Ip_ Papillary muscle rupture

• E Pulmonary infarction

O F. Recurrent ischemia

• G. Right ventricular infarction

• H_ Ventricular aneurysm

• I. Ventricular free wall rupture

1

2 a

7

I ▪ 9 ' 10 i 11 i 12 • 13 • 14 • - • 15 • 16 •

• 17

• 19 • ARM

21 —1 22

• 23 • 24 • 25 • 26

27 28 I

• 29

' 30

i 32 ■

• 33 ' 34

• 36

' 38 • 39 ' 40

41 42

117 Notes . Calculator .

Item: 1 of 44 Lab Values.

r Mark Previous Next

Feedback, Suspend End Block

A 40-year-old man comes to the emergency department with slowly progressive headaches. dyspnea. and blurry vision over the last 2 days. His other medical problems include hypertension treated with hydrochlorothiazide and lisinopril for the past 2 years. He has been noncompliant with his medical regimen and physician follow-ups_ He smokes 1 pack a day and occasionally consumes alcohol On initial evaluation, his blood pressure is 2201140 mm Hg and pulse is 75.1min_ Which of the following establishes the diagnosis of malignant hypertension in this patient?

1

2

3

5

7 • a

• 9

• 10

• 11

• 12

• 13

▪ 14 • • 15

▪ 16 • • 17

• 19

22

• 23

• 24

• 25

▪ 26

27

28 I

▪ 29 • 30

• 32 ■

• 33

▪ 34-

• 36

• 37

I 39

• 40

41

42

• A_ Left ventricular hypertrophy on electrocardiogram

B. Reduced glomerular filtration rate

• C. Papilledema and retinal hemorrhages

D. Diastolic blood pressure >12G mm Hg

• E. Pulmonary congestion on chest x-ray

1

3

▪ 9

' 10

i 11

i 12

• 13

• 14 • -

• 15

• 16 • • 17

• 19

21

22

• 23

• 24

• 25

• 26

27

I • 29

• 1

i 32 ■

• 33

' 34

' 36

' 38

• 39

' 40

' 30

Item: 7 of 44

r Mark 127

Previous

Next

Lab Values, Notes Calculator.

A 32-year-old man is brought to the emergency department complaining of sudden-onset. excruciating chest and neck pain. He has no prior history of dyspnea, chest pain, or lower-extremity edema. He is 6 feet. 5 inches tall with long extremities. His hand joints show significant extensibility. Pectus carinatum deformity is present Which of the following additional findings is likely in this patient?

• A_ Early diastolic murmur

• 13_ Fixed splitting of the second heart sound

• C_ Opening snap

C. D. Pericardial friction rub

E. Pulsus parvus et tardus

F. Third heart sound

Feedback, Suspend

End Block

1

3

5

7

8

9

• 10

• 11

• 12

• 13

▪ 14 • • 15

▪ 16 • • 17

• 19

22

• 23

• 24

• 25

▪ 26

27

28 I ▪ 29

• 30

• 32 ■

• 33

▪ 34-

• 36

• 37

I 39

• 40

41

42

Item: 8 of 44

117 Notes

r Mark Previous Next Lab Values. Calculator .

A 34-year-old woman is admitted to the hospital because of septic shock secondary to a urinary tract infection. In the intensive care unit. she receives intravenous fluids and antibiotics. Her initial ECG shows sinus tachycardia but is otherwise unremarkable. Chest x-ray shows no abnormalities. An internal jugular vein catheter is placed on the right side using ultrasound guidance to locate the vein_ The patient is properly draped_ and the skin is cleaned with chlorhexidine solution_ Blood is freely aspirated from all ports after insertion. Vital signs are stable. and oxygenation is maintained_ Which of the following is the most appropriate next step in managing this patient?

• A Antibiotic instillation into the catheter

• 13_ Echocardiography

• C_ Heparin instillation into the catheter

• D_ Portable chest x-ray

• E. Repeat 12-lead ECG

Feedback, Suspend

End Block

2

a

5

7

9

10

• 11

' 12

' 13

▪ 14 • • 15

▪ 16 • • 17

• 19

22

• 23

• 24

• 25

▪ 26

27

28 I ▪ 29 ' 30

• 1

' 32 ■

• 33

' 34

• 36

• 37

• 39

' 40

41

42

Item: 9 of 44

r Mark 127

Previous

Next

Lab Values. Notes . Calculator.

A 64-year-old male presents to the ER with chest pain. His history is significant for a hospitalization 2 weeks ago for an acute myocardial infarction. The patient reports that he is afraid his heart attack is recurring. He localizes the pain to the middle of the chest. and says that he can only take shallow breaths because deep breaths make the pain worse_ Leaning forward in his chair makes the pain somewhat better_ He notes some associated neck pain and general malaise, but denies shortness of breath. palpitations, syncope or cough_ His temperature is 367° C (980 F). blood pressure is 135/84 mmHg and heart rate is 9almin_ EKG shows ST segment elevations in all limb and precordial leads except in aVR. where ST depression is seen_ Which of the following is the best next step in the management of this patient'?

O A_ Anticoagulation with heparin

• 13_ Anticoagulation with warfarin

O C. Nitroglycerin and beta-blockers

O D. Broad-spectrum antibiotics

• E_ Non-steroidal antiinflammatory agent

Feedback, Suspend

End Block

1

3

5

7

9

• 10 I•

• 12

• 13

• 15

• 16 •

• 17

• 19

22

• 23

• 24

• 25

• 26

27

28 I

• 29

' 30

• 1

' 32 ■

• 33

' 34

• 36

• 37

• 39

' 40

41

42

Item: 10 of 44

r Mark 117

Previous

Next

Lab Values. Notes Calculator.

A 25-year-old woman is undergoing an elective cholecystectomy after 2 episodes of acute calculous cholecystitis. The procedure is converted from laparoscopy to open due to difficulty with visualization. The patient suddenly becomes hypotensive and develops a diffuse generalized rash. Her past medical history is significant for obesity and polycystic ovarian syndrome. Two months ago. she developed hives after having protected sex with her partner for the first time. Which of the following is the most likely cause of this patient's hypotension?

O A. Acute blood loss

• 13_ Adrenal insufficiency

• C. Anaphylactic shock

• D_ Cardiac tamponade

O E. Disseminated intravascular coagulation

O F. Myocardial infarction

• G_ Pulmonary embolism

• H. Septic shock

• I. Tension pneumothorax

Feedback, Suspend

End Block

2 3

1 1 4 5

7

8 9

11

I. 12 • 13

• 15 • 16 •

• 17

• 19

22 • 23 • 24 • 25 • 26

27 28 I

I. 29 ' 30

' 32 ■

• 33 ' 34- El

• 36 • 37

• 39 ' 40

41 42

Item: 11 of 44 r Mark 127

Previous Next Lab Values. Notes Calculator.

A 62-year-old man with a past medical history of bladder cancer and recurrent hematuria comes to the emergency department after "passing out." The patient was standing up to urinate shortly after waking when he felt dizzy. He was able to return to his bed before losing consciousness for several minutes. The patient's electrocardiogram (EGG) at the time of admission is shown in the Exhibit_ Which of the following most likely accounts for the observed ECG changes?

• A Atrial fibrillation

• E. Atrial premature beats

• C_ Impaired atrioventricular node conduction

O a Impaired sinoatrial node automaticity

• E Ventricular preexcitation

1.1 Feedback, Suspend End Block

Feedback, Suspend End Block

Media 1 of 1

rt iimmounn 11 MILE =EMI

I

;

I

• :

: • • : I •,,:.•.. . • • ; • -

MENEMEN =NNE •

1r

MENNEM.li MI

imummionim

=IT

f • : • • • 4 • • • • 1. 1 : • .1 i • i • i I. ..•

• •1

Media Exhibit

12

▪ 13

▪ 14

▪ 15

▪ 16

▪ 17

▪ 1 a

▪ 19

▪ 20

▪ 21

▪ 22

▪ 23

▪ 24

▪ 25

▪ 26

▪ 27

▪ 28

▪ 29

▪ 30

▪ 1

• 32

▪ 33

▪ 34

▪ 36

• 38

• 39

40

▪ 41

▪ 42 Ars

127 Notes . Calculator.

Item: 12 of 44 Lab Values.

rMark Previous Next

Feedback, Suspend End Block

1

3

5

7

8 9

10 I 11

I 12 13

I • 14 1. ' 15 • 16 • • 17

• 19 • ARM

21 —1 22

' 23 ' 24 ' 25 • 26

27 28 I

• 29

• 30 • 1 ' 32 ■

• 33 • 34-

• 36 • 37

• 39 • 40

41 42

A 47-year-old female smoker with type 2 diabetes mellitus, mild normocytic anemia, and stage 3 chronic renal disease is being evaluated for exertional dyspnea. Her family history is insignificant. Blood pressure is 142/88 mm Hg and pulse is 76/min. Her body mass index is 32 kg/m2. Single photon emission CT images are obtained at rest and after 5 minutes of treadmill exercise by using separate intravenous injections of technetium-99-labeled perfusion agent, as shown in the images below.

This patient would benefit most from which of the following medications?

SUPINE

APEX

SEP 1111 LAT

BASE

STRESSJRNO(G)

SEP AT

BASE

REST_IMIC

• A. Albuterol

• Antiplatelet agent

• C_ Appetite suppressant

Erythropoietin

• E Metformin

1

3

5

7 • a

• 9 • 10 • 11 • 12

13 14 15 16 ■

17

• 19

21 22

• 23 • 24 • 25 ▪ 26

27 28 I

▪ 29

• 30

• 32 ■

• 33 ▪ 34-

• 36 • 37

I 39 • 40

41 42

Item: 13 of 44

rNlark 117

Previous Next

Lab Values. Notes . Calculator.

A 65-year-old man comes to the physician for fatigue, poor appetite, and lower-leg swelling_ Past medical history is significant for hypertension. type 2 diabetes mellitus. chronic cough, chest infections, and lower-extremity peripheral artery disease. The patient has a 40-pack-year smoking history_ He does not use alcohol or illicit drugs. Physical examination shows a barrel-shaped chest, with bilateral scattered wheezing on auscultation. The patient's abdomen is distended, and the liver edge is palpated 4 cm below the right costal margin. There is 2+ pitting edema and dilated, tortuous superficial veins over both lower extremities. Manual pressure on the abdomen causes persistent distension of the jugular veins_ His serum sodium level is 135 mEq/L, and creatinine is 12 mgidL. An abnormality of which of the following is most likely the cause of this patient's edema?

• A_ Portal venous resistance

0 13_ Pulmonary artery systolic pressure

• C_ Pulmonary capillary wedge pressure

O D. Serum albumin level

• E. Urinary protein excretion

Feedback, Suspend

End Block

1

3

5

7 • a

• 9

• 10

• 11

• 12

▪ 13

• 15

L.. 17

• 19

21

22

• 23

• 24

• 25

▪ 26

27

28 I

▪ 29 • 30

• 32 ■

• 33

▪ 34- El

• 36

• 37

• 39

• 40

41 42

Item: 14 of 44 r Mark 117

Previous Next

Lab Values, Notes Calculator.

Neighbors find a 65-year-old male unconscious in his garden and immediately call EMS. The man is intubated at the scene. In the ER, he develops sustained ventricular tachycardia and requires cardioversion. He is started on an antiarrhythmic agent and, once recovered, is discharged to home. Three months later he returns to his physician complaining of dyspnea on exertion_ Chest-x ray reveals bilateral lung fibrosis_ All cultures are negative and lung biopsy reveals lipoid pneumonitis_ Which of the following medications is most likely responsible for his current condition?

• Procainamide

• Amiodarone

• C. Lidocaine

0 D. Quinidine

O E. Digoxin

C. F. Metoprolol

• G_ Disopyramide

O H. Flecainide

O I_ Ibutilide

O J. Sotalol

Feedback, Suspend

End Block

1

3

5

7

• 9 • 10

• 11 • 12

13 14

• 15

17

19

21 22

• 23 • 24 • 25 ▪ 26

27 28 I

• 29 • 30

• 32 ■

• 33 I 34

• 36

• 38 I 39 • 40

41 42

Item: 15 of 44

r Mark 117

Previous Next

Lab Values. Notes . Calculator.

A 21-year-old man comes to the emergency department with dizziness and palpitations that started abruptly an hour ago. He has had similar episodes provoked by fatigue or strong emotions. The patient can usually stop the episodes by squatting and taking a deep breath. However, this time these actions did not work. Blood pressure is €5/40 mm Hg and pulse is 24Gimin_ He is diaphoretic and his extremities are cold. Electrocardiogram (EGG) rhythm strip shows a regular, narrow-complex tachycardia. Which of the following is the best next step in management of this patient?

• A. Adenosine

• Amiodarone

• C_ Intravenous fluid resuscitation

Procainamide

• E. Sedation and direct current cardioversion

• F. Verapamil

Feedback, Suspend

End Block

1

1 6 3

5

• 7

• 9

• 10 • 11 • 12 • 13 • 14 • 15

16 • 17

19

21 22

• 23 • 24 • 25 • 26

27 I

• 29

• 1 • 32 ■

• 33 • 34-

• 36 • 37

• 39 • 40

• 30

Item: 16 of 44 r Mark 117

Previous Next

Lab Values. Notes Calculator.

A 41-year-old man comes to the physician for a routine health examination_ He has no symptoms except for mild increase in frequency of urination. He has a history of elevated glucose levels but no other medical problems and takes no medications. His family history is significant for type 2 diabetes mellitus. The patient does not use tobacco, alcohol. or illicit drugs_ He works as an editor for a local magazine company. His temperature is 36_7 C (98 F), blood pressure is 132/80 mm Hg, pulse is 72/min, and respirations are 16 min. Body mass index is 30 kg/m2. Fasting laboratory results are as follows:

Glucose 165 mgldL Total cholesterol 210 mgldL Low-density lipoprotein 140 mgfdL Triglycerides 140 mgldL High-density lipoprotein 35 mg/dL Hemoglobin 1. 7.2% Serum creatinine 1.1 mgftIL

Which of the following is the most appropriate next step in management of this patient?

A_ Fish oil supplementation

B. Initiation of oral glimepiride

111 C. Intense lifestyle modification only

111 D. Lifestyle modification plus niacin

111 E. Lifestyle modification plus rosuvastatin

Feedback, Suspend

End Block

1

3

5

7 • a

• 9

• 10

• 11

• 12

• 13

▪ 14 •

• 15

16 —I 17

19 • -

21

22

• 23

• 24

• 25

▪ 26

27

28 I

▪ 29 • 30

• 32 ■

• 33

▪ 34-

• 36

• 37

I 39

• 40

41

42

Item: 17 of 44 Mark 117

Previous

Next

Lab Values. Notes Calculator .

An 82-year-old male presents for evaluation of chronic back pain_ On physical examination. he is found to have a blood pressure of 1&F/8 mmHg while supine and 135/ID mmHg while standing. He is otherwise healthy; his only medicine is occasional ibuprofen for back pain_ Which of the following age-related changes best explains the observed finding?

• A Increased left ventricular wall stiffness

• E Decreased left ventricular contractility

• D_ Decreased baroreceptor responsiveness

• D_ Decreased stress-mediated catecholamine release

• E Decreased glomerular filtration rate

Feedback, Suspend

End Block

117 Notes Calculator.

Item: 18 of 44 Lab Values.

r Mark Previous Next

Feedback, Suspend End Block

A 42-year-old woman is evaluated for depression, mood swings, and poor sleep. She also complains of mild headaches and muscle weakness. She has had 2 emergency department visits for kidney stones over the past year. She does not use illicit drugs. Her blood pressure is 1601105 mm Hg and pulse is 85/min.

Laboratory results are as follows:

Sodium 140 mEq/L Potassium 3.6 rriEgiL Chloride 101 mEq/L Bicarbonate 24 mEq/L Blood urea nitrogen 13 mg/dL Creatinine C1.9 mg/dL Glucose 98 mgldL Calcium 113 mgldL Albumin 33 WI:IL

Which of the following is the most likely cause of this patient's hypertension?

O A. Renal parenchymal disease

O B. Renal vascular stenosis

• C_ Coarctation of the aorta

0 Hypothyroidism

C. E. Parathyroid gland disease

O Cushing's syndrome

O G_ Amphetamine abuse

1

3

5

7

• 9 • 10

• 11 • 12

• 13

• 14

• 15

• 16

17

18

19

20 1. 21

• 22 • 23

• 24

• 25

▪ 26

27

28 I

• 29

• 30

32 ■

• 33

34-

36

• 37

I 39

40

41

42

Mark Item: 19 of 44 Lab Values. Notes Calculator . Previous Next

Feedback, Suspend End Block

A 54-year-old man comes to the physician after having woken up in the middle of the night with substernal discomfort, which he describes as a burning sensation. The patient also has left-sided neck pain and feels sweaty and short of breath. He has never had similar pain before. His past medical history is significant for type 2 diabetes mellitus and hypertension_ He has a 30-pack-year smoking history_ Which of the following physical findings is most commonly associated with this patient's clinical presentation?

• A. Ejection-type systolic murmur

• 13. Fixed splitting of S2

O C_ Fourth heart sound

O a Friction rub

• E Pulsus paradoxus

1

3

5

7 • a

• 9

• 10

• 11

• 12 • 13 ▪ 14 • • 15 ▪ 16 • • 17 • 18

19

2 21

22 ▪ 23

• 24 • 25 ▪ 26

27

28 I

▪ 29

• 30

• 32 ■

• 33 ▪ 34-

• 36 • 37

• 39 • 40

41

42

1

3

5

7 • a

• 9

• 10

a 11

• 12

• 13

▪ 14 • • 15

▪ 16 • 17

19

a 20

21

22 I, 23

• 24

• 25

▪ 26

27

28 I

▪ 29

• 30

• 32 ■

• 33

▪ 34-

• 36

• 37

I 39

• 40

41

42

Item: 20 of 44

rNlark 127

Previous

Next

Lab Values. Notes Calculator .

A 24-year-old man experiences syncope while shoveling snow_ He regains consciousness within 1 minute_ He has had some shortness of breath and chest pains recently, related primarily to exercise. He denies illicit drug use. His temperature is 37.2° C (98.9° F), blood pressure is 126/76 mm Hg, pulse is 76/min, and respirations are 14/min_ Physical examination shows a well-built man in no apparent distress_ Lungs are clear_ A crescendo-decrescendo systolic murmur is heard along the left sternal border without carotid radiation_ Chest x-ray is normal_ Which of the following is the most likely cause of his syncopal episode?

• Atrioventricular conduction delay

• 13. Interventricular septal hypertrophy

• C_ Aortic dissection

• a Aortic stenosis

• E. Mitral valve degeneration

Feedback, Suspend

End Block

1

3

5

7 • a

• 9

• 10

a 11

• 12

• 13

▪ 14 • • 15

▪ 16 • 17

19

• 2°

a 21

22 23

▪ 24

• 25

▪ 26

a 27

28 I ▪ 29

• 30

• 32 ■

• 33

▪ 34-

• 36

• 37

I 39

• 40

41

42

Item: 21 of 44

r Mark 127

Previous I

Next

Lab Values. Notes Calculator.

A 53-year-old man is admitted to the hospital with a 4-week history of fatigue and decreased exercise tolerance. Climbing 2 flights of stairs causes significant dyspnea. He has had occasional palpitations for months but no chest pain. Past medical history is unremarkable. and a routine check-up 6 months ago was normal. He had 2 binge-drinking episodes last month but usually only drinks in moderation_ The patient does not routinely take any medications_ His blood pressure is 150190 mm Hg and pulse is 130/min and irregular_ Lungs are clear on auscultation_ Electrocardiogram (ECG) does not show clear P waves_ Echocardiography shows ejection fraction of 35%; moderate central mitral regurgitation. and left atrial and left ventricular dilatation with global hypokinesis_ Which of the following interventions is most likely to restore left ventricular function in this patient?

• A_ Coronary revascularization

• B_ Decreasing afterload

• C. Inotropic medications

• D_ Preload optimization

• E Rate or rhythm control

• F. Valve surgery

Feedback, Suspend

End Block

-<=.1

Previous

117 Notes . Calculator.

Item: 22 of 44 Lab Values.

rNlark Next

Feedback, Suspend End Block

A 36-year-old woman comes to the emergency department complaining of chest pain that started suddenly while she was shopping at the mall. She also reports shortness of breath, palpitations, and diaphoresis. The pain is retrosternal and radiates to the left arm. There are no aggravating or relieving factors. She appears to be in mild discomfort On review of systems, the patient reports having had a runny nose, sore throat, and dry cough for the past 3 days. Her other medical problems include panic attacks treated with paroxetine and dysfunctional uterine bleeding treated with estrogen. Family history is significant for her father's sudden death at age 44 from a heart attack_ The patient has a 15-pack-year smoking history. Her blood pressure is 144/90 mm Hg and pulse is 1 041min and regular. Her body mass index is 29 kg/m2. Which of the following is the most appropriate initial therapy for this patient?

• A. Acetaminophen

• 8_ Aspirin

• C. Heparin

• D_ Ibuprofen

• E Lorazepam

• F_ Oxycodone

1

3

5

7

• 9

• 10

• 11

• 12

• 13

▪ 14-

• 15

▪ 16 •

17

19

20 I

21

• 22 23

24

• 25

▪ 26

• 27

28 I

• 29

• 30

• 32 ■

• 33

I 34

• 36

• 38

I 39

• 40

41

42

1

3

5

7 • a

9 • 10

• 11 • 12 • 13 ▪ 14 • • 15 ▪ 16 • • 17

19

213°11. 21 —1 22-1 23

• 24 • 25 ▪ 26 • 27

28 I ▪ 29

• 30

• 32 ■

• 33 ▪ 34-

• 36 • 37

I 39 • 40

41 42

Item: 23 of 44

117 Notes

rNlark Previous Next Lab Values, Calculator .

An 82-year-old woman comes to the emergency department with sudden-onset epigastric pain. The pain started an hour ago and is 8110 in severity_ The patient has been nauseated and vomited twice in the past hour. Her past medical history includes type 2 diabetes mellitus, hyperlipidemia, hypertension, peptic ulcer disease 10 years ago, and cholelithiasis. She has a 40-pack-year smoking history and drinks alcohol occasionally_ Her temperature is 36_8 C (981 F), blood pressure is 140/80 mm Hg, pulse is 90/min, and respirations are 141min_ The lungs are clear to auscultation_ Abdominal examination shows a soft abdomen with no tenderness_ Murphy's sign is negative. Which of the following studies should be performed first in this patient?

O A. Abdominal ultrasound

• 13_ Electrocardiogram

C_ Serum amylase and lipase

D_ Upper gastrointestinal endoscopy

• E Upright abdominal x-ray

Feedback, Suspend

End Block .

127 Notes Calculator .

Item: 24 of 44 Lab Values.

r Mark Previous Next

Feedback, Suspend End Block

44.

A 12-year-old male is admitted to the hospital with chest pain_ EKG and cardiac enzymes confirm a

myocardial infarction and he is started on nitroglycerine. aspirin, simvastatin and low molecular weight

heparin. Four days later, he complains of leg pain. Physical examination reveals a cold, pale leg. The

angiogram is shown below.

What is the best next step in his management?

1

3

5

7

• 9

• 10

• 11

• 12

• 13

■ 14

• 15

■ 16 •

• 17

• 19

▪ ARM

21

i 24

25

26 _I

27

28 I

• 29

• 30

• 32 ■

• 33

▪ 34-

• 36

• 37

I 39

• 40

41

42

• 1

2 6

3

Iffle • Item: 24 of 44 111 'Mark

Calculator Lab Values Notes Next Previous

End Block 11/11 0

Feedback, Suspend

5 6 6

7 6 8

▪ 9

6 10

6 11

6 12

▪ 13

14 15

i • 16 • 17

• 113

• 19

• 20

I • 21

23

• 24

25

27

I • 28

29

1 . 30 .31

1• .32

34

* 36 37

1 . 38 39

1• 40

41

What is the best next step in his management?

O A. Intravenous streptokinase

O B. Clopidogrel

O C. Cilostazol

• D. Leg elevation and warm compress

O E. Embolectomy V

1

3

5

7 • a

• 9

• 10

• 11

• 12

• 13

▪ 14 • • 15

▪ 16 • • 17 ▪ a—. • 19

24

I 25

26

27

▪ 29

• 30

• 32 ■

• 33

▪ 34-

• 36

• 37

I 39

• 40

41

42

Item: 25 of 44

lirMark 127

Previous

Next

Lab Values. Notes Calculator .

A 36-year-old man comes to the physician for a routine pre-employment physical_ He has no complaints except for occasional morning headaches_ His father died suddenly at age 54_ The patient's blood pressure is 175/103 mm Hg in the right arm and 180/105 in the left, and pulse is 82/min. The lungs are clear bilaterally and heart sounds are normal. Bilateral, nontender, upper abdominal masses are palpated on examination. His hemoglobin level is 151 g/dL and creatinine concentration is 0.8 mgicIL. Which of the following is the most appropriate next step in evaluating this patient's condition?

• A 24-hour urine cortisol

• 13_ Abdominal ultrasound

O C. Captopril-enhanced radionuclide renal scan

O D. Plasma aldosterone/renin ratio

• E_ Urine metanephrines

Feedback, Suspend

End Block

11275011°' Notes

rNlark Next Previous Calculator . Lab Values.

Item: 26 of 44

262 mg/I:IL 50 mgfdL 162 mg/dL 150 mgfdL 100 mgfdL

Total cholesterol High-density lipoprotein Low-density lipoprotein Triglycerides Fasting glucose

Feedback, Suspend End Block

A 46-year-old man comes to the physician for a routine health maintenance visit He has no medical problems. He smokes a pack of cigarettes daily and does not drink alcohol_ His mother has hypertension, and his father died of prostate cancer at age 10. The patient's blood pressure is 128/76 mm Hg and pulse is 80 min. His body mass index is 29 kg/m2. Laboratory results are as follows:

His calculated 10-year risk of cardiovascular disease is 9%. Which of the following is the most appropriate next step in management of this patient's dyslipidemia?

1

3

5

7 • a

9

• 10

• 11

• 12

• 13

▪ 14 • • 15

▪ 16 • • 17

• 19

22

• 23

24

25

26

27

28

• 29

• 30

• 32 ■

• 33

▪ 34-

I 37

I 39

• 40

41

42

• A_ Fish oil supplementation

Lifestyle modification only

O C_ Low-dose aspirin

D. Oral niacin therapy

E. Oral rosuvastatin

1

3

5

7 • a

9

• 10

• 11

• 12

• 13

▪ 14 • • 15

▪ 16 • • 17

• 19

22

• 23

1 . 24

25

26

27

29

• 30

• 32 ■

• 33

▪ 34-

• 36

• 37

I 39

• 40

41

42

Item: 27 of 44

127 Notes

r Mark Previous Next Lab Values. Calculator.

A 5:3-year-old woman is admitted to the intensive care unit with hypotension. She is receiving no vasoactive agents and is breathing room air. Blood pressure measured by an infra-arterial line is 72/46 mm Hg. Her pulse is 120/min and regular. Invasive hemodynamic monitoring is established and initial readings are as

i_Ilmonary capillary wedge pressure 6 mm Hg (N: 6-12 mm Hg) Mixed venous oxygen saturation 78% (N: 65%-75%)

Which of the following is the most likely cause of this patient's condition?

C. A. Cardiogenic shock

O B. Pericardial tamponade

C_ Right ventricular infarction

O Septic shock

• E. Volume depletion

Feedback, Suspend

End Block .

mohammed
Sticky Note
start here sat

Item: 28 of 44 : II rNlark Calculator, Lab Values, Previous ! Notes, Next

Feedback, Suspend End Block

1

3

5

7 • a

9

• 10

• 11

• 12

• 13

▪ 14 • • 15

▪ 16 • • 17

• 19

22

• 23

• 24

25 26

27

I 28

29

I. 30

• 32 ■

• 33

▪ 34-

I 37

I 39

• 40

41

42

A 34-year-old male who recently immigrated from Brazil presents with a 5 month history of exertional dyspnea without associated chest pain. palpitations, dizziness, or syncope_ His past medical history is significant for an episode of megacolon. which was treated 2 years ago_ On physical examination, there is 1+ pedal edema and mild jugular venous distention. Cardiac exam is significant for the presence of an S3, but no murmurs are heard_ Chest x-ray reveals prominent cardiomegaly_ Based on these findings_ which of the following is most likely causing his symptoms?

• A. [)iphtheric myocarditis

0 B. Coronary artery disease

• C_ Protozoal disease

ID. Giant cell myocarditis

• E. Rickettsial myocarditis

1

3

5

7

• 9 • 10 • 11 • 12 • 13 ▪ 14 • 15 ▪ 16 •

• 17

• 19 ▪ ARM

21 22

• 23 • 24 • 25

I. 26 27 28

30

32 ■

• 33 34-

36 • 37

I 39 40 41 42

Item: 29 of 44

rNlark -<=.1

117 Previous i Next

Lab Values. Notes . Calculator.

A 29-year-old man comes to the clinic for a follow-up 2 weeks after an emergency department visit for epistaxis requiring anterior nasal packing. In the emergency department. his blood pressure was 170/110 mm Hg. He complains of occasional headaches and fatigue but has no chest pain. palpitations. or syncope. His past medical history is unremarkable and he does not use tobacco_. alcohol, or illicit drugs. His current blood pressure is 180/112 mm Hg and his pulse is 78/min and regular_ Cardiac auscultation in the supine position shows no murmurs or additional sounds_ Abdominal examination shows no periumbilical bruits. Electrocardiogram shows normal sinus rhythm, high-voltage ORS complexes.. downsloping ST-segment depression. and T-wave inversion in leads V5 and V6_

Laboratory results are as follows:

Hemoglobin 14_2 gIdL Platelets 231:1,01:1Dim M3 Creatinine 111 mg/dL

Which of the following is the best next step in evaluation of this patient?

• A_ Ambulatory blood pressure monitoring

• B_ Bilateral arm and leg blood pressure measurements

• C. Cardiac auscultation in squatting and standing positions

O D_ Carotid sinus massage

E Exercise stress testing 0

Feedback, Suspend

End Block

1

3

5

7

9 • 10 • 11 • 12 • 13 ▪ 14- • 15 ▪ 16 •

• 17

• 19 ▪ ARM

21 22

• 23 • 24 • 25 ▪ 26

27 28 I 29 30 31

• 33 I 34

• 36

• 38 I 39 • 40

41 42

Decreased

Decreased

Decreased

Increased

Normal

• A_ Decreased

• 13. Increased

• C_ Decreased

• D. Increased

• E Normal

Increased

Decreased

Increased

Increased

Increased

Increased

Decreased

Decreased

Normal

Normal

Item: 30 of 44

127 Notes

r Mark Previous Next Lab Values, Calculator.

A 25-year-old woman, gravida 2. para 1, at 28 weeks gestation is brought to the emergency department by her boyfriend who found her lying in a pool of blood on the bed. She is very drowsy but has no pain or uterine contractions. She has no other medical conditions and has had an uneventful pregnancy so far. Her only medication is a prenatal vitamin. The patient's temperature is 36_7 C (8 F), pulse is 120/min. and respirations are 181min. Physical examination shows that the fetus is in a transverse lie_ Inspection of the perineum shows gross blood and active bleeding per vagina_ Which of the following set of parameters would most likely be seen in this patient?

Cardiac

Pulmonary Capillary Systemic Blood Output

Wedge Pressure Vascular Resistance Pressure

Feedback, Suspend

End Block

1

3

5

7

• 9

• 10

• 11

• 12

• 13

▪ 14 • • 15

▪ 16 • • 17

• 19

22

• 23

• 24

• 25

▪ 26

27

28 I

29

31

• 33

▪ 34-

I 37

I 39

• 40

41

42

Item: 31 of 44

1>-

rNlark

Previous

Next

Lab Values. Notes

Calculator.

A 64-year-old man complains of palpitations and progressive shortness of breath over the past several hours_ He says that he also develops a choking sensation every time he tries to lie down. His medical history is significant for hypertension for the past 20 years and medication non-compliance. He also has a 35 year smoking history_ He reports that his father died of a heart attack at age 10 and his mother suffered from asthma On physical examinatioft his blood pressure is 170/100 mmHg and his heart rate is 130/min and irregularly irregular_ Lung exam reveals bibasilar crackles_ There is 2+ pitting edema of the lower extremities_ Bedside echocardiography shows a left ventricular ejection fraction of 55%_ Which of the following is most likely responsible for his symptoms?

• A. Cardiogenic shock

• B. Diastolic dysfunction

C. C. High-output heart failure

C. D. Small airway bronchoconstriction

• E_ Increased lung compliance

Feedback, Suspend

End Block .

1P4

Notes r Mark

Calculator,

: : II

Lab Values, SI Item: 32 of 44

Previous Next

Suspend Feedback, End Block ,

A 31-year-old healthy Caucasian male is seen in your office for a routine physical examination. He denies any symptoms or illness_ He says he smokes a pack a day and drinks one to two beers every weekend. He has no allergies. Examination is unremarkable. The EKG reveals normal sinus rhythm with a heart rate of 72: there are frequent premature atrial beats present The blood pressure is 120/65mm Hg. The next step in his management is:

1

3

5

7 • a

9

• 10

• 11

• 12

• 13

▪ 14 • -

• 15

▪ 16 •

• 17

• 19

22

• 23

• 24

• 25

▪ 26

27

28 I ▪ 29

• 30

▪ 31

32

33

▪ 34

• 36

• 37

I 39

• 40

41

42

C.) A. Digoxin

B. Lidocaine

C. Order potassium levels

• D_ Complete electrophysiological study

• E Observation

Feedback, Suspend End Block

A 58-year-old woman is evaluated for chest discomfort that occurs while climbing stairs and %.valking uphill. She also complains of associated shortness of breath. She has no known history of coronary artery disease or stroke. Her past medical history is significant for anxiety disorder. hypertension. hyperlipidemia, type 2 diabetes mellitus. and acid reflux disease_ Her body mass index is 3G kgim2_ Baseline electrocardiogram shows normal sinus rhythm_ Chemistry panel shows normal electrolyte levels and creatinine of 1_1 mgicIL. An exercise (treadmill) stress test is scheduled to diagnose coronary artery disease_ Which of the following daily medications should be held 24-48 hours before the test?

Long-acting nifedipine

Metoprolol Ramipril Chlorthalidone Simvastatin

1

1 6 3

5

• 7

• 9

' 10 i 11 i 12 • 13 • 14- • 15 • 16 •

• 17

• 19

21 22

• 23 • 24 • 25 • 26

27

I • 29

• 1 32

I 33

' 36 • 37

• 39 ' 40

' 30

117 Lab Values. Notes . Calculator.

O A. Yes Yes No No No

O B. Yes Yes Yes Yes No

O C_ Yes No No No No

O a No Yes Yes No No

O E. No No Yes No Yes

Item: 33 of 44

r Mark Previous Next

1

3

5

7 • a

• 9 • 10

• 11 • 12

• 13

▪ 14 • -

• 15

▪ 16 • • 17

• 19

22 • 23

• 24

• 25

▪ 26

27

28 I ▪ 29 • 30

32

33

24

• 36

• 37

I 39

• 40

41

42

Item: 34 of 44

r Mark 117

Previous

Next

Lab Values. Notes Calculator.

A 34-year-old man rushes into the ER complaining of severe substernal chest pain that began abruptly 30 minutes ago. He says that he also feels as though his heart 'is racing,' but denies any shortness of breath. cough or fever. He has never experienced pain like this before. His past medical history is significant for an appendectomy one year ago. The patient reports that his father died at age 64 due to "some heart problem" and his mother died of ovarian cancer. On physical examination, the patient is agitated and sweating profusely. His pulse is 110 min. blood pressure is160/100 mm Hg, and respirations are 14/min. Physical examination is normal except for dilated pupils and a small amount of blood at the external nares. EKG shows ST elevations in leadsV1_v4. liAlhat is the most likely explanation for his symptoms?

• A_ Atherosclerotic vascular disease

• 11 Acute pericarditis

• C_ Pleurodynia

O D. Drug-induced vasospasm

O E. Aortic dissection

• F. Pulmonary infarction

Feedback, Suspend

End Block

Next

117 Notes

r Mark Previous I Calculator . Lab Values.

Item: 35 of 44

Feedback, Suspend End Block

A 42-year-old man comes to the physician for recurrent palpitations. After initial evaluation. he is diagnosed with paroxysmal atrial fibrillation. Echocardiogram shows normal left ventricular function and no valvular disease. He is initiated on medication therapy. Two weeks later, the patient undergoes a treadmill exercise test During the test, his heart rate increases from 75/min to 165/min and the QRS complex duration increases from 109 seconds to 1113 seconds_ Which of the following medications is most likely responsible for the observed findings?

• Amlodipine

O B. Digoxin

• C_ Flecainide

Metoprolol

• E. Verapamil

1

3

5

7 • a

9

• 10

• 11

• 12

• 13

▪ 14 • -

• 15

▪ 16 • • 17

• 19

22

• 23

• 24

• 25

▪ 26

27

28 I

▪ 29 • 30

1

• 32 ■

• 33

▪ 34 II

35

• 36

• 37

• 38 I

I 39

• 40

41

42

127 Notes . Calculator .

Item: 36 of 44 Lab Values,

r Mark Previous Next

Suspend Feedback, End Block .

1

3

5

7 • a

• 9

• 10

• 11

• 12

• 13

▪ 14 • • 15

▪ 16 • • 17

• 19

22

• 23

• 24

• 25

▪ 26

27

28 I

▪ 29 • 30

• 32

• 33

▪ 34

35

• 36

37

• 38

I 39

• 40

41

42

A 41-year-old man develops sudden onset of mid-sternal chest pain and diaphoresis during a meeting in his office. While waiting for emergency medical personnel to arrive, he complains of dizziness and becomes unresponsive_ His coworkers perform cardiopulmonary resuscitation and he regains consciousness after 60 seconds_ The patient has a history of diet-controlled type 2 diabetes mellitus, hypertension. and hyperlipidemia_ In the emergency department, his blood pressure is 142/88 mm Hg and pulse is 92/min. Electrocardiogram shows normal sinus rhythm, ventricular premature complexes, and a 3-mm ST-segment elevation in leads V1-V3_ What is the most likely primary pathophysiologic mechanism responsible for this patient's syncopal episode?

• Asystole

• 13_ Atrial fibrillation

C_ Atrio-ventricular conduction block

D_ Paroxysmal supraventricular tachycardia

C. E. Pulseless electrical activity

C. F. Reentrant ventricular arrhythmias

' 41

1

1 6 3

5

• 7

• 9

' 10 i 11 i 12 • 13 • 14 • • 15 • 16 •

• 17

• 19

21 22

• 23 • 24 • 25 • 26

27

I • 29

• 1 i 32 ■

• 33 ' 34- El I

' 36 37

• 38 • 39 ' 40

' 30

Item: 37 of 44

r Mark 117

Previous Next

Lab Values. Notes Calculator.

A 64-year-old nondiabetic man complains of cramping pain in his right thigh after walking 2 blocks. The pain goes away once he stops and rests for several minutes. He is an ex-smoker with 35-pack-year history. On physical examination. the distal pulses are diminished on the right leg. His ankle brachial index is 0.98 on the left and 0.75 on the right. Which of the following is this patient most likely to suffer over the next 5 years?

• A_ Abdominal aortic aneurysm rupture

• 11 Above-knee amputation

• a Below-knee amputation

• D_ Intracranial hemorrhage

• E Myocardial infarction

1.1 Feedback, Suspend End Block

UP Notes

-441

Previous

Item: 38 of 44 Calculator,

:iii

Lab Values, rNlark

Next

Feedback, Suspend End Block

A 37-year-old Cambodian woman presents to the emergency room with acute onset of left-sided weakness. She has been experiencing progressive exertional dyspnea. nocturnal cough and occasional hemoptysis over the past six months. She also describes frequent episodes of palpitations and irregular heart beats. ii'Vhich of the following is the most likely diagnosis?

1

3

5

7

• 9 • 10

• 11 • 12 • 13 ▪ 14 • 15 ▪ 16 •

• 17

• 19 ▪ ARM

21 22

• 23 • 24 • 25 ▪ 26

27 28 I

• 29 • 30

• 32 ■

• 33 ▪ 34-

I 37

39 • 40

41 42

• A_ Mitral stenosis

Hypertrophic cardiomyopathy

• C_ Aortic insufficiency

• D_ Wolf-Parkinson-White syndrome

• E Primary pulmonary hypertension

1

3

5

7 • a

• 9 • 10

• 11 • 12 • 13 ▪ 14 • • 15 ▪ 16 • • 17

• 19 • ARM

21 —1 22

• 23 • 24 • 25 ▪ 26

27 28 I

▪ 29 • 30

• 32 ■

• 33 ▪ 34-

• 36 I 37

38 I • 39

41 • 42

Item: 39 of 44

Mr Mark 127

Previous

Next

Lab Values. Notes . Calculator.

A 65-year-old female is admitted to the hospital with increasing shortness of breath, weight gain and lower extremity edema. She has a history of hypertension, nonischemic cardiomyopathy with an ejection fraction of 30%, and hyperlipidemia. Her home medications include oral aspirin, digoxin, furosemide, metoprolol, lisinopril and atorvastatin. She is started on intravenous furosemide_ On day three of hospitalization telemetry reveals six beats of wide complex ventricular tachycardia_ Physical examination now shows decreased leg edema and clear lungs_ Which of the following is the most appropriate next step in the management of this patient's tachycardia?

• A Add spironolactone

O B. Add metolazone

C. Measure serum electrolytes

• D. Discontinue atorvastatin

• E. Discontinue metoprolol

Feedback, Suspend

End Block .

1

1 6 3

5

• 7

• 9

• 10 • 11 • 12 • 13 • 14 • • 15 • 16 •

• 17

• 19

21 22

• 23 • 24 • 25 • 26

27

I • 29

• 1 • 32 ■

• 33 • 34-

• 36 • 37

39 a 40

• 30

Item: 40 of 44

r Mark 117

Previous

Next

Lab Values. Notes Calculator.

A 14-year-old woman comes to the physician because of diarrhea, nausea, and decreased appetite_ She has had these symptoms for 'I week. She also complains of increasing fatigue and occasional palpitations over the past few days. Her other medical problems include chronic atrial fibrillation and cardiomyopathy. Her cardiac ejection fraction measured 3 months ago showed 40%_ Her medications include furosemide. metoprolol, digoxin, and warfarin_ She has smoked one pack of cigarettes daily for the past 50 years. Her blood pressure is 140/90 mm Hg and pulse is 70lmin and irregularly irregular. Lung auscultation reveals scattered wheezes. Her abdomen is soft and nontender, with a liver span of 8 cm and nonpalpable spleen_ There is no ankle edema. Her latest INR measured 2 weeks ago was 2.3. ..1%.`hich of the following is the most appropriate initial test in this patient?

• A_ Echocardiography

• B_ Chest x-ray

C. C. Pulmonary function testing

O D. Thyroid function testing

C. E. Blood drug level

O F. Upper gastrointestinal endoscopy

G. ECG stress test

Feedback, Suspend

End Block .

127 Notes Calculator .

Item: 41 of 44 Lab Values,

r Mark Previous Next

Suspend Feedback, End Block .

A 59-year-old man comes to visit a friend in the hospital and collapses in the parking lot. He had been feeling unwell all day due to vague chest discomfort. A bystander witnesses his collapse. finds no pulse. and immediately calls for help. Which of the following is the most important factor for survival in this patient?

1

3

5

7

• 9

• 10

• 11

• 12

• 13

▪ 14 • -

• 15

▪ 16 • • 17

• 19

22

• 23

• 24

• 25

▪ 26

27

28 I

▪ 29 • 30

• 32 ■

• 33

I 34

• 36

• 38 ■

I 39

I • 40

• 41

• 42

C. A. Time to rescue breathing

• E Time to rhythm analysis and defibrillation

C. C. Time to endotracheal intubation

D. Time to epinephrine injection

• E_ Time to cardiac catheterization lab

Item: 42 of 44 rNlark Previous ! Next

Feedback, Suspend End Block

A 64-year-old man comes to the physician due to shortness of breath and abdominal distension He %,,.as treated for Hodgkin lymphoma with radiation and chemotherapy 18 years ago and was told that he was c.urecl The patient drinks alcohol on a regular basis. His temperature is 36.7 C (98 F). blood pressure is 120/76 mm Hg. pulse is 92/min, and respirations are 201min. Neck examination shows jugular venous pulsations 9 cm above the sternal angle. The abdomen is distended with a positive fluid wave. The liver edge is palpated 5 cm below the right costal edge. There is bilateral lower-extremity pitting edema. Initial laboratory results are as follows:

Serum creatinine 0.8 mgldL Albumin 4.0 g/dL Total bilirubin 1.0 Prothrombin time 11 sec

Which of the following is the most likely cause of this patient's condition?

1

3

5

7 • a

• 9

• 10

• 11

• 12

• 13

▪ 14 • • 15

▪ 16 • • 17

• 19

22

• 23

• 24

• 25

▪ 26

27

28 I

▪ 29 • 30

• 32 ■

• 33

▪ 34-

I 37

I 39

I• 40

41

I 42

Lab Values. Notes

!Ili

Calculator.

• A. Inelastic pericardium

• 13_ Portal vein compression

• C_ Portal vein thrombosis

O a Thoracic duct obstruction

• E Urinary protein loss

• F. Venous valve incompetence

117 Notes . Calculator.

Item: 43 of 44 Lab Values.

r Mark Previous Next

Feedback, Suspend End Block

A 43-year-old woman with a past medical history of bronchial asthma comes to the physician for evaluation of high blood pressure. On her previous 2 visits, her blood pressure has been 154/88 and 150/92 mm Hg. She has no headaches or complaints. The patient uses fluticasone and as-needed albuterol for her chronic asthma She does not smoke or use alcohol and has no known drug allergies_ Her family history is significant for myocardial infarction in her father at age 65_ She exercises regularly and has followed a low-salt diet over the last 2 months_ The patient's blood pressure during this visit is 157/93 mm Hg and pulse is 69/min. Her body mass index is 27 kgim2_ The heart and lung examinations are unremarkable_ No vascular bruits are heard. and peripheral pulses are 2+ and symmetrical_ Electrocardiogram shows normal sinus rhythm. Complete blood count and chemistry panel are normal_ Which of the following is the most appropriate next step in management of this patient?

1

1 6 3

5

• 7

• 9

' 10 i 11 i 12 • 13 • 14 • 15 • 16 •

• 17

• 19

21 22

• 23 • 24 • 25 • 26

27

I • 29

• 1 i 32 ■

• 33 ' 34-

• 36 • 37

• 39 ' 40 ' 41 i 42

• A. 24-hour urine cortisol excretion

• 8_ Lipid panel and urinalysis

• C. Plasma renin activity

• D_ Renal ultrasound

• E_ Thyroid-stimulating hormone and urine metanephrines

' 30

1

3

5

7 • a

• 9

• 10

• 11

• 12

• 13

▪ 14 • • 15

▪ 16 • • 17

• 19

22

• 23

• 24

• 25

▪ 26

27

28 I ▪ 29

• 30

• 32 ■

• 33

▪ 34-

• 36

• 37

I 39

• 40

41

42

Item: 44 of 44

r Mark 117

Previous

Next

Lab Values. Notes Calculator .

A 64-year-old nondiabetic man complains of cramping pain in his right thigh after walking 2 blocks. The pain goes away once he stops and rests for several minutes. He is an ex-smoker with 35-pack-year history. He has no history of heart disease or stroke. On physical examination, the femoral pulses are normal. but the popliteal and dorsalis pedis pulses are diminished on the right leg_ His ankle brachial index is 0.98 on the left and 172 on the right. He is started on low-dose aspirin and lipid-lowering therapy. liAbirhich of the following is the best initial management for this patient?

• Arteriography followed by surgical bypass procedure

• 13. Computed tomography angiography followed by stenting

• C_ Exercise therapy

Pentoxifylline

• E. Warfarin

Feedback, Suspend

End Block

End Block 1.1

Feedback,

Effect of maneuvers on hypertrophic cardiomyopathy

Change in murmur intensity

Valsalva (straining phase)

Abrupt standing (from sitting or supine

position)

Nitroglycerin administration

t Preload

Physiologic effect

• 1 2

3

I 5

• 7

• a 9

• 10

• 11

• 12

• 13

• 14

• 15

• 16

• 17

• 19

• ARM

21

22

• 23

• 24

• 25

• 26

27

• 29

• 1

' 32 ■

• 33

• 34

' 36

• 37

• 39

• 40

I

• 30

Item: 6 of 44 Mark Previous Next

i • 1270 ..... . Lab Values Notes Calculator.

44.

A 27-year-old man has a history of repeated attacks of dyspnea upon exertion_ He does not use tobacco, alcohol, or illicit drugs. The patient's blood pressure is 130/81 mm Hg and pulse is 78/min. On examination, there is no peripheral edema or jugular venous distension. His lungs are clear to auscultation_ There is a IIINI ejection-type systolic murmur along the lower left sternal border that decreases in intensity when the patient moves from the standing to the squatting position. .1"1.1hich of the following is true regarding the inheritance of this disease?

A_ Autosomal dominant [67%]

8_ Autosomal recessive [9%]

C_ Multifactorial [1856]

ID_ X-linked dominant [3'36]

E X-linked recessive [4%]

Explanation:

Effect of maneuvers on hypertrophic cardiomyopathy

Physiologic effect Change in murmur

intensity

Valsalva (straining phase)

Preload t Abrupt standing

(from sitting or supine position)

Nitroglycerin administration

Sustained hand grip I Afterioad

Squatting (from standing position)

t Afterload & f preload

Passive leg raise t Preload

QUWarld

This patient's clinical presentation is consistent with hypertrophic cardiomyopathy (HCM). Many patients

with HCM are asymptomatic and are detected when an abnormal murmur or electrocardiogram (ECG) abnormalities are noted during evaluation for other purposes_ The clinical manifestations of HCM include:

• Exertional dyspnea, chest pain, fatigue, palpitations, presyncope, or syncope

• Harsh crescendo-decrescendo systolic murmur heard best at the apex and lower left sternal border,

with characteristic changes in intensity during physiologic maneuvers

1

2 3

4 II

II • 36

5

I . 7 8

1 . 9 10

12

1 6 13

N 15

17

• 19

21

6 23 24

25

26_11 • 27

• 28

• 29

• 30

• 32

• 39

• 40

41

42

• 33

• 34

44.

1.1 Feedback, End Block

Item: 6 of 44

rNlark OPio

Previous Next

Lab Values, Notes Calculator

r

References:

1. Genetics of hypertrophic cardiomyopathy after 20 years: clinical perspectives.

• 1 2

▪ 3

4 G 5

7

• 9

• 10

▪ 11

• 12

• 13

• 14

• 15

• 16

• 17

▪ 1 • 19

• 2

- 21

• 23

• 24

• 25

• 26

• 27

• 28 I

• 29

• 30

• 32

• 33

▪ 34- • '1=.

• 36

• 37 •

I 39

• 40

• 41

42

Item: 6 of 44

r Mark 1>-

Previous

Next

Lab Values

Notes

Calculator

Sustained hand grip Afterioad

Squatting (from standing position)

Afterload & fi preload

Passive leg raise t Preload

CLIWorld

This patient's clinical presentation is consistent with hypertrophic cardiomyopathy (HM}_ Many patients with HCryl are asymptomatic and are detected when an abnormal murmur or electrocardiogram (ECG) abnormalities are noted during evaluation for other purposes_ The clinical manifestations of Hari include:

• Exertional dyspnea, chest pain, fatigue, palpitations. presyncope. or syncope • Harsh crescendo-decrescendo systolic murmur heard best at the apex and lower left sternal border,

with characteristic changes in intensity during physiologic maneuvers

HM is an autosomal dominant genetic disorder caused by mutations in one of several sarcomere genes encoding the myocardial contractile proteins of the heart_ The two most common mutations (responsible for about 70% of identifiable mutations) occur in the cardiac myosin binding protein C gene and the cardiac beta-myosin heavy chain gene_ First-degree relatives of an affected patient with a known disease-causing mutation can be offered genetic testing to identify the risk for developing HM_

Educational objective: Hypertrophic cardiomyopathy (HM) is an autosomal dominant genetic disorder caused by mutation in one of the several genes encoding the myocardial contractile proteins of the cardiac sarcomere_ Mutations in the cardiac myosin binding protein C gene and cardiac beta-myosin heavy chain gene are responsible for about 70% of identifiable mutations in patients with

Copyright © Morld Last updated: [11/11/2014]

1.1 Feedback, End Block

44. Media Exhibit

End Block 1.1 Feedback,

• 2 3 4

• 5

▪ 7 . 8

• 9 ' 10 i 11 i 12 • 13

• 15

• 17 • 18 • 19 • 211.11.

21

• •

23 24

• 25 • •

26 27

• 28 ' '

29 30

• 32 • 33 •

' 36

' 38 • .39 ' 40 ' 41

Item: 6 of 44 rMark Previous Next

117 Lab Values Notes Calculator.

Hypertrophioyopathy

Hypertrophic cardionwopathy

▪ Leftricular hypertrophy: Tall R wave in aVL + Deep S wave iri V3 (Cornell criteria)

▪ Relization changes in anterolateral leads (I, aVL, V4, V5, V6)

eiLISMLEWC

I 1 2

3

5

7

9

' 10 i 11

i 12 • 13

14 • • 15

16 •

• 17

• 19

21

22

• 23

• 24

• 25

• 26

27

I • 29

' 30

• 1

i 32 ■

• 33

' 34- El

' 36

• 37

• 39

' 40

41

42

Item: 2 of 44

rNlark CP,*

Previous Next Lab Values. Notes Calculator

A 32-year-old man comes to the physician because of 6 months of progressive weakness and exertional dyspnea. His past medical history is significant for a knife injury. with a stab wound to the right thigh 10 months ago. He saw a doctor, had suturing, and has a healed scar. He has been arrested several times for robbery. He regularly drinks alcohol and occasionally smokes crack. His younger brother died of cystic fibrosis. He is afebrile, and his blood pressure is 160160 mm Hg and pulse is 10Gimin. His right leg feels warmer and appears more flushed compared to his left leg. Carotid upstroke is brisk. The point of maximal impulse is displaced to the left, and a soft systolic murmur is heard over the cardiac apex that does not change with the Valsalva maneuver. Which of the following is the most likely cause of this patient's symptoms?

A. Decreased cardiac output [19%]

B. Hvpertrophic cardiomyopathy [14%]

• C_ Increased cardiac preload [36°A]

D_ Papillary muscle dysfunction [20%]

E. Pulmonary hypertension 0%]

Explanation:

Congenital .1 Acquired

Patent ductus arteriosus Trauma

Angiomas latrogenic (e.g., femoral catheterization)

Pulmonary AVF Atherosclerosis (e.g., aortocaval fistula)

C115 AVF Cancer

This patient presents with signs/symptoms of high-output heart failure after a traumatic injury to his right thigh, likely causing a symptomatic arteriovenous fistula (AVF) of the popliteal or iliac artery at the site of injury. Symptomatic AVF can be congenital or acquired (as shown in the table above) and creates an abnormal connection between the arterial and venous systems that bypasses the capillary beds. Shunting of a large amount of blood through the fistula decreases systemic vascular resistance. increases cardiac preload, and

1.1 Feedback, End Block

1P4 Notes

Item: 2 of 44 Calculator Lab Values, Next

rNlark Previous

1>-

End Block 1.1

Feedback,

44.

This patient presents with signs/symptoms of high-output heart failure after a traumatic injury to his right thigh, likely causing a symptomatic arteriovenous fistula (AVF) of the popliteal or iliac artery at the site of injury_ Symptomatic AVF can be congenital or acquired (as shown in the table above) and creates an abnormal connection between the arterial and venous systems that bypasses the capillary beds_ Shunting of a large amount of blood through the fistula decreases systemic vascular resistance, increases cardiac preload, and increases cardiac output Clinical signs include widened pulse pressure, strong peripheral arterial pulsation (e_g_, brisk carotid upstroke), systolic flow murmur (as in this patient), tachycardia, and usually flushed extremities_ The left ventricle hypertrophies. and the point of maximal impulse is displaced to the left. An EGG usually shows left ventricular hypertrophy_

In patients with AVF and significant AV shunting, there is a compensatory increase in the heart rate and stroke volume to meet the oxygen requirements of the peripheral tissues. Normal hearts are usually capable of increasing the stroke volume and cardiac output without significant problems. However, cardiac function can occasionally decompensate over a period of time and result in cardiac failure. Such patients are considered to have heart failure (despite their higher cardiac output) because the circulation is unable to meet the oxygen demand of the peripheral tissues. Other causes of high-output cardiac failure include thyrotoxicosis, Paget disease, anemia and thiamine deficiency_ Doppler ultrasonography is the preferred test to diagnose an AVF in the extremity. and surgical therapy is indicated for a large AVE (as in this patient)_

(Choice A) Decreased cardiac output in patients with ischemic heart disease. hypertension, or cardiomyopathy is associated with normal or decreased pulse pressure. weak carotid upstroke, and cold and pale extremities_

(Choice B) The clinical findings of heart failure due to hypertrophic cardiomyopathy are more or less similar to those of low-output cardiac failure_ The patients have both systolic (decreased cardiac output) and diastolic (decreased ventricular filling) dysfunction_ Patients usually have a murmur that is crescendo-decrescendo and increases with the Valsalva maneuver_

(Choice 0) Papillary muscle dysfunction is one of the causes of mitral (and tricuspid) regurgitation, which usually results from ischemic heart disease_

(Choice E) Patients with pulmonary hypertension usually present with weakness, angina, syncope (or presyncope). and dyspnea_ Physical examination reveals jugular venous distention, reduced carotid upstroke, right ventricular lift, loud pulmonary second heart sound, and tricuspid regurgitation_

Educational objective:

1 2

3

5

7

• 9 • 10 ▪ 11 ▪ 12 • 13 ▪ 14 1. • 15 ▪ 16 •

• 17

• 19 ▪ ARM

21 22

• 23 • 24 • 25 ▪ 26

27 28 I

• 29 • 30 • 1 ▪ 32 ■

• 33 ▪ 34- El

▪ 36 • 37

I 39 • 40

41 42

3

5 G

127 Notes Calculator.

Item: 2 of 44 Lab Values.

r Mark Previous Next

Copyright CI Morld Last updated: [8/11/2014] a 1.1

Feedback, End Block

In patients with AVF and significant AV shunting. there is a compensator), increase in the heart rate and stroke volume to meet the oxygen requirements of the peripheral tissues_ Normal hearts are usually capable of increasing the stroke volume and cardiac output without significant problems However, cardiac function can occasionally decompensate over a period of time and result in cardiac failure_ Such patients are considered to have heart failure (despite their higher cardiac output) because the circulation is unable to meet the oxygen demand of the peripheral tissues. Other causes of high-output cardiac failure include thyrotoxicosis, Paget disease, anemia. and thiamine deficiency. Doppler ultrasonography is the preferred test to diagnose an AVF in the extremity. and surgical therapy is indicated for a large AVF (as in this patient_

(Choice A) Decreased cardiac output in patients with ischemic heart disease, hypertension, or cardiomyopathy is associated with normal or decreased pulse pressure, weak carotid upstroke, and cold and pale extremities.

(Choice B) The clinical findings of heart failure due to hypertrophic cardiomyopathy are more or less similar to those of low-output cardiac failure_ The patients have both systolic (decreased cardiac output) and diastolic (decreased ventricular filling) dysfunction_ Patients usually have a murmur that is crescendo-decrescendo and increases with the Valsalva maneuver.

(Choice 0) Papillary muscle dysfunction is one of the causes of mitral (and tricuspid) regurgitation_ which usually results from ischemic heart disease_

(Choice E) Patients with pulmonary hypertension usually present with weakness, angina. syncope (or presyncope), and dyspnea. Physical examination reveals jugular venous distention, reduced carotid upstroke, right ventricular lift. loud pulmonary second heart sound, and tricuspid regurgitation_

Educational objective: Arteriovenous fistula causes high-output cardiac failure by shunting the blood from the arterial to venous side, thereby increasing cardiac preload_ The patient develops heart failure despite maintaining a normal or high cardiac output because the circulation is unable to meet the oxygen demand of the peripheral tissues.

References:

1. Endovascular repair of a traumatic popliteal arteriovenous fistula

2. Aortocaval and iliac arteriovenous fistulas: recognition and treatment

1 2

7

• 9 • 10 • 11 • 12 • 13 • 14 • 15 • 16 • 17 ▪ 1 • 19 • 2 - 21 • 22 • 23 • 24 • 25 • 26 • 27

• 28 I

• 29 • 30

• 32 • 33 ▪ 34- • K

• 36 • 37 •

I 39 40

• 41 • 42

1

2

6

7

• 9

• 10

• 11

• 12

• 13

• 14

• 15

• 16 •

• 17

• 19

21

22

• 23

• 24

• 25

• 26

27

I • 29

• 1

• 32 ■

• 33

• 34-

• 36

• 37

• 39

• 40

I

• 30

Item: 3 of 44 Mark 117

Previous Next Lab Values. Notes . Calculator.

44.

A 35-year-old woman who has recently emigrated from Asia presents to the emergency room with acute onset of dyspnea_ She denies any cough, chest pain. or fever. She has a history of rheumatic heart disease as a teen. On examination. she has an irregular pulse of 97/min. blood pressure of 125/75 mm of Hg and temperature of 37.2°C (98.9°F). The first heart sound is loud and a mid-diastolic rumble is heard at the apex_ Crackles are present in both lung fields_ ECG shows an irregularly, irregular heart rhythm and the absence of 'F" waves.. Which of the following is the most likely cause of this patient's abnormal heart rhythm?

A_ Left atrial dilatation [81%]

H. Right atrial dilatation [8%]

C. Left ventricular dilatation [5%]

D. Left ventricular hypertrophy [4%]

E. Pulmonary hypertension [2%]

Explanation:

This patient has mitral stenosis as a consequence of rheumatic heart disease_ The mitral stenosis has in turn led to dilatation of her left atrium and resulted in atrial fibrillation_ Rheumatic fever is more common in patients from developing countries. VVhile rheumatic fever can cause damage to any heart valve, mitral stenosis is the most common valvular manifestation_ The auscultative findings of a loud SI and a mid diastolic rumbling murmur at the apex are classic for mitral stenosis_ The pressure in the left atrium can become greatly elevated in order to compensate for the high resistance of the stenotic mitral valve. The increased pressure in the left atrium causes it to dilate, which predisposes to the development of atrial fibrillation_ In fact. up to 70°./6 of patients with mitral stenosis develop atrial fibrillation_ The ECG with an irregularly irregular rhythm and loss of f' waves describes atrial fibrillation. Atrial fibrillation causes a lack of an "atrial kick". which could cause worsening flow through the stenotic mitral valve and increased congestion in the lungs. thus leading to the patient's acute onset of dyspnea_

I rr

1.1 Feedback, End Block

44.

• 2

• 22

•1

6

7

• 9

• 10

• 11

• 12

• 13

• 14

• 15

• 16 •

• 17

• 19

21

• 23

• 24

• 25

• 26

27

I • 29

• 1

• 32 ■

• 33

• 34-

• 36

• 37

• 39

• 40

• 30

Item: 3 of 44 Mark -c2:1

117 Previous Next

Lab Values. Notes . Calculator.

D. Left ventricular hypertrophy [4%]

E Pulmonary hypertension [2%]

Explanation:

This patient has mitral stenosis as a consequence of rheumatic heart disease. The mitral stenosis has in turn led to dilatation of her left atrium and resulted in atrial fibrillation_ Rheumatic fever is more common in patients from developing countries_ While rheumatic fever can cause damage to any heart valve, mitral stenosis is the most common valvular manifestation_ The auscultative findings of a loud S1 and a mid diastolic rumbling murmur at the apex are classic for mitral stenosis_ The pressure in the left atrium can become greatly elevated in order to compensate for the high resistance of the stenotic mitral valve. The increased pressure in the left atrium causes it to dilate, which predisposes to the development of atrial fibrillation_ In fact, up to 10% of patients with mitral stenosis develop atrial fibrillation_ The ECG with an irregularly irregular rhythm and loss of 'P' waves describes atrial fibrillation_ Atrial fibrillation causes a lack of an "atrial kick", which could cause worsening flow through the stenotic mitral valve and increased congestion in the lungs, thus leading to the patient's acute onset of dyspnea.

(Choice B) Rheumatic fever can affect any valve, including the tricuspid valve_ Tricuspid stenosis could lead to right atrial enlargement, which in turn would increase the patient's propensity to develop atrial fibrillation. However. the murmur of tricuspid stenosis would be most prominent at the left lower sternal border and symptoms of right sided heart failure (peripheral edema. hepatomegaly, jugular venous distention) would predominate.

(Choice C & 0) The left ventricle is usually normal in patients with mitral stenosis from rheumatic disease unless they have associated aortic valve disease_

(Choice E) Mitral stenosis can result in secondary pulmonary venous hypertension_ However, mitral stenosis is the primary disease process in this patient, and the cause of atrial fibrillation_

Educational objective: Up to 70°.....6 of patients with mitral stenosis will develop atrial fibrillation because of the significant left atrial dilatation.

Copyright (D UWorld Last updated: [8/221201 4]

1.1 Feedback, End Block

Em

• 7

• 9 • 10 • 11 • 12 • 13

• •

14 15 16

• 17 • 1E1. • 19

21!I mr.2

• 2 • 23 • 24 • 25 • 26 1 • 27

• 2B

• 29 • 30

• 33 • 34-

• 32

• 36 • 37

• 39

• 40

• 41

• 42

Item: 4 of 44 r Mark -441 Previous

1>- Next

CPO !Wm

Notes Calculator

A

A 64-year-old white female presents for evaluation of two weeks of decreased appetite and nausea_ She also notes occasional palpitations, which have been especially prominent over the past two days. Her medical history is significant for an anterior wall myocardial infarction one year ago and secondary congestive heart failure with left ventricular systolic dysfunction_ Her current medications include aspirin.. digoxin, furosemide, enalapril and metoprolol_ On physical examination, her blood pressure is 120/80 mm Hg, pulse is 106/min and respirations are 15.1min_ The remainder of her exam is unremarkable_ Chest x-ray shows an enlarged cardiac silhouette and normal lung fields. On laboratory testing, her digoxin level is twice the upper limit of normal_ You order an EKG_ Which of the following arrhythmias is most specific for digitalis toxicity?

A. Atrial flutter [6%]

B. Atrial fibrillation [139/0]

C_ rylobitz type II second-degree AV block [24%]

D. Atrial tachycardia with AV block []

E Multifocal atrial tachycardia [12%]

Explanation:

Atrial tachycardia with AV block is the arrhythmia most specific for digitalis toxicity_ Digitalis can increase ectopy in the atria or ventricles, which can lead to atrial tachycardia. Atrial tachycardia is distinguished from atrial flutter by its somewhat slower atrial rate (150-250 bpm as opposed to 250-350 bpm). P-waves are present, but may appear different from the p-waves normally seen when conduction originates in the SA node_ In atrial tachycardia, the closer the ectopic focus is to the SA node, the closer the resemblance of its p-waves to normal p-waves originating from the SA node_ In addition to causing ectopic rhythms. digitalis can also increase vagal tone and decrease conduction through the AV node, potentially causing AV block. Since it is rare for both ectopy and AV block to occur at the same time, when they do, the combination is fairly specific for digitalis toxicity_

(Choice A) Digitalis toxicity does not commonly cause atrial flutter_

(Choice 6) There are many potential causes of atrial fibrillation, but digitalis toxicity is not one of them. Digitalis can be used to increase vagal tone and is sometimes used to treat atrial fibrillation if beta-blockers or calcium channel blockers have not been completely effective_

1.1 Feedback, End Block

1 2

1 6 3

5

• 7

• 9

• 10 • 11

• 12 • 13 • 14 • 15 • 16 •

• 17

• 19

21

22

• 23

• 24

• 25

• 26

27

I • 29

• 1

• 32 ■

• 33

• 34-

• 36

• 37

• 39

• 40

• 30

Item: 4 of 44 Mark -c21

1>- CP,*

Previous Next

Lab Values. Notes Calculator

A_ Atrial flutter [/o]

13_ Atrial fibrillation [13%]

C_ Mobitz type II second-degree AV block [24%]

4.0 D_ Atrial tachycardia with AV block [44%]

E. Multifocal atrial tachycardia [12%]

Explanation:

Atrial tachycardia with AV block is the arrhythmia most specific for digitalis toxicity_ Digitalis can increase ectopy in the atria or ventricles, which can lead to atrial tachycardia_ Atrial tachycardia is distinguished from atrial flutter by its somewhat slower atrial rate (150-250 bpm as opposed to 250-350 bpm). P-waves are present. but may appear different from the p-waves normally seen when conduction originates in the SA node. In atrial tachycardia, the closer the ectopic focus is to the SA node, the closer the resemblance of its p-waves to normal p-waves originating from the SA node. In addition to causing ectopic rhythms, digitalis can also increase vagal tone and decrease conduction through the AV node, potentially causing AV block_ Since it is rare for both ectopy and AV block to occur at the same time. when they do, the combination is fairly specific for digitalis toxicity_

(Choice Al Digitalis toxicity does not commonly cause atrial flutter_

(Choice B) There are many potential causes of atrial fibrillation, but digitalis toxicity is not one of them. Digitalis can be used to increase vagal tone and is sometimes used to treat atrial fibrillation if beta-blockers or calcium channel blockers have not been completely effective_

(Choice C) Digitalis would not be expected to cause Pylobitz type II second-degree AV block, as this involves pathology of the conduction system below the AV node.

(Choice E) Multifocal atrial tachycardia is rarely associated with digitalis use. It is more commonly a consequence of pulmonary disease.

Educational objective: Digitalis toxicity causes increased ectopy and increased vagal tone. Atrial tachycardia with AV block occurs from the combination of these two digitalis effects. and is relatively specific for digitalis toxicity_

Copyright (D UWorld Last updated: [8/22/2014]

44.

1.1 Feedback, End Block

Mechanical complication of

acute MI

Time course

Coronary artery typically involved

Clinical findings Echocardiography

Right ventricular

failure

▪ Hypotension & clear lungs

▪ Kussrnaul sign

Acute RCA • Hypokinetic RV

42

2

• 10

• 12

▪ 14 •

▪ 16 •

▪ ARM

22

• 24

▪ 26

I

• 30

• 32 ■

• 34-

• 36

• 40

Item: 5 of 44 Mark

Previous

Next

Lab Values. Notes Calculator.

44.

A 45-year-old mildly overweight man admitted to the hospital 5 days ago with anterior wall myocardial infarction develops sudden onset of left-sided chest pain_ He underwent cardiac catheterization on admission that showed complete occlusion of the left anterior descending artery_ However, no revascularization was performed due to late presentation_ He has had no dyspnea, extremity swelling, or palpitations since admission_ His past medical history is significant for diabetes mellitus type 2 and hyperlipidemia_ He appears agitated and restless_ Two minutes later, he is unresponsive_ His pulse is not palpable and the electrocardiogram shows sinus tachycardia of 30/min. Which of the following is the most likely diagnosis?

A. Acute pericarditis [1%]

B. Aortic dissection [1%]

C. Interventricular septum rupture [FA]

D. Papillary muscle rupture [6%]

E. Pulmonary infarction [2%]

F. Recurrent ischemia. [5%]

G. Right ventricular infarction [2%]

H_ Ventricular aneurysm [3%]

• I_ Ventricular free wall rupture [13%]

Explanation:

1.1 Feedback, End Block

USHLD'oodel,ILC

F. Mark Item: 5 of 44 Lab Values. Notes Calculator. Previous Next

111. Feedback, End Block

• 30 ■

Mechanical complication of

acute MI

Time urse

co

i

Coronary artery typically involved

i

Clinical findings

i

Echocardiography

.

Right ventricular

failure Acute

i

RCA

.

• Hypotension ei clear lungs

. Kuss:maul sign i

• Hvpokinetic RV

.•

P apillary sc mu le rupture

-+.

Acute & within

3-5 days

RCA

. ...

• Acute, severe pulmonary edema

• New holosystolic murmur

• Severe mitral regurgitation

with flail leaflet

Inter ventricular septum

rupture/defect

Acute & within 3-5 days

;

• LAD —>

apical tal sep rupture

• RCA —> basal septa! rupture

.

- Shock & chest p ain

+ New holosystolic murmur

- Biventricular failure

i

• Left-to-right shunt at level of ventricle

• Step-up oxygen level between right atrium &

ventricle .•

Free wall rupture

Within

days- 2 weeks

first 5• LAD

• Shock & chest

pain Jugular venous distention

- Distant heart

sounds

• Pericardial effusion with tamponade

This patient's clinical presentation suaoests left ventricular (LV) free wall rupture_ LV free wall rupture is a

1

2

a 4

5

I. 8 • g

• 10

• 11

• 12

' 13

' 15

• 16 MI

• 17

• 19

▪ ARM 21

22

• 23

' 24

' 25

25

• 27

• 28

• 29

• 32 111

• 33

• •4

' 36

• 38

• .39

• 40

• 41

42

II

1

2 ▪ a

4

7

9

• 10 • 11 • 12 • 13 • 14 1. • 15 • 16 •

• 17

• 19 ▪ ARM

21 22

• 23 • 24 • 25 • 26

27

I • 29

• 32 111 • 33 • 34- El

• 36 • 37

• 39 • 40

• 30

Item: 5 of 44 Mark

SI Lab Values!

:iii

Previous Next Notes Calculator 1

44.

This patient's clinical presentation suggests left ventricular (LV) free wall rupture_ LV free wall rupture is a mechanical complication of transmural myocardial infarction (Ml) that usually occurs within the first 5 days to 2 weeks after ML The majority of ventricular free wall ruptures occur after anterior MI, as in this patient LV free wall rupture occurs in <1?./0 of acute MIs due to increased use of early reperfusion therapy and medical management Patients usually present with sudden onset of chest pain and profound shock.

Abrupt LV rupture often leads to hemopericardium and eventual cardiac tamponade_ Blood in the pericardial sac compresses the left ventricle and decreases stroke volume, resulting in hypotension with compensator sinus sinus tachycardia_ The severe mechanical compromise can rapidly progress to pulseless electrical activity (PEA) with the electrocardiogram (ECG) showing low voltage from the ensuing cardiac tamponade_ LV free wall rupture should be suspected in patients with PEA after recent first Ml and no signs of heart failure_ Rapid diagnosis with echocardiogram, supportive care, pericardiocentesis, and possible surgical repair are required to save the patient's life_

(Choice A) Acute peri-infarct pericarditis can occur within 1-3 days after an Ml and can present with a pericardial friction rub with or without chest pain_ It is usually self-limited and resolves with supportive care_ Dressler syndrome is post-MI pericarditis occurring weeks to months after the ML Patients present with pleuritic chest pain and pericardial friction rub and usually improve with nonsteroidal anti-inflammatory drugs_

(Choice B) Aortic dissection can cause acute onset of chest pain and lead to acute pericardial tamponade if the dissection involves the aortic root However, it is not a typical complication after ML

(Choice C) Interventricular wall rupture is pathologically similar to ventricular free wall rupture and both have a peak incidence around 5 days after infarction_ However, inter-ventricular wall rupture causes a ventricular septal defect and not pericardial tamponade_ Patients typically present with sudden onset of hypotension, congestive heart failure (predominantly right heart failure), and a loud holosystolic murmur heard best at the lower left sternal border_

(Choice 0) Papillary muscle rupture occurs 3-5 days after infarction and can cause hypotension secondary to severe acute mitral regurgitation (holosystolic murmur at the apex). Although acute in onset, it typically presents with hypotension and pulmonary edema It is unlikely to cause the rapid decompensation seen in this patient

(Choice E) Acute massive pulmonary embolism (not pulmonary infarction) can cause hypotension and syncope. leading to pulseless electrical activity in some patients_ However, this patient's recent Ml makes LV free wall rupture more likely_

1.1 Feedback, End Block

Previous Next

▪ 1

2

▪ a

-6- • 7

9

• 10

• 11

• 12

• 13

• 14

• 15

• 16

• 17

• 19

• 2

- 21

• 22

• 23

• 24

• 25

• 26

• 27

• 28 I

• 29 • 30

• 31

• 32

• 33

• 34- • •

• 36

• 37 •

• 39

• 40

• 41

42

SI Lab Values,

1P4

Notes

: :iii

Calculator,

Item: 5 of 44

rNlark

lower left sternal border.

(Choice 0) Papillary muscle rupture occurs 3-5 days after infarction and can cause hypotension secondary to severe acute mitral regurgitation (holosystolic murmur at the apex). Although acute in onset. it typically presents with hypotension and pulmonary edema_ It is unlikely to cause the rapid decompensation seen in this patient.

(Choice E) Acute massive pulmonary embolism (not pulmonary infarction) can cause hypotension and syncope, leading to pulseless electrical activity in some patients_ However, this patient's recent Ml makes LV free wall rupture more likely_

(Choice F) Recurrent cardiac ischemia (or post-infarction angina) presents with recurrent chest pain and new or worsening ECG changes of ischemia (eg, ST elevation). It can cause ventricular tachycardia/fibrillation. which can degenerate further into asystole_

(Choice G) Right ventricular infarction occurs in patients with acute inferior wall Ml due to occlusion of the proximal right coronary artery Patients typically present with hypotension or shock, jugular vein distension, and clear lung fields_ ECG would show either an inferior Ml and/or ST elevation in leads 4R— 3R_

(Choice H) Ventricular aneurysm occurs as a late (weeks to months) complication of acute ST- segment elevation. It is characterized by scarred or fibrotic myocardial wall as a result of healed transmural ML Patients may present with heart failure. refractory angina, ventricular arrhythmias, or systemic arterial embolism due to mural thrombus formed within the aneurysm_

Educational objective: Rupture of the ventricular free wall is a mechanical complication that usually occurs within 5 days to 2 weeks after an acute myocardial infarction (usually anterior). Patients present with acute onset of chest pain and profound shock, with rapid progression to pulseless electrical activity and death_

References:

1. Left ventricular free-wall rupture in acute myocardial infarction: a blow-out type case.

2. Reliability of electromechanical dissociation in the diagnosis of left ventricular free wall rupture in acute myocardial infarction.

Copyright © Morld Last updated: [11 /25/2014]

1.1 Feedback, End Block

1 2

1 6 3 I

5

• 7

• 9 • 10 • 11 • 12 • 13 • 14 • • 15 • 16 • • 17 • • 19

21 22

• 23 • 24 • 25 • 26

27

I • 29

• 1 • 32 ■

• 33 • 34- El

• 36 • 37

• 39 • 40

• 30

Item: 6 of 44 Mark 117

Previous Next Lab Values, Notes Calculator.

44.

A 40-year-old man comes to the emergency department with slowly progressive headaches, dyspnea. and blurry vision over the last 2 days. His other medical problems include hypertension treated with hydrochlorothiazide and lisinopril for the past 2 years. He has been noncompliant with his medical regimen and physician follow-ups. He smokes 1 pack a day and occasionally consumes alcohol. On initial evaluation. his blood pressure is 220/140 mm Hg and pulse is 75/min. Which of the following establishes the diagnosis of malignant hypertension in this patient?

A. Left ventricular hypertrophy on electrocardiogram [1 °A]

B. Reduced glomerular filtration rate [1%]

• C_ Papilledema and retinal hemorrhages [61%]

D. Diastolic blood pressure >120 mm Hg [31%]

E. Pulmonary congestion on chest x-ray [1 96]

Explanation:

Hypertensive complications

Hypertensive

urgency

• Severe hypertension (usually -18.01120 mm Hg) with no

symptoms or acute end-organ damage

Hypertensive

emergency

Severe hypertension with acute, life-threatening, end-organ

cornplicat ons

• Malignant hypertension: Severe hypertension with retinal

hemorrhages, exudates, or papilledema

• Hypertensive encephalopathy: Severe hypertension with

cerebral edema & non-localizing neurologic symptoms & signs

LISMLEINwld, LLC

1.1 Feedback, End Block

117 Notes . Calculator .

Item: 6 of 44 Lab Values.

Mark Previous Next

1.1 Feedback, End Block

44.

Hypertensive complications

Hypertensive

urgency

,

. Severe hypertension Mually 180/120 mm Hg) with no

symptoms or acute end-organ damage

.

Hypertensive

emergency

Severe hypertension with acute, life-threatening, end-organ

complications

• Malignant hypertension: Severe hypertension with retinal

hemorrhages, exudates, or papilledema

• Hypertensive encephalopathy: Severe hypertension with

cerebral edema & non-localizing neurologic symptoms & signs L

USIVILEWorld, LLC

This patient's presentation (headaches, dyspnea, and blurred vision) is consistent with hypertensive emergency. Severe hypertension 80/120 mm Hg) can lead to autoregulation failure, vascular endothelial disruption. fibrinous deposition, and narrowed vascular lumen_ Hypertensive urgency is defined as severe hypertension without symptoms or end-organ damage. Hypertensive emergency is defined as marked severe hypertension associated with malignant hypertension or hypertensive encephalopathy_

Malignant hypertension is usually seen in patients with long-standing and uncontrolled hypertension_ It is associated with retinal hemorrhages, exudates, and/or papilledema_ Patients also can develop malignant nephrosclerosis (acute renal failure, hematuria, and proteinuria). However, renal findings are not always present and are not required for the diagnosis of malignant hypertension (Choice By

Hypertensive encephalopathy is associated with cerebral edema due to breakthrough vasodilation from failure of autoregulation_ Patients usually develop insidious onset of headache, nausea, and vomiting followed by non-localizing neurologic symptoms (eg, restlessness, confusion, seizures/coma if untreated)_ Patients also can develop subarachnoid or intracerebral hemorrhage_

(Choice Al Left ventricular hypertrophy can occur as a complication of long-standing uncontrolled hypertension_ However, its presence is not a specific criterion for diagnosing malignant hypertension_

1 2 3 4 5

7 • a

• 9 • 10 • 11 • 12 • 13 ▪ 14 • 15 ▪ 16 •

• 17

• 19 ▪ ARM

21 22

• 23 • 24 • 25 ▪ 26

27 28 I

• 29 • 30

• 32 ■

• 33 ▪ 34-

• 36 • 37

I 39 • 40

41 42

References:

1. Hypertension crisis.

2. Malignant hypertension: ocular manifestations.

Item: 6 of 44

r Mark 117

Previous Next Lab Values. Notes . Calculator.

disruption, fibrinous deposition, and narrowed vascular lumen. Hypertensive urgency is defined as severe hypertension without symptoms or end-organ damage_ Hypertensive emergency is defined as marked severe hypertension associated with malignant hypertension or hypertensive encephalopathy.

Malignant hypertension is usually seen in patients with long-standing and uncontrolled hypertension. It is associated with retinal hemorrhages, exudates, and/or papilledema. Patients also can develop malignant nephrosclerosis (acute renal failure, hematuria, and proteinuria). However, renal findings are not always present and are not required for the diagnosis of malignant hypertension (Choice By

Hypertensive encephalopathy is associated with cerebral edema due to breakthrough vasodilation from failure of autoregulation_ Patients usually develop insidious onset of headache, nausea, and vomiting followed by non-localizing neurologic symptoms (eg. restlessness. confusion, seizures/coma if untreated). Patients also can develop subarachnoid or intracerebral hemorrhage.

(Choice Al Left ventricular hypertrophy can occur as a complication of long-standing uncontrolled hypertension_ However. its presence is not a specific criterion for diagnosing malignant hypertension_

(Choice 0) Blood pressure :M80/120 mm Hg is also seen in hypertensive urgency and is not specific for defining malignant hypertension_

(Choice E) Sudden increases in blood pressure can abruptly increase left ventricular afterload, leading to left ventricular failure and acute pulmonary edema. Although hypertensive emergency can lead to the finding of pulmonary edema on chest x-ray, it is not a specific diagnostic criterion for malignant hypertension.

Educational objective: Hypertensive emergency is defined as marked. severe hypertension associated with malignant hypertension or hypertensive encephalopathy. Malignant hypertension is defined as the presence of severe hypertension (usually M80/120 mm Hg) associated with retinal hemorrhages, exudates. andlor papilledema_ Hypertensive encephalopathy is associated with cerebral edema_

Copyright © Morld Last updated: [11/7/201.El]

1.1 Feedback, End Block

Item: 7 of 44

-<=.1

Previous

Mark Next Lab Values. Notes Calculator .

1

3 44.

▪ 9

A 32-year-old man is brought to the emergency department complaining of sudden-onset, excruciating chest

and neck pain. He has no prior history of dyspnea, chest pain, or lower-extremity edema. He is 6 feet, 5

inches tall with long extremities. His hand joints show significant extensibility. Pectus carinatum deformity is

present Which of the following additional findings is likely in this patient? • 10

• 11

• 12

• 13

• 14 •

• 15

• 16 •

• 17

• A_ Early diastolic murmur [58%]

11 Fixed splitting of the second heart sound [5%]

C. Opening snap [14%]

a Pericardial friction rub [3%]

E. Pulsus par-vus et tardus [16%]

F. Third heart sound [51Y0] • 19

21 Explanation:

22

• 23

• 24

• 25

• 26

27

• 29

• 30

• 32 ■

• 33

• 34-

• 36

41

Clinical features of Marfan syndrome

Skeletal

...-- •

• ,

height

• Joint

Arachinodactyly

Upper-to-lower-body-segment ratiort arrn-to-

ratio

Pectus deformity, scoliosis, or kyphosis

hypermobility

Ocular . Ectopi lentil .

Cardiovascular • Aortic dilatation, regurgitation, or dissection

• Mitral valve prolapse ..

• 37

• 39

• 40

1

3

▪ 9

• 10

• 11

• 12

• 13

• 14

• 15

• 16 • • 17

• 19

21

22

• 23

• 24

• 25

• 26

27

I • 29

• 1

• 32 ■

• 33

• 34-

• 36

• 37

• 39

• 40

• 30

Item: 7 of 44 Mark Previous Next

Lab Values. Notes Calculator

LI 5M LEWorla LLC

This patient appears to have marfanoid habitus with chest and neck pain suggesting an acute aortic dissection_ Pylarfan syndrome is an autosomal dominant disorder due to mutations in the extracellular matrix protein fibrillin-1. Clinical manifestations include cardiac (e.g., aortic dissection, mitral valve prolapse), skeletal (e.g., disproportionately long extremities, increased arm-span-to-height ratio), pulmonary (e.g., spontaneous pneumothorax), and ocular (e.g., ectopia lentil) abnormalities_

All Marfan syndrome patients with acute chest pain require evaluation for acute aortic dissection as nearly 50% of aortic dissections in patients age <4Q are due to the syndrome. Examination may also show >20 mm Hg systolic blood pressure variation between arms. Patients with aortic dissection and/or progressive aortic root dilatation can develop aortic regurgitation_ Examination shows an early decrescendo diastolic murmur heard best along the left sternal border at the third and fourth intercostal space with the patient sitting up_ leaning forward, and holding the breath after full expiration_

(Choice 6) A wide and fixed splitting of the second heart sound is present in patients with atrial septal defect. Holt-Gram (heart-hand) syndrome is associated with both upper-limb defects (e.g., deformities of the radius. carpal bones) and atrial septal defect

(Choice C) Mitral stenosis usually presents with an opening snap in early diastole, loud first heart sound. and a mid-diastolic murmur stemming from turbulent flow across the atrioventricular valves. Mitral valve prolapse. not mitral stenosis, is frequently seen in P...lartan syndrome and causes a mid- to late-systolic murmur_

(Choice 0) Pericardial friction rub is seen with pericarditis_ However, Mar-fan syndrome is not associated with pericarditis.

(Choice E) Pulsus parvus et tardus is an arterial pulse with decreased amplitude and delayed peak_ It is commonly seen in severe aortic stenosis, which is not usually associated with Marfan syndrome_

(Choice F) Third heart sound is a low-frequency diastolic sound that is a normal finding in healthy young adults and an abnormal finding in those with dilated ventricles_ Mitral valve prolapse resulting in regurgitation and heart failure can cause third heart sound_ However, this patient has no symptoms of heart failure_

Educational objective: Aortic dissection is the most dangerous complication of Marfan syndrome. It typically presents as tearing chest pain radiating to the back and neck and must be identified immediately to decrease the risk of rinfk Ant-Fir rani lrrrifafinr is a rnmrrI rafirrn of inrFir rlir•r•nri-inn nrac•arti-c• rni irrni

1.1 Feedback, End Block

Item: 7 of 44

r Mark 127

Previous Next

Lab Values. Notes Calculator. purirciriuuu priuurriariurdx). drILI ULLildr uclupid

All Marfan syndrome patients with acute chest pain require evaluation for acute aortic dissection as nearly 50% of aortic dissections in patients age <40 are due to the syndrome. Examination may also show >20 mm Hg systolic blood pressure variation between arms_ Patients with aortic dissection and/or progressive aortic root dilatation can develop aortic regurgitation_ Examination shows an early decrescendo diastolic murmur heard best along the left sternal border at the third and fourth intercostal space with the patient sitting up_ leaning forward. and holding the breath after full expiration_

(Choice B) A wide and fixed splitting of the second heart sound is present in patients with atrial septal defect. Holt-Gram (heart-hand) syndrome is associated with both upper-limb defects (e.g., deformities of the radius. carpal bones) and atrial septal defect

(Choice C) Mitral stenosis usually presents with an opening snap in early diastole, loud first heart sound. and a mid-diastolic murmur stemming from turbulent flow across the atrioventricular valves. Mitral valve prolapse. not mitral stenosis, is frequently seen in Marfan syndrome and causes a mid- to late-systolic murmur_

(Choice 0) Pericardial friction rub is seen with pericarditis_ However, Marfan syndrome is not associated with pericarditis.

(Choice E) Pulsus parvus et tardus is an arterial pulse with decreased amplitude and delayed peak_ It is commonly seen in severe aortic stenosis, which is not usually associated with ivlarfan syndrome_

(Choice F) Third heart sound is a low-frequency diastolic sound that is a normal finding in healthy young adults and an abnormal finding in those with dilated ventricles_ Mitral valve prolapse resulting in regurgitation and heart failure can cause third heart sound_ However, this patient has no symptoms of heart failure_

Educational objective: Aortic dissection is the most dangerous complication of Marfan syndrome_ It typically presents as tearing chest pain radiating to the back and neck and must be identified immediately to decrease the risk of death_ Aortic regurgitation is a complication of aortic dissection and presents with an early diastolic murmur_

References:

1. Therapy insight: aortic aneurysm and dissection in Marfan's syndrome.

Copyright CI Morld Last updated: [9/20/2014]

1.1 Feedback, End Block

Clinical features of aortic dissection

Risk factorsl

associations

— .

- Hypertens[on 4rost common)

- Marfan syndforre

- Cocaine use

Clinical

features

• Severe, sharp, tearing chest or back pain

• >20 mm Hg variation in systolic blood

pressure between arms

Complications

in lved (vo

structure)

I. Stroke (carotid arteries)

Acute aortic regurgitation (aortic valves)

Hornerrs syndrome (superior cervical

sympathetic ganglion)

. Acute myocardial ischemia/infarction

(coronary artery}

. Pericardial effusion/cardiac tamponade

(pericardial cavity)

Hemothorax (pleural cavity)

Lower-extremity weakness or ischemia

(spinal or common iliac arteries)

Abdominal pain {mesenteric artery) v

Okiimil realizes of aortic dissection 1

• 1

117 Notes Calculator.

Item: 7 of 44

2

3

r Mark

Previous

Lab Values.

Next

▪ 4-

▪ 5

• 6

• 10 • 11

• 12

• 13

• 14

• 15

• 16

• 17

• 18

• 19

• 20 • 21

• 22

• 23

• 24

• 25

• 26

• 27

• 28

• 29

• 30 • 1

• 32

• 33

• 36

▪ 38

• 39

• 40

Media Exhibit

• 41 42

127 Notes Next

lirMark Previous Calculator. Lab Values.

Item: 8 of 44

End Block 1.1

Feedback,

44.

A 34-year-old woman is admitted to the hospital because of septic shock secondary to a urinary tract infection. In the intensive care unit. she receives intravenous fluids and antibiotics. Her initial ECG shows sinus tachycardia but is otherwise unremarkable. Chest x-ray shows no abnormalities. An internal jugular vein catheter is placed on the right side using ultrasound guidance to locate the vein_ The patient is properly draped_ and the skin is cleaned with chlorhexidine solution_ Blood is freely aspirated from all ports after insertion. Vital signs are stable. and oxygenation is maintained_ Which of the following is the most appropriate next step in managing this patient?

Explanation:

Central venous catheters are commonly used for access in patients with difficult vascular access and for long-term administration of medications. Complications from central venous catheter placement occur in 1%-5% of the cases and include arterial puncture. pneumothorax, hemothorax, thrombosis, air embolism. sepsis, vascular perforation, and myocardial perforation leading to tamponade_ A chest x-ray should always be obtained to confirm proper placement of the catheter and absence of complications_

To avoid myocardial perforation, the catheter tip should be located proximal to either the cardiac silhouette or the angle between the trachea and the right mainstem bronchus. Ideally, the catheter tip should lie in the superior vena cava. Tip placement in smaller veins (e.g., subclavian, jugular, azygous, and internal mammary) predisposes to vascular perforation_ Proper placement is essential prior to administering medications and other agents due to the possibility of administration into the wrong anatomic space (Choices A and C).

(Choice 6) Echocardiography can detect fluid in the pericardium but is not useful in determining the placement of a catheter tip or showing pneumothorax.

fir.hrliena Fl RpnpAt 17_1pAri Frr-i is not ri iritp r-hpqt .:e_ry in qhnwinn thi nrnnpr nIrpmpnt ref

1

5

7 8

▪ 9

• 10 • 11 • 12 • 13 • 14 • 15 • 16 • • 17

• 19

21 22

• 23 • 24 • 25 • 26

27

I • 29

31 • 32 ■ • 33 ▪ 34-

• 36 • 37

I 39 ▪ 40

I

• 30

A_ Antibiotic instillation into the catheter [9%]

B_ Echocardiography [2%]

C_ Heparin instillation into the catheter [11%]

• D_ Portable chest x-ray [67%]

E. Repeat 12-lead ECG [4%]

• 1

▪ 3

G 5

7

8

9

• 10

▪ 11

▪ 12

• 13

• 14

• 15

• 16

• 17

▪ 1 • 19

▪ 2

- 21

▪ 22

• 23

• 24

• 25

▪ 26

• 27

• 28 I • 29

• 30

▪ 32

• 33

▪ 34- • '1=.

▪ 36

• 37 •

I 39

• 40

• 41

• 42

Item: 8 of 44

rNlark -c21

SI

EVI*

Previous

Next Lab Values, Notes

Calculator

• D_ Portable chest x-ray [67%]

E. Repeat 12-lead ECG [4%]

Explanation:

Central venous catheters are commonly used for access in patients with difficult vascular access and for long-term administration of medications_ Complications from central venous catheter placement occur in 1%-5% of the cases and include arterial puncture. pneumothorax, hemothorax, thrombosis, air embolism, sepsis, vascular perforation. and myocardial perforation leading to tamponade_ A chest x-ray should always be obtained to confirm proper placement of the catheter and absence of complications_

To avoid myocardial perforation. the catheter tip should be located proximal to either the cardiac silhouette or the angle between the trachea and the right mainstem bronchus. Ideally, the catheter tip should lie in the superior vena cava_ Tip placement in smaller veins (e.g.. subclavian, jugular, azygous, and internal mammary) predisposes to vascular perforation_ Proper placement is essential prior to administering medications and other agents due to the possibility of administration into the wrong anatomic space (Choices A and C).

(Choice B) Echocardiography can detect fluid in the pericardium but is not useful in determining the placement of a catheter tip or showing pneumothorax.

(Choice E) Repeat 12-lead ECG is not as accurate as chest x-ray in showing the proper placement of a central venous catheter or monitoring the complications.

Educational objective: A chest x-ray should be obtained in all patients who undergo central venous catheterization to confirm proper placement of the catheter tip and absence of complications before administering drugs or other agents through the catheter_

References:

1. Aberrant locations and complications in initial placement of subclavian vein catheters

2. Central venous catheters. Placement and monitoring tips.

Copyright © Morld

Last updated: [11 /1 0/201 4]

a 1.1

Feedback, End Block

• 6

22 •

2 a

7

■ 5

9

10 11

' 12 ' 13 ▪ 14 1. • 15 ▪ 16 •

• 17

• 19 ▪ ARM

21

• 23 • 24 • 25 ▪ 26

27

I • 29

• 1 ' 32 ■

• 33 ' 34- El

' 36 • 37

• 39 ' 40

I

' 30

Item: 9 of 44

r Mark 127

Previous Next

Lab Values. Notes . Calculator .

44.

A 64-year-old male presents to the ER with chest pain. His history is significant for a hospitalization 2 weeks ago for an acute myocardial infarction. The patient reports that he is afraid his heart attack is recurring. He localizes the pain to the middle of the chest. and says that he can only take shallow breaths because deep breaths make the pain worse_ Leaning forward in his chair makes the pain somewhat better_ He notes some associated neck pain and general malaise, but denies shortness of breath. palpitations, syncope or cough_ His temperature is 367° C (98° F). blood pressure is 135/84 mmHg and heart rate is 9almin_ EKG shows ST segment elevations in all limb and precordial leads except in aVR. where ST depression is seen_ Which of the following is the best next step in the management of this patient'?

A_ Anticoagulation with heparin [31C]

B_ Anticoagulation with warfarin [1%]

C_ Nitroglycerin and beta-blockers [9913]

D_ Broad-spectrum antibiotics [69C]

E Non-steroidal antiinflammatory agent [80%]

Explanation:

The pattern of this patient's pain (worse with deep inspiration, improved on leaning forward) suggests pericarditis rather than myocardial infarction_ Furthermore, the EKG findings described are classic for pericarditis (diffuse ST elevation with the exception of reciprocal depression in aVR), whereas only specific leads are affected in ML Post-MI acute pericarditis typically occurs in the first several days after the infarction_ This patient most likely has Dressler's syndrome. a pericarditis that can occur weeks after an ML Dressler's syndrome is believed to be due to immunologic phenomena Malaise and sometimes fever are also characteristic_ The erythrocyte sedimentation rate is typically elevated_ NSAIDs are the mainstay of therapy for Dressler's syndrome_ Corticosteroids can be used in refractory cases or when NSAIDs are contraindicated_

(Choice A) Anticoagulation with heparin would be appropriate if this patient was having another myocardial infarction. but the characteristics of his pain as well as the EKG findings are more suggestive of pericarditis_

(Choice B) Coumadin is used for long-term anticoagulation, and should probably be avoided in a patient with pericarditis to prevent the development of a hemorrhagic pericardial effusion_

1.1 Feedback, End Block

• 1 • 2 ▪ a • 4 ▪ 5 ▪ 6

7

9

10 • 11 • 12 • 13 • 14 • 15 • 16 • 17 ▪ 1E1. • 19

21 .1r.

22 • 23 • 24 • 25 ▪ 26 1 • 27 • 2B • 29 • 30

• 33 • 34-

32

• 36 ;3111.17

• 39 • 40 • 41 • 42

Al

Item: 9 of 44

rNlark -c21

1>- CP,*

Previous Next

Lab Values. Notes Calculator

L

11 Anticoagulation with warfarin [1%]

C_ Nitroglycerin and beta-blockers [9%)]

D_ Broad-spectrum antibiotics [13c.1.70]

• E Non-steroidal antiinflammatory agent [80%]

Explanation:

The pattern of this patient's pain (worse with deep inspiration, improved on leaning forward) suggests pericarditis rather than myocardial infarction_ Furthermore, the EKG findings described are classic for pericarditis (diffuse ST elevation with the exception of reciprocal depression in aVR), whereas only specific leads are affected in MI_ Post-MI acute pericarditis typically occurs in the first several days after the infarction_ This patient most likely has Dressler's syndrome, a pericarditis that can occur weeks after an Ml. Dressler's syndrome is believed to be due to immunologic phenomena Malaise and sometimes fever are also characteristic_ The erythrocyte sedimentation rate is typically elevated. NSAIDs are the mainstay of therapy for Dressler's syndrome_ Corticosteroids can be used in refractory cases or when NSAIDs are contraindicated_

(Choice Al Anticoagulation with heparin would be appropriate if this patient was having another myocardial infarction, but the characteristics of his pain as well as the EKG findings are more suggestive of pericarditis.

(Choice El) Coumadin is used for long-term anticoagulation. and should probably be avoided in a patient with pericarditis to prevent the development of a hemorrhagic pericardial effusion.

(Choice C) Nitroglycerine and beta-blockers are treatments for myocardial infarction but are not helpful in the treatment of pericarditis_

(Choice 1:1) Broad-spectrum antibiotics are not indicated for Dressler's syndrome since it is caused by an immunologic abnormality and not infection_

Educational objective: Patients with Dressler's syndrome present weeks after a myocardial infarction with chest pain that is improved by leaning forward. NSAIDs are the treatment of choice. Anticoagulation should be avoided to prevent development of a hemorrhagic pericardial effusion.

Copyright (D U'Norld

Last updated: [8/22/2014]

1.1 Feedback, End Block

1

9 • 10 • • • • •

11 12 13 1

17 • 16 • 17 • 1E1. • 19

212

• 2 • 23 • 24 • 25 • 26 1 • 27 • 28 • 29 • 30 • 1

33 • 34-

• 32 •

36 • 37

• 39 • 40

• 42

(Choice B) Adrenal insufficiency should also be suspected in any patient who suddenly becomes • r • . • • .1

1.1 Feedback, End Block

Item: 10 of 44 Mark CP,*

Previous Next

Lab Values Notes Calculator

A 25-year-old woman is undergoing an elective cholecystectomy after 2 episodes of acute calculous cholecystitis_ The procedure is converted from laparoscopy to open due to difficulty with visualization. The patient suddenly becomes hypotensive and develops a diffuse generalized rash_ Her past medical history is significant for obesity and polycystic ovarian syndrome_ Two months ago, she developed hives after having protected sex with her partner for the first time_ Which of the following is the most likely cause of this patient's hypotension?

A Acute blood loss [1%]

B. Adrenal insufficiency [39C]

• C_ Anaphylactic shock [93%]

D_ Cardiac tamponade [0%]

E. Disseminated intravascular coagulation [2°A]

H Myocardial infarction [0%]

G_ Pulmonary embolism [0%]

H_ Septic shock [1%]

I_ Tension pneumothorax [0%]

Explanation:

The clinical scenario described is suggestive of anaphylactic shock caused by latex The widespread use of latex in medical products has made it a common cause of allergy_ The development of hives after sexual intercourse in this patient was likely due to sensitization to latex condoms_ Latex gloves used by surgeons provoked the anaphylactic reaction. Pre-existing atopic dermatitis increases the risk of latex allergy. Health care workers and surgical patients, especially those undergoing abdominal or genitourinary surgery, are at higher risk as well_ Latex allergy rates peaked in the 1990s and have since decreased with the use of powder-free latex gloves and latex alternatives. but the risk remains elevated in these populations_

(Choices A, 0, F, G, and I) Hemorrhage, cardiac tamponade, myocardial infarction, pulmonary embolism, and tension pneumothorax can cause hypotension but do not cause a rash.

r Mark Item: 10 of 44 Lab Values. Notes . Calculator. Previous Next

Copyright © Morld Last updated: [10/13/2014] a

End Block 1.1

Feedback,

11,1.01111,Na 11.1.11,1, L

Explanation:

The clinical scenario described is suggestive of anaphylactic shock caused by latex. The widespread use of latex in medical products has made it a common cause of allergy. The development of hives after sexual intercourse in this patient was likely due to sensitization to latex condoms_ Latex gloves used by surgeons provoked the anaphylactic reaction_ Pre-existing atopic dermatitis increases the risk of latex allergy_ Health care workers and surgical patients, especially those undergoing abdominal or genitourinary surgery, are at higher risk as well_ Latex allergy rates peaked in the 1990s and have since decreased with the use of powder-free latex gloves and latex alternatives. but the risk remains elevated in these populations.

(Choices A, 13, F, G, and I) Hemorrhage. cardiac tamponade. myocardial infarction, pulmonary embolism. and tension pneumothorax can cause hypotension but do not cause a rash_

(Choice B) Adrenal insufficiency should also be suspected in any patient who suddenly becomes hypotensive under a stressful condition such as surgery. Patients with primary adrenal insufficiency (Addison's disease) can have gradual development of hyperpigmentation due to increased production of melanocyte-stimulating hormone_ However, a sudden onset of rash is not seen with adrenal insufficiency_

(Choice E) Disseminated intravascular coagulation may cause hypotension due to excessive bleeding from the operative site and venous punctures. Petechiae and ecchymoses. not hives. are commonly present.

(Choice H) Septic shock is characterized by fever, and an established focus of infection is usually present

Educational objective: Anaphylaxis can result from exposure to latex-containing products such as surgical gloves and condoms_ Health care workers and patients with atopic disease are at higher risk of latex allergy_

References:

1. Outcomes of a natural rubber latex control program in an Ontario teaching hospital.

2. Natural rubber latex allergy among health care workers: a systematic review of the evidence.

1

3

G 5

7

9

• 10 • 11

• 12 • 13 •

• 15 • 16 • 17

1 • 19 • 2 - 21 • 22

• 23 • 24 • 25 • 26 • 27 • 28 I

• 29 • 30 • 31

• 32 • 33 • 34- • '1=.

• 36 • 37 •

• 39 • 40 • 41 • 42

2

3

I• 5 6 7 8

▪ 9

• 10 11

I. 12 • 13

• 15 • 16 • • 17

• 19 ▪ ARM

21 22

• 23 • 24 • 25 • 26

27

I • 29

' 32 ■

• 33 ' 34- .1

' 36 • 37

• 39 ' 40

I

' 30

Item: 11 of 44 Mark

Previous Next

Lab Values. Notes Calculator.

44.

A 62-year-old man with a past medical history of bladder cancer and recurrent hematuria comes to the emergency department after "passing out." The patient was standing up to urinate shortly after waking when he felt dizzy. He was able to return to his bed before losing consciousness for several minutes. The patient's electrocardiogram (ECG) at the time of admission is shown in the Exhibit_ Which of the following most likely accounts for the observed ECG changes?

A Atrial fibrillation [2%]

11 Atrial premature beats [5136]

C_ Impaired atrioventricular node conduction [82%]

D_ Impaired sinoatrial node automaticity [7°•o]

E Ventricular preexcitation [490]

Explanation:

This patient's electrocardiogram (ECG) shows progressive prolongation of PR interval leading to a non-conducted P wave and a 'dropped' QRS complex consistent with Mobitz type I second degree atrioventricular (AV) block. Pdobitz type I AV block (often referred to as VVenckebach block) is an intermittent AV block due to impaired conduction within the AV node. Characteristic features of Mobitz type I AV block include:

• Constant P-P interval • Increasing PR interval • Decreasing R-R interval • Group beating (repeating clusters of beats followed by a dropped QRS)

The PR interval prolongation is best appreciated by measuring PR intervals just before and after the 'dropped' QRS complexes. Mobitz type I AV block is often asymptomatic and has a benign prognosis in young patients with good ventricular function. However, it may occasionally cause significant bradycardia with symptoms of inadequate cardiac output (eg, fatigue, lightheadedness, angina, syncope, heart failure), especially in older patients with acute ischemia or additional cardiac comorbidity.

(Choice Al In atrial fibrillation, there is no organized atrial activity, and impulses are conducted in a random

1.1 Feedback, End Block

Item: 11 of 44 r Mark 117

Previous Next

Lab Values. Notes Calculator.

The PR interval prolongation is best appreciated by measuring PR intervals just before and after the 'dropped' QRS complexes_ Mobitz type I AV block is often asymptomatic and has a benign prognosis in young patients with good ventricular function_ However, it may occasionally cause significant bradycardia with symptoms of inadequate cardiac output (eg, fatigue, lightheadedness. angina, syncope, heart failure), especially in older patients with acute ischemia or additional cardiac comorbidity.

(Choice A) In atrial fibrillation. there is no organized atrial activity, and impulses are conducted in a random and unpredictable manner_ This leads to an absence of any discernible P waves and irregularly irregular rhythm with varying R-R interval_

(Choice B) Atrial premature beats indicate depolarization of the atria originating in a focus outside the sinoatrial (SA) node_ They are seen on ECG as a P wave with abnormal morphology, often occurring earlier in the cardiac cycle than a normal P wave_

(Choice ID) Impaired SA node automaticity leads to sick sinus syndrome, which is often due to degeneration and/or fibrosis of the SA node and surrounding atrial myocardium. Patients may present with fatigue, lightheadedness. palpitations. presyncope, or syncope_ ECG often shows bradycardia, sinus pauses/arrest, SA exit block. or alternating bradycardia and atrial tachyarrhythmias (tachycardia-bradycardia syndrome).

(Choice E) Ventricular preexcitation (Wolff-Parkinson-White syndrome) is due to an accessory conduction pathway that directly connects the atria and ventricle. bypassing the AV node. The ECG shows a characteristic pattern of short PR interval (<112 seconds). widened ORS complex. and "slurred' upstroke of QRS complex ("delta wave")_

Educational objective: In Mobitz type I atrioventricular block. there is progressive prolongation of PR interval leading to a non-conducted P wave and a 'dropped' ORS complex_ The PR interval prolongation is best appreciated by measuring PR intervals just before and after the 'dropped' QRS complexes_

References:

1. Long-term follow-up of older patients iuvith Mobitz type I second degree atrioventricular block.

2. Is Mobitz type I atrioventricular block benign in adults?

Copyright 0 UWorld

Last updated: [1G/16/K1.El]

a 1.1

Feedback, End Block

Next Mark

Previous

Item: 11 of 44 • 1270 Lab Values Notes Calculator .

Media Exhibit 1

Mobil7Type 1 Second degree AV block

Mobitz Type I Second degree AV block

Progressive PR interval lengthening leading to a drop of a ORS complex after a non-conducting P wave

Area of dropped QRS complex PR interval i5 ionger before than after QRS complex drop uworld.corn

• 1

2 3

4

5

6

7

8

9

10

11

12

• -13

• 14

• 15

• 16

• 17

• 18

• 19

• 20

• 21

• 22

• 23

• 24

• 25

• 26

• 27

▪ 28

▪ 29

▪ 30

• 32

• 33

▪ 34

• 36

▪ 38

▪ 39

▪ 40

• 41

42

4 Exhibit

r Mar k Previous Lab Values. Next I

Item: 11 of 44 Notes Calculator .

End Block 1.1

Feedback,

▪ 5

▪ 6

▪ 7

▪ 8

▪ 9

11

• 12

• 13

14

15

• 16

• 17

18

19

• 20

• 21

• 22

23

24

• 25

• 26

• 27

• 28

• 29

30

1

• 32

• 33

• 34

• 36

• 38

• .39

• 40

Atrial fibrillation

Atrial fibrillation

• No distinct p waves in AF

• Irregularly, irregular R-R intervals

▪ Narrow GAS complexes typically seen

▪ Atrial rate is usually 350 beats per minute

T waves can be seen

. .... -• • l• •-• .1-•••- •

......

LLLL

rlffinliFfrrrit . . •i •i •i • • i •i i• •

. : .

. i .

! No r row QRS complexes

.41111b-

vs

I Irregularly irregular rhythm

t ISMLEftrld. LL(

I

• 41

• 42

• 1

2

▪ 3

Premature atrial contractions (PACs)

I FAG

1 41 . I -1 ...I • EA

II • I •

\--A-1 4 —f r '-1 1 -

1

PI i

Rhythm 1. wave] I QRS

60-100 Irregular

P wave of the PAC often has a different shape from the one originating in the SA node

NOirlflai

uworld.opm

1.1 Feedback, End Block

Item: 11 of 44

r Mark Previous Next Lab Values. Notes Calculator .

Media Exhibit 44.

2 ▪ 3

4

▪ 5

▪ 6

▪ 7

▪ 8

▪ 9

• 10

11

• 12

• -13

• 14-

• 15

• 16

• 17

• la

• 19

• 20

• 21

• 22 • 23

• 24

• 25

• 26

• 27

• 28

▪ 29

▪ 30

▪ 1

• 32

• 33

• 36

• 38

• 39

• 40

• 41

42

Item: 11 of 44

4

▪ 6

▪ 8

• 10

• 14

• 16

• 18

• 20

• 22

• 24

• 26

• 28

• 30

• 32

• 34

• 36

• 38

▪ 3

r Mark

Media Exhibit

Wolff-Parkinson-White Syndrome

Preexcitation of the ventricles via an abnormal bypass tract • Short PR interval • Presence of initial slow upstroke of the ORS complex (delta wave) - ST and T wave abnormalities

• 12

1.1 Feedback, End Block

• 41

▪ 5

• 15

• 19

• 23

• 27

• 33

127 Notes . Calculator.

Item: 12 of 44 Lab Values.

Mark Previous Next

1

3

5

7

8 9

10 I 11

* 12 13

I • 14 ' 15 • 16 •

• 17

• 19 • ARM

21 —1 22

' 23 ' 24 ' 25 • 26

27 28 I

• 29

• 30 • 1 • 32 ■

• 33 • 34-

44.

A 47-year-old female smoker with type 2 diabetes mellitus, mild normocytic anemia, and stage 3 chronic renal disease is being evaluated for exertional dyspnea. Her family history is insignificant. Blood pressure is 142/88 mm Hg and pulse is 76/min. Her body mass index is 32 kg/m2. Single photon emission CT images are obtained at rest and after 5 minutes of treadmill exercise by using separate intravenous injections of technetium-99-labeled perfusion agent, as shown in the images below.

This patient would benefit most from which of the following medications?

A. Albuterol [13%]

Antiplatelet agent [66%]

C_ Appetite suppressant [3%]

Erythropoietin [14%]

E. Metformin [4%]

Explanation: • 36 • 37

• 39 • 40

41 42

SUPINE

APEX

SEP 1111

BASE

STRESSJRNO(G)

SUP.n SEP AT

BASE

REST JRNC

BASE STRESS !RUM)

1

1 6 3

I• 5 6

7 8

9 10 I

I 12

13

• 15

• 16 •

• 17

• 19

21

22

• 23

• 24

• 25

• 26

27

I • 29

' 30

' 32 ■

• 33

• 34- El

' 36

• 37

• 39

' 40

41

Item: 12 of 44 r Mark •i

Previous Next Lab Values Notes . Calculator.

44.

Technetium-99-labeled (Tc-99m) perfusion agents (sestamibi or tetrofosmin) have a half-life of 6 hours. passively diffuse into perfused myocardial cells, have minimal redistribution afterward, and can provide an assessment of both myocardial function and perfusion. Tc-99m is injected at rest and images are obtained during the gated single photon emission CT (SPECT) scan (bottom panel above)_ The patient then undergoes stress testing (physiologic or pharmacologic) and has repeat gated SPECT images (top panel above)_ Patients with normal tracer uptake at both rest and exercise have an excellent prognosis with <1% annual risk of coronary artery disease (DADS_

A decreased tracer uptake both at rest and with exercise (fixed defect) indicates likely scar tissue with decreased perfusion and CAD. A decreased tracer uptake with stress but normal uptake at rest (reversible defect) indicates inducible ischemia and likely CAD, as seen in this patient's apical images_ Current guidelines recommend antiplatelet therapy (eg, aspirin) for prevention of myocardial infarction. beta blockers. and modification of the patient's risk factors (eg, smoking cessation, lipids, diabetes).

(Choice A) Albuterol is a beta agonist that may cause tachycardia, elevated blood pressure, and possible arrhythmias in a patient with suspected CAD. Current guidelines recommend beta blockers to reduce the potential for future coronary events_

(Choice C) Appetite suppressants are sympathomimetics that can cause significant elevations in heart rate and blood pressure and are contraindicated in patients with CAD.

1.1 Feedback, End Block

127 Notes Calculator.

Item: 12 of 44 Lab Values.

r Mark Previous Next

Copyright 0 UWorld Last updated: [1211/2014]

End Block 1.1

Feedback,

I !L.! IL LAI I...ALALI I I I ip....11-L41 LAIL41 I L41 15....LILAI I L411 1....1 Li 1 1..4.1 1L..1 1 I_ I Lr -V.....1 I I II L4L I L aL L411 1....1 II I IW LL.11

during the gated single photon emission CT (SPECT) scan (bottom panel above)_ The patient then undergoes stress testing (physiologic or pharmacologic) and has repeat gated SPECT images (top panel above). Patients with normal tracer uptake at both rest and exercise have an excellent prognosis with <1% annual risk of coronary artery disease (CAD)_

A decreased tracer uptake both at rest and with exercise (fixed defect) indicates likely scar tissue with decreased perfusion and CAD. A decreased tracer uptake with stress but normal uptake at rest (reversible defect) indicates inducible ischemia and likely CAD, as seen in this patient's apical images_ Current guidelines recommend antiplatelet therapy (eg, aspirin) for prevention of myocardial infarction. beta blockers. and modification of the patient's risk factors (eg, smoking cessation, lipids, diabetes).

(Choice A) Albuterol is a beta agonist that may cause tachycardia, elevated blood pressure, and possible arrhythmias in a patient with suspected CAD_ Current guidelines recommend beta blockers to reduce the potential for future coronary events.

(Choice C) Appetite suppressants are sympathomimetics that can cause significant elevations in heart rate and blood pressure and are contraindicated in patients with CAD_

(Choice 0) Erythropoietin stimulates ervthropoiesis for this patient's anemia but can cause serious cardiovascular events. thromboembolic events. stroke. and increased mortality in patients with cardiovascular disease

(Choice E) Metformin may cause lactic acidosis when given to patients with conditions predisposing to hypo is (eg. cardiovascular disease, chronic kidney disease) and should be avoided in this patient

Educational objective: Single photon emission CT scan is a useful tool to evaluate for coronary artery disease and indicates inducible ischemia when a reversible defect is noted on stress and rest images_ Antiplatelet therapy is the preferred treatment to prevent coronary artery disease in these patients_

References:

1. SPECT imaging for detecting coronary artery disease and determining prognosis by noninvasive assessment of myocardial perfusion and myocardial viability.

• 1

▪ 3

▪ 5

6 ■

▪ 7

▪ 9

10 I • 11

I 12

• 13

14

• 15

• 16

• 17

• 19

• 2

- 21

• 22

• 23

• 24

• 25

• 26

• 27

• 28 I

• 29

• 30

• 31

• 32

• 33

• 34- • '1=.

• 36

• 37 •

• 39

• 40

• 41

42

1

3

5

• 7

• 9 • 10 • 11 • 12

13

• 1 .E. .11

17

• 19

21

• 15 16 •

22 • 23 • 24 • 25 • 26

27 I

• 29

• 1 • 32 ■

• 33 • 34-

• 36 • 37

• 39 • 40

I

• 30

Item: 13 of 44 rNlark

Previous Next Lab Values. Notes Calculator.

44.

A 65-year-old man comes to the physician for fatigue, poor appetite, and lower-leg swelling. Past medical history is significant for hypertension. type 2 diabetes mellitus. chronic cough, chest infections, and lower-extremity peripheral artery disease. The patient has a 40-pack-year smoking history. He does not use alcohol or illicit drugs. Physical examination shows a barrel-shaped chest, with bilateral scattered wheezing on auscultation. The patient's abdomen is distended, and the liver edge is palpated 4 cm below the right costal margin. There is 2+ pitting edema and dilated, tortuous superficial veins over both lower extremities. Manual pressure on the abdomen causes persistent distension of the jugular veins. His serum sodium level is 135 mEq/L, and creatinine is 12 mgict. An abnormality of which of the following is most likely the cause of this patient's edema?

A. Portal venous resistance [19%]

B. Pulmonary artery systolic pressure [45%]

C. Pulmonary capillary wedge pressure [21%]

a Serum albumin level [1390]

E Urinary protein excretion [1%]

Explanation:

Characteristic findings of cod' pulmonale

• COPD (most common)

Common • Interstitial lung disease

etiologies • Pulmonary vascular disease (eg, thromboernbolic)

• Obstructive sleep apnea

• Dyspnea on exertion, fatigue, lethargy

Symptoms • Exertional syncope (due to j cardiac output)

• Exertional angina (due to T myocardial demand)

Lai

1.1 Feedback,

End Block

• 1270 Lab Values Notes . Calculator.

Item: 13 of 44 r Mark Previous Next

di This patient's clinical findings—long-term tobacco use, chronic cough, barrel-shaped chest, elevated jugular

End Block 1.1 Feedback,

1

3

5

7

9

10 11

I. 12 • 13

15

16 ■

17

• 19

21

24

• 27

• 28 I

• 29

• 30

• 32 j • 33

• 34 El

• 36

• 38

• 39

• 40

• 41 42

44.

Characteristic findings of cor pulmonale . ,

Common etiologies

• COPD (most common) • Interstitial lung disease • Pulmonary vascular disease (eg, thromboenibo4c) • Obstructive sleep apnea

Sy mptom s

• Dyspnea on exertion, fatigue, lethargy * Exertional syncope (due to j cardiac output) • Exertional angina (due to T myocardial demand)

Examination

• Peripheral edema

• I Jugular venous pressure with prominent a wave • Loud S2 • Right-sided heave • Pulsate liver from congestion • Tricuspid regurgitation murmur

Imaging

• Electrocardiogram (ECG): Partial or complete right bundle branch block, right axis deviation, right ventricular hypertrophy, right atrial enlargement

• Echocardiogram: Pulmonary hypertension, dilated right ventricle, tricuspid regurgitation

9 Right heart catheterization: Gold standard for diagnosis showing right ventricular dysfunction, pulmonary hypertension & no left heart disease

0USMLEWOrld, LLC

44.

1

1 6 3

5

• 7

• 9

• 10 i 11

i 12 • 13

• 1 .E. .11

17

• 19

21

• 15

16 •

22

• 23 • 24 • 25 • 26

27

I

• 29

• 1 i 32 ■

• 33 • 34

• 36 • 37

• 39 • 40

• 30

Item: 13 of 44 Mark 117

Previous

Next

Lab Values

Notes . Calculator .

USIVILEWorldr LLC

This patient's clinical findings—long-term tobacco use, chronic cough, barrel-shaped chest, elevated jugular venous pressure. and enlarged liver—are consistent with chronic obstructive pulmonary disease (C013D) with cor pulmonale_ Cor pulmonale refers to impaired function of the right ventricle caused by pulmonary hypertension that occurs due to underlying diseases of the lungs (COPD, interstitial lung disease), pulmonary vasculature (idiopathic pulmonary arterial hypertension), or obstructive sleep apnea_ By convention, right ventricular dysfunction due to left heart disease or congenital heart disease is not considered cor pulmonale_

Patients with cor pulmonale typically present with exertional symptoms (eg, dyspnea, angina, syncope), anorexia or abdominal pain (due to hepatic congestion). and peripheral edema_ Physical examination may show a loud 1j2 (pulmonic component of 2nd heart sound) and a right ventricular 3rd heart sound, tricuspid regurgitation murmur (holosystolic at the left lower sternal border. augments with inspiration), elevated jugular venous pressure with hepatojugular reflux, hepatomegaly with pulsatile liver, and occasionally ascites or pleural effusion. The diagnosis of car pulmonale is based primarily on clinical features and echocardiographic findings (eg, right ventricular hypertrophy. tricuspid regurgitation with right atrial enlargement). If necessary, definitive diagnosis can be made using right heart catheterization showing elevated pulmonary artery systolic pressure (>25 mm Hg).

(Choice A) Increased portal venous resistance (portal hypertension) is seen in the setting of hepatic cirrhosis or extrahepatic portal vein thrombosis. Chronic liver disease is associated with stigmata such as palmar erythema, spider angiomata.. caput medusae, and gynecomastia_ Complications of cirrhosis with portal hypertension include ascites, spontaneous bacterial peritonitis, and variceal bleeding_

(Choice C) The pulmonary capillary wedge pressure (P WP) is an estimation of left ventricular end-diastolic pressure. and is elevated in patients with left ventricular systolic and/or diastolic dysfunction. Patients with elevated PCWF typically have signs of pulmonary edema on lung auscultation_

(Choices D and E) Hypoalbuminemia may occur due to decreased hepatic synthesis (cirrhosis) or increased urinary or gastrointestinal protein loss (ie, nephrotic syndrome. protein-losing enteropathy). Nephrotic syndrome typically presents with periorbital and pedal edema that may progress to generalized edema (anasarca), ascites, and pleural effusions. Patients can also develop severe intravascular hypovolemia due to over-diuresis and fluid movement into the interstitial spaces_ This patient does not have anasarca or facial edema; he also does not have stigmata of chronic liver disease or diarrhea (to suggest enteropathy).

1.1 Feedback, End Block

5 G

117 Notes Calculator.

Item: 13 of 44 Lab Values.

r Mark Previous Next

Copyright © Morld Last updated: [9/11 /2014] I 39

1.1 Feedback, End Block

(Choice Al Increased portal venous resistance (portal hypertension) is seen in the setting of hepatic cirrhosis or extrahepatic portal vein thrombosis. Chronic liver disease is associated with stigmata such as palmar erythema, spider angiomata, caput medusae. and gynecomastia_ Complications of cirrhosis with portal hypertension include ascites, spontaneous bacterial peritonitis, and variceal bleeding_

(Choice C) The pulmonary capillary wedge pressure (PCWP) is an estimation of left ventricular end-diastolic pressure, and is elevated in patients with left ventricular systolic and/or diastolic dysfunction. Patients with elevated PCWP typically have signs of pulmonary edema on lung auscultation.

(Choices 0 and E) Hypoalbuminemia may occur due to decreased hepatic synthesis (cirrhosis) or increased urinary or gastrointestinal protein loss (ie, nephrotic syndrome, protein-losing enteropathy)_ Nephrotic syndrome typically presents with periorbital and pedal edema that may progress to generalized edema (anasarca). ascites, and pleural effusions. Patients can also develop severe intravascular hypovolemia due to over-diuresis and fluid movement into the interstitial spaces_ This patient does not have anasarca or facial edema; he also does not have stigmata of chronic liver disease or diarrhea (to suggest enteropathy).

Educational objective: Cor pulmonale is impaired function of the right ventricle due to pulmonary hypertension that usually occurs due to chronic lung disease_ Signs of right ventricular failure include:

• Elevated jugular venous pressure • Right ventricular 3rd heart sound • Tricuspid regurgitation murmur • Hepatomegaly with pulsatile liver • Lower-extremity edema, ascites, and/or pleural effusions

Echocardiogram will show signs of increased right heart pressures, and right heart catheterization will show pulmonary systolic pressure >25 mm Hg_

• 1

▪ 3

7

• 37

• 9

• 11

13

• 15

• 17

• 19

• 21

• 23

• 25

• 27

• 29

• 33

I 17

• 41

References:

1. Pulmonary hypertension in chronic lung diseases.

1

5

▪ 7

▪ 9

• 10 i 11 i 12 • 13

• 15

17

• 19

21 22

• 23 • 24 • 25 ▪ 26

27

I

• 29

• 1 i 32 ■

• 33 • 34- El

• 36 • 37

• 39 • 40

I

• 30

Item: 14 of 44

r Mark 117

Previous Next

Lab Values. Notes . Calculator.

44.

Neighbors find a 65-year-old male unconscious in his garden and immediately call EMS. The man is intubated at the scene. In the ER, he develops sustained ventricular tachycardia and requires cardioversion. He is started on an antiarrhythmic agent and. once recovered, is discharged to home. Three months later he returns to his physician complaining of dispnea on exertion_ Chest- x ray reveals bilateral lung fibrosis. All cultures are negative and lung biopsy reveals lipoid pneumonitis. Which of the following medications is most likely responsible for his current condition?

A_ Procainamide [3%]

Amiodarone [85%]

C_ Lidocaine [1%]

D. Quinidine [2%]

E. Digo in [2%]

F. Metoprolol [1%]

G. Disopyramide [1%]

H. Flecainide [2%]

Ibutilide [1%]

J. Sotalol [1%]

Explanation:

The above patient had an episode of ventricular tachycardia and, after receiving treatment, developed lipoid pneumonitis. Amiodarone is an anti-arrhythmic known to cause lung damage. Recall the 5 common side effects of amiodarone:

• Pulmonary toxicity: Amiodarone lung toxicity is associated with total cumulative dose rather than serum drug levels_ The longer the patient uses amiodarone. the greater the chance of pulmonary fibrosis developing_ It is an excellent drug for both atrial and ventricular arrhythmias. but its multiple side effects limit long-term use_

• Thyroid dysfunction: Hypothyroidism is more commonly seen (85%), but hyperthyroidism can occur as well (15%). Since thyroid dysfunction is so common with amiodarone, patients should have TSH levels checked before starting treatment, and also at 3-4 month intervals thereafter_

1.1 Feedback, End Block

41

1

1 6 3

5

▪ 7

▪ 9

• 10

• 11

• 12

• 13

• 15

17

• 19

21

22

• 23

• 24

• 25

▪ 26

27

I

• 29

• 32 111 • 33

• 34- El

• 36

• 37

• 39

• 40

• 30

Item: 14 of 44 r Mark

Previous Next

Lab Values. Notes Calculator.

The above patient had an episode of ventricular tachycardia and, after receiving treatment_. developed lipoid 44.

pneumonitis. Amiodarone is an anti-arrhythmic known to cause lung damage_ Recall the 5 common side effects of amiodarone:

• Pulmonary toxicity: Amiodarone lung toxicity is associated with total cumulative dose rather than serum drug levels_ The longer the patient uses amiodarone, the greater the chance of pulmonary fibrosis developing. It is an excellent drug for both atrial and ventricular arrhythmias, but its multiple side effects limit long-term use_

• Thyroid dysfunction: Hypothyroidism is more commonly seen (85%), but hyperthyroidism can occur as well (15%). Since thyroid dysfunction is so common with amiodarone, patients should have TSH levels checked before starting treatment, and also at 3-4 month intervals thereafter_

• Hepatotoxicity: Up to 25% experience transient elevations of aminotransferase levels_ This is usually asymptomatic, though the drug should be stopped if elevations greater than 2-fold are seen_

• Corneal deposits: Corneal deposits are a common side effect, as amiodarone is secreted by the lacrimal gland_ Vision is usually not affected_ Development of corneal deposits does not necessitate drug discontinuation_

• Skin changes: Some patients develop a blue-gray skin discoloration, typically most prominent on the face.

(Choice Al Procainamide is used to treat atrial and ventricular arrhythmias_ Potential side effects include nausea, drug-induced lupus, agranulocytosis, and QT prolongation_

(Choice C) Lidocaine is a class IP anti-arrhythmic agent used to treat ventricular arrhythmias. High doses can cause confusion. seizures and respiratory depression, but it does not cause lung fibrosis_

(Choice 0) Quinidine is a class IA anti-arrhythmic agent used to treat atrial arrhythmias_ It does have a few well-known side effects including diarrhea. tinnitus, OT prolongation, torsades de pointes, hemolytic anemia and thrombocytopenia, but it does not cause lung fibrosis_

(Choice E) Digoxin is an inotrope used in the treatment of atrial arrhythmias_ It can cause nausea, anorexia, AV block_ and ventricular and supraventricular arrhythmias. It does not affect the lung_

Educational objective: Amiodarone is a class III antiarrhythmic agent. well known for causing pulmonary fibrosis_ Thyroid dysfunction (hypo and hyper). hepatotoxicity, corneal deposits and skin discoloration are other potential side effects_

Copyright (D UWorld

Last updated: [8/22/2014]

a 1.1

Feedback, End Block

Item: 15 of 44 r Mark Calculator Lab Values. Previous Next Notes

I EIL 1 IPS Ob. I% M...1%. ; ". 011 ; aria 1 i.01. arli. ;

1

I Persistent tachyarrhythmia causing:1

Feedback, End Block

44.

A 21-year-old man comes to the emergency department with dizziness and palpitations that started abruptly an hour ago. He has had similar episodes provoked by fatigue or strong emotions. The patient can usually stop the episodes by squatting and taking a deep breath. However, this time these actions did not work. Blood pressure is €51.40 mm Hg and pulse is 24Gimin_ He is diaphoretic and his extremities are cold. Electrocardiogram (EGG) rhythm strip shows a regular, narrow-complex tachycardia. Which of the following is the best next step in management of this patient?

A. Adenosine [40%]

Amiodarone [6°A]

C_ Intravenous fluid resuscitation [19%]

Procainamide

• E Sedation and direct current cardioversion [27%]

F. Yerapamil [3%]

Explanation:

Adult tachycardia algorithm (with pulse) 2010 ACLS guidelines

1

3

5

7

• 9

• 10 • 11

• 12

-13

14

I 15

17

19

21

22

• 23

• 24

• 25

▪ 26

27

28 I

• 29

• 30 • 1

• 32 1. • 33

▪ 34 iV

• 36

• 37

• 39

• 40

41 42

Assess appropriateness for clinical conditon Heart rate typically >150/min if tachyarrhythmia

Identify & treat underlying cause ▪ Maintain patient airway; assist breathing if needed ▪ Oxygen ▪ Cardiac monitor to identify rhythm, monitor blood pressure & oximetry

Mark Item: 15 of 44 Lab Values Calculator. Previous Notes Next

Feedback, End Block

1

3

5 • 6 • 7 • 8

• 9 • 1Q i 11 i 12

-13 14

• 15

17

19

21 22

• 23 • 24 • 25 • 26 • 27 • 28 I • 29 • 30 • 1 • 32 ■

• 33 • .34

• 36 • 7

• 38 • 39 • 40 • 41

42

44.

Adult tachycardia algorithm (with pulse) -2010 ACLS guidelines

Assess appropriateness for clinical condition Heart rate typically >150Imin if tachyarrhythmia

Identify & treat underlying cause • Maintain patient airway assist breathing if needed • Oxygen • Cardiac monitor to identify rhythm, monitor blood pressure & oximetry

Synchronized cardioversion •Sedation • If regular narrow complex,

consider adenosine

I Na

No 1

• IV access & 12-lead ECG • Vagal maneuvers • Adenosine (if regular rhythm) • Beta-blocker or calcium channel

blocker • Consider expert consultation

• IV access & '12-lead ECG

• Consider adenosine if regular & monomorphic

• Consider antiarrhythmic infusion • Consider expert consultation

Persistent tachyarrhythmia causing: • Hypotension • Altered mental status • Signs of shock • Ischemic chest discomfort • Acute heart failure

Yes

ORS duration >0.12 second

Mark Next

-c21

Previous Lab Values. Item: 15 of 44

Notes Calculator.

Feedback, End Block

This patient has a regular, narrow-complex tachycardia (supraventricular tachycardia [SVT) causing hemodynamic instability with signs of poor perfusion (cool extremities). Based on 2010 ACLS guidelines. all patients with persistent tachyarrhythmia (narrow- or wide-complex) causing hemodynamic instability should be managed with immediate synchronized DC cardioversion_ Whenever possible, patients should be provided with adequate sedation and analgesia prior to cardioversion_

Common causes of regular. narrow-complex tachycardia include inappropriate sinus tachycardia, atrial tachycardia. atrial flutter. sinoatrial nodal reentrant tachycardia, atrioventricular nodal reentrant tachycardia (AVNRT). and atrioventricular reentrant (or reciprocating) tachycardia_ An initial 12-lead electrocardiogram (ECG) can identify the correct arrhythmia in approximately 80% of patients_ However, in those with hemodynamic instability, immediate cardioversion should be performed prior to detailed evaluation due to the risk of rapid clinical deterioration_

(Choice A) Patients who have minimal symptoms or remain hemodynamically stable during an episode of regular, narrow-complex SVT can be managed initially with a trial of vagal maneuvers (eg. carotid sinus massage) or intravenous adenosine or calcium channel blockers. Adenosine transiently decreases conduction through the AV node and is effective in terminating most episodes of AVNRT_

(Choices B and 0) Intravenous antiarrhythmics (eg, amiodarone, procainamide) can be used in patients with stable recurrent or refractory wide-complex tachycardia. In patients with unstable narrow-complex tachycardia, synchronized cardioversion is the initial therapy of choice. These drugs can also worsen hypotension and are not recommended acutely in patients with hypotension or hemodynamic instability.

(Choice C) Intravenous fluid resuscitation can improve blood pressure transiently. However, it will not treat the underlying cause of hypotension and may result in unnecessary delay for definitive management of this patient.

(Choice F) Verapamil is a calcium channel blocker that slows conduction through the AV node_ It can be used as a second-line pharmacologic agent to terminate SVT in stable patients if initial vagal maneuvers and adenosine fail to terminate the tachycardia.

Educational objective: Patients with persistent tachyarrhy-thmia (narrow- or wide-complex) causing hemodynamic instability should be managed with immediate synchronized DC cardioversion_ Patients who have minimal symptoms and remain hemodynamically stable during an episode of regular, narrow-complex supraventricular tachycardia can be managed initially with a trial of vagal maneuvers (eg, carotid sinus massage) and/or intravenous adenosine.

1

3

5

7

• 9 • 10

• 11 • 12

13 14

• 15

17

19

21 22

• 23 • 24 • 25 ▪ 26

27 28 I

• 29 • 30

• 32 ■

• 33 ▪ 34-

• 36 • 37

I 39 • 40

41 42

44.

r

References:

1. Part 8: adult advanced cardiovascular life support: 2010 American Heart Association guidelines for cardiopulmonary resuscitation and emergency cardiovascular care.

Item: 15 of 44 Mark

Previous Next

Lab Values. Notes Calculator.

(AVNRT). and atrioventricular reentrant (or reciprocating) tachycardia_ An initial 12-lead electrocardiogram 44.

(ECG) can identify the correct arrhythmia in approximately 80% of patients_ However, in those with hemodynamic instability, immediate cardioversion should be performed prior to detailed evaluation due to the risk of rapid clinical deterioration.

(Choice A) Patients who have minimal symptoms or remain hemodynamically stable during an episode of regular, narrow-complex SVT can be managed initially with a trial of vagal maneuvers (eg, carotid sinus massage) or intravenous adenosine or calcium channel blockers_ Adenosine transiently decreases conduction through the AV node and is effective in terminating most episodes of AVNRT_

(Choices B and 0) Intravenous antiarrhythmics (eg. amiodarone, procainamide) can be used in patients with stable recurrent or refractory wide-complex tachycardia. In patients with unstable narrow-complex tachycardia, synchronized cardioversion is the initial therapy of choice_ These drugs can also worsen hypotension and are not recommended acutely in patients with hypotension or hemodynamic

(Choice C) Intravenous fluid resuscitation can improve blood pressure transiently_ However, it will not treat the underlying cause of hypotension and may result in unnecessary delay for definitive management of this patient

(Choice F) Verapamil is a calcium channel blocker that slows conduction through the AV node. It can be used as a second-line pharmacologic agent to terminate SVT in stable patients if initial vagal maneuvers and adenosine fail to terminate the tachycardia_

Educational objective: Patients with persistent tachyarty-thmia marrow- or wide-complex) causing hemodynamic instability should be managed with immediate synchronized DC cardioversion_ Patients who have minimal symptoms and remain hemodynamically stable during an episode of regular, narrow-complex supraventricular tachycardia can be managed initially with a trial of yagal maneuvers (eg, carotid sinus massage) andlor intravenous adenosine.

Copyright CI Morld Last updated: [9/9/2014]

1.1 Feedback, End Block

1270 Notes Calculator.

Item: 16 of 44 Lab Values.

r Mark Previous Next

e-TE, I-J I Clinically significant atherosclerotic disease

End Block 1.1 Feedback,

A 41-year-old man comes to the physician for a routine health examination_ He has no symptoms except for mild increase in frequency of urination. He has a history of elevated glucose levels but no other medical problems and takes no medications. His family history is significant for type 2 diabetes mellitus. The patient does not use tobacco, alcohol. or illicit drugs_ He works as an editor for a local magazine company. His temperature is 36_7 C (98 F), blood pressure is 132/80 mm Hg, pulse is 72/min, and respirations are 16 min. Body mass index is 30 kg/m2. Fasting laboratory results are as follows:

Glucose 165 mgldL Total cholesterol 210 mgldL Low-density lipoprotein 140 mgfdL Triglycerides 140 mgldL High-density lipoprotein 35 mg/dL Hemoglobin 1. 7.2% Serum creatinine 1.1 mgftIL

Which of the following is the most appropriate next step in management of this patient?

Explanation:

1

3

5

7 • a

• 9

• 10

• 11

• 12

• 13

• 14

• 15

16

• 17

I' 19

21

22

• 23

• 24

• 25

▪ 26

27

28 I

▪ 29

• 30

• 32 ■

• 33

▪ 34-

• 36

• 37

I 39

• 40

41

42

Guidelines for lipid-lowering therapy

Indication Recommended therapy

A. Fish oil supplementation [0%]

B. Initiation of oral glimepiride [13%]

C. Intense lifestyle modification only [23%]

a Lifestyle modification plus niacin [4%]

E Lifestyle modification plus rosuvastatin [60%]

Mark Item: 16 of 44 Lab Values, Notes Calculator. Previous Next

44.

Guidelines for lipid-lowering therapy

Indication Recommended therapy

Clinically significant atherosclerotic disease • ACS, MI • Stable or unstable angina • Coronary or other arterial' revascularization • Stroke, TIA, PAD

• Age 5.75: High-intensity stalk)

• Age >75: Moderate-intensity statin

,

LOL L190 mgidL High-intensity statin

Age 40-75 with diabetes

e 10-year ASCVD risk ?7.5%: High-intensity statin

• 10-year ASCVD risk <7.5%: Moderate-intensity statin

,

Estimated 10-year ASCVD risk ?.:7.5% (Pooled Cohort Equations)

Moderate- to high-intensity sign*

ACS = Acute coronary syndrome; ASCVD = atherosclerotic cardiovascular disease; MI = myocardial infarction; PAD = peripheral arterial disease; TIA = transient ischemic attack_

"'High-intensity statiris include atorvastatin 40-80 mg, rosuvastatin 20-40 mg; moderate-intensity stalins include atorvastatin 10-20 mg, rosuvastatin 5-10 mg, simvastatin 20-40 mg, pravastatin 40-80 mg, lovastatin 40 nig.

CLIWorld

This patient has increased frequency of urination with an elevated fasting glucose and hemoglobin Al C, confirming the diagnosis of new-onset type 2 diabetes mellitus_ Patients with diabetes have an increased risk of atherosclerotic cardiovascular disease compared to the general population_ Patients age >4Q with diabetes have been shown to benefit significantly from lipid-lowering therapy with statin drugs and initiation of appropriate dietary and exercise lifestyle modifications (Choice CY The dose of medication chosen is

1

I6 3

5

▪ 7

9

• 10

• 11

• 12

• 13

• 14

• 15

16

• 17

▪ -1M 19

21

22

• 23

• 24

• 25

▪ 26

27

▪ 29

• 1

' 32 111

• 33

• 34 El

' 36

• 37

• .39

• 40

41 42

Feedback, End Block

• 30

II I I

Notes . M ark

Next

-c:=1

Previous Calculator, Lab Values,

Item: 16 of 44

• I. I. LEM ro.••••.r• •vra••-•.• 1.1•1.r. b.

1.1 Feedback, End Block

1

3

5

7

• 9

4 10

• 11

• 12

• 13

• 14

• 15

16

• 17 •

19

21

22

• 23

• 24

• 25

▪ 26

27

28 I

• 29

4 30

• 32 ■

• 33

4 34-

• 36

• 37

4 39

4 40

41

42

44.

High-intensity statins include atorvastatin 40-80 mg, rusuvastatin 20-40 mg; moderate-intensity statins include atormstatin 10-20 mg, rostivastatin 5-10 mg, simvastaliri 20-40 mg, pravastatin 40-80 Mg, lovastaitin 40 mg.

OUWorld

This patient has increased frequency of urination with an elevated fasting glucose and hemoglobin Al C, confirming the diagnosis of new-onset type 2 diabetes mellitus. Patients with diabetes have an increased risk of atherosclerotic cardiovascular disease compared to the general population_ Patients age >40 with diabetes have been shown to benefit significantly from lipid-lowering therapy with statin drugs and initiation of appropriate dietary and exercise lifestyle modifications (Choice Cy The dose of medication chosen is partially dependent on overall risk of cardiovascular disease_ Diabetic patients age 40-75 with an estimated risk L7.5% should be given high-intensity statin therapy (atorvastatin 4G-80 mg or rosuvastatin 20-40 mg daily). Those with a 10-year risk <7_5% may be considered for moderate-intensity therapy (eg. atorvastatin 10-2G mg, simvastatin 2G-40 mg). This patient's estimated risk is >7.5% (risk calculator) and he therefore needs high-intensity statin therapy.

(Choice A) Fish oil supplements high in omega-3 polyunsaturated fatty acids can effectively reduce serum triglyceride levels_ However, a statin is the first-line therapy in patients with mild-to-moderate hypertriglyceridemia for whom the goal of therapy is cardiovascular risk reduction_ Fish oil supplementation has not been found to significantly improve cardiovascular outcomes or mortality_

(Choice 6) Most experts now recommend initiation of metformin therapy at the time of diagnosis of diabetes_ Some suggest that motivated patients with a hemoglobin Al c only minimally above goal (<7.5%) may consider a brief trial of intensive lifestyle modification before initiating medication_ But in either strategy, a sulfonylurea such as glimepiride would not be recommended as initial therapy_

(Choice 0) Niacin (nicotinic acid) is effective in raising high-density lipoprotein and lowering triglycerides and has a modest effect in lowering very low-density lipoprotein and low-density lipoprotein cholesterol_ However, it may worsen glucose control in diabetic patients. Studies have failed to show additional benefit of niacin in the prevention of coronary heart disease in patients on statin therapy_ Therefore. niacin is not recommended for this patient

Educational objective: Diabetic patients age 40-75 should be treated with statin therapy in addition to lifestyle modification and glucose control. Those with a 10-year risk of atherosclerotic cardiovascular disease <7_5% should receive moderate-intensity statin therapy. and those with a risk L7.5% should receive high-intensity statin therapy_

Item: 16 of 44

r Mark 127

Previous Next

Lab Values. Notes Calculator.

of atherosclerotic cardiovascular disease compared to the general population_ Patients age >40 with diabetes have been shown to benefit significantly from lipid-lowering therapy with statin drugs and initiation of appropriate dietary and exercise lifestyle modifications (Choice C). The dose of medication chosen is partially dependent on overall risk of cardiovascular disease_ Diabetic patients age 40-75 with an estimated risk X7.5% should be given high-intensity statin therapy (atorvastatin 40-80 mg or rosuvastatin 20-40 mg daily). Those with a 10-year risk <7.5% may be considered for moderate-intensity therapy (eg. atorvastatin 10-20 mg, simvastatin 20-40 mg). This patient's estimated risk is >7.5% (risk calculator) and he therefore needs high-intensity statin therapy_

(Choice A) Fish oil supplements high in omega-3 polyunsaturated fatty acids can effectively reduce serum triglyceride levels_ However, a statin is the first-line therapy in patients with mild-to-moderate hypertriglyceridemia for whom the goal of therapy is cardiovascular risk reduction_ Fish oil supplementation has not been found to significantly improve cardiovascular outcomes or mortality_

(Choice B) Most experts now recommend initiation of metformin therapy at the time of diagnosis of diabetes_ Some suggest that motivated patients with a hemoglobin Al c only minimally above goal (<L5%) may consider a brief trial of intensive lifestyle modification before initiating medication. But in either strategy, a sulfonylurea such as glimepiride would not be recommended as initial therapy.

(Choice ID) Niacin (nicotinic acid) is effective in raising high-density lipoprotein and lowering triglycerides and has a modest effect in lowering very low-density lipoprotein and low-density lipoprotein cholesterol_ However, it may worsen glucose control in diabetic patients_ Studies have failed to show additional benefit of niacin in the prevention of coronary heart disease in patients on statin therapy_ Therefore, niacin is not recommended for this patient

Educational objective: Diabetic patients age 40-75 should be treated with statin therapy in addition to lifestyle modification and glucose control. Those with a 10-year risk of atherosclerotic cardiovascular disease <7.5% should receive moderate-intensity statin therapy. and those with a risk X7.5% should receive high-intensity statin therapy_

References:

1. Primary prevention of major cardiovascular and cerebrovascular events with statins in diabetic patients: a meta-analysis.

Copyright @ UWorld Last updated: [1212612014]

1.1 Feedback, End Block

• 4

Item: 16 of 44 r Mar k

Previous Next 3 Lab Values. Notes Calculator.

Media Exhibit

5

▪ 6

▪ 7

• 8

• 9

• 10

• 11

• 12

• 13

• 14

• 15

1t • 17

• 18

• 19

• 20

• 21

• 22

• 23

• 24

• 25

• 26

• 27

• 28

• 29

• 30

• 32

• 33

• 34

• 36

• 38

• 39

• 40

• 41

Pooled Cohort Equation for estimating 10-year risk

Age

Female

Gender

[Male

Smoker

Lees

Diabetes

Calculate

Sample screenshot of 10-year ASCVD risk calculator

Race

I White

El African American

Other

Total Cholesterol (m.gidlL)

HDL - Cholesterol (rrigiciL)

Systolic Blood Pressure Treatment for Hypertension

Hes No

(DLISM•LEWeirld,il_C

No Yes No

i 41 Item: 17 of 44 rNlark Calculator Lab Values. Previous Notes Next

1.1 Feedback, End Block

44.

An 82-year-old male presents for evaluation of chronic back pain_ On physical examination. he is found to have a blood pressure of 160/85 mmHg while supine and 135/70 mmHg while standing. He is otherwise healthy; his only medicine is occasional ibuprofen for back pain. Which of the following age-related changes best explains the observed finding?

A_ Increased left ventricular wall stiffness [9%]

B_ Decreased left ventricular contractility [796]

• C_ Decreased baroreceptor responsiveness [7596]

D_ Decreased stress-mediated catecholamine release [6°A]

E Decreased glomerular filtration rate [3%]

Explanation:

This patient has orthostatic hypotension, which is defined as a postural decrease in blood pressure by 20 mmHg systolic or 10 mmHg diastolic (sometimes accompanied by an increase in heart rate) that occurs on standing_ In general, orthostatic hypotension results from insufficient constriction of resistance and capacitance blood vessels in the lower extremities on standing, which may be due to a defect in autonomic reflexes, decreased intravascular volume, or medications_ Some baroreceptor sensitivity is lost as a normal part of aging. Arterial stiffness, decreased norepinephrine content of sympathetic nerve endings, and reduced sensitivity of the myocardium to sympathetic stimulation all contribute to a tendency toward orthostatic hypotension with age_

(Choices A & 6) Normal changes to the aging heart include decreased resting and maximal cardiac output, decreased maximum heart rate, increased contraction and relaxation time of heart muscle, increased myocardial stiffness during diastole, decreased myocyte number, and pigment accumulation in myocardial cells_

(Choice 0) The adrenal medulla continues to release catecholamines in response to stress (fight or flight) throughout life. The adrenal medulla does not participate in the fine control of blood pressure.

(Choice E) While the GFR does tend to decline with old age, a decreased GFR would promote sodium retention and expansion of the intravascular volume, counteracting orthostatic hypotension_

1

5

• 7

• 9

• 10

i 11

i 12

• 13

• 14

16

17

• 19

21

22

• 23

• 24

• 25

• 26

27

I • 29

• 1

i 32 ■

• 33

• 34

• 36

• 37

• 39

• 40

I

• 30

1

1 6 3

5

▪ 7

▪ 9

• 10

• 11

• 12

• 13

• 14 1.

16

17

• 19 • -

21

22

• 23

• 24

• 25

• 26

27

I • 29

• 30

• 1

• 32 ■

• 33

• 34- El

• 36

• 37

• 39

• 40

41

42

Item: 17 of 44

r Mark 1>- CP,*

Previous Next

Lab Values Notes Calculator

A_ Increased left ventricular wall stiffness [9%]

11 Decreased left ventricular contractility [7%]

C_ Decreased baroreceptor responsiveness [75%]

D_ Decreased stress-mediated catecholamine release [6%]

E Decreased glomerular filtration rate [3%]

Explanation:

This patient has orthostatic hypotension. which is defined as a postural decrease in blood pressure by 20 mmHg systolic or 10 mmHg diastolic (sometimes accompanied by an increase in heart rate) that occurs on standing_ In general, orthostatic hypotension results from insufficient constriction of resistance and capacitance blood vessels in the lower extremities on standing. which may be due to a defect in autonomic reflexes, decreased intravascular volume. or medications. Some baroreceptor sensitivity is lost as a normal part of aging_ Arterial stiffness, decreased norepinephrine content of sympathetic nerve endings and recluc. ecl sensitivity of the myocardium to sympathetic stimulation all contribute to a tendency toward orthostatic hypotension with age_

(Choices A & B) Normal changes to the aging heart include decreased resting and maximal cardiac output decreased maximum heart rate, increased contraction and relaxation time of heart muscle, increased myocardial stiffness during diastole, decreased myouite number. and pigment accumulation in myocardial cells_

(Choice 0) The adrenal medulla continues to release catecholamines in response to stress (fight or flight) throughout life. The adrenal medulla does not participate in the fine control of blood pressure_

(Choice E) While the GFR does tend to decline with old age, a decreased GFR would promote sodium retention and expansion of the intravascular volume, counteracting orthostatic hypotension_

Educational objective: Progressively decreasing baroreceptor sensitivity and defects in the myocardial response to this reflex are the main reasons for the increased incidence of orthostatic hypotension in the elderly.

Copyright CI Morld

Last updated: [8/22/2014]

a 1.1

Feedback, End Block

End Block 1.1 Feedback,

"in i fi " rk lirrirko I r-li icialf=alf mrcia

Secondary causes of hypertension

1

3

5

▪ 7

▪ 9

• 10 • 11 • 12 . 13 . 14 1. . 15

I. 16 •

17 18

• 19 20 • 21

• 22

. 23

. 24

. 25

. 26 27

I . 29

• 32 111 . 33 • 34- .1

• 36 . 37

• 39 • 40

I

• 30

Item: 18 of 44 r Mark

Previous Next Lab Values. Notes Calculator.

A 42-year-old woman is evaluated for depression, mood swings, and poor sleep. She also complains of mild headaches and muscle weakness. She has had 2 emergency department visits for kidney stones over the past year. She does not use illicit drugs. Her blood pressure is 160.11G5 mm Hg and pulse is 85/min.

Laboratory results are as follows:

Sodium 140 mEq/L Potassium 3.6 mEq/L Chloride 101 mEq/L Bicarbonate 24 mEq/L Blood urea nitrogen 13 mg/dL Creatinine 0.9 mgldL Glucose 98 mg/dL Calcium 117 mgldL Albumin 37 gldL

Which of the following is the most likely cause of this patient's hypertension?

A. Renal parenchymal disease [7%]

E Renal vascular stenosis [9136]

C_ Coarctation of the aorta [0%]

D_ Hypothyroidism [5%]

E Parathyroid gland disease [72%]

• Cushing's syndrome [5%]

G. Amphetamine abuse [3%]

Explanation:

1270 Notes . Calculator.

Item: 18 of 44 Lab Values.

F. Mark Previous Next

111. Feedback, End Block

1

3

• . Secondary causes of hypertension

. Condition

. Clinical clues/features

. Renal parenchymal

disease

. • Elevated serum creatinine • Abnormal urinalysis (proteinuria, red blood cell casts)

.

.

Renovascular disease

. • Severe hypertension (180 mm Hg systolic and/or 120

mm Hg diastolic) after age 55

* Possible recurrent flash pulmonary edema or resistant heart failure

• Unexplained rise in serum creatinine

* Abdominal bruit , .

Primary aldosteronism

. * Easily provoked hypokalerriia • Slight hypematremia • Hypertension with adrenal incidentaloma

.

Pheochromocytoma

. • Paroxysmal elevated blood pressure with tachycardia • Pounding headaches, palpitations, diaphoresis • Hypertension with an adrenal incidentaloma

Cushing's syndrome

• Central obesity, facial plethora * Proximal muscle weakness, abdominal striae • Ecchymosis, amenorrhea1erectile dysfunction • Hypertension with adrenal incidentaloma

. ,

Hypothyroidism • Fatigue, dry skin, cold intolerance * reinqtinAtinn Idvpicilit rain hrArivrtArrii.A

44.

5

• 7

i 11

• 13

17

18

19

21

• 23

• 25

• 27

• 29

• 1

• 33

iV

• .39

• 41

End Block 1.1

Feedback,

: 18 of 44 r ,Mark Previous Next

Cushing's syndrome

.

• Central obesity, lacial plethora • Proximal muscle weakness, abdominal striae • Ecchymosis, amenoirhea,'erectile dysfunction • Hypertension with adrenal incidentaloma

, .

Hypothyroidism

. • Fatigue, dry skin, cold intolerance • Constipation, weight gain, bradycardia

, .

Primary

hyperparathyroidism

• Hypercalcemia (polyuria, polydipsia) • Kidney stones • Neuropsychiatric presentations (confusion, depression,

psychosis)

• Coarctation of the

aorta

• Differential hypertension with brachial-femoral pulse delay

-- --- -- ---- - - - - - ot ,

This patient's presentation (muscle weakness. recurrent nephrolithiasis, neuropsychiatric symptoms. and hypercalcemia) is consistent with primary hyperparathyroidism (PHPT). Excess parathyroid hormone causes hypercalcemia due to increased renal calcium reabsorption, gastrointestinal calcium absorption. and bone resorption_ Patients can present with bone pain. kidney stones, gastrointestinal symptoms. and neuropsychiatric symptoms ("bones, stones, abdominal moans, and psychic groans"). The majority (80°A) of PHPT cases are due to parathyroid adenoma.

PHPT can cause hypertension, arrhythmias, ventricular hypertrophy, and vascular and valvular calcification_ However, it is unclear how PHPT causes hypertension_ Significant hypertension with PHPT suggests possible multiple endocrine neoplasia syndrome type 2 with pheochromocytoma and requires further evaluation_

(Choice A) Renal parenchymal disease can cause secondary hypertension but usually presents with elevated serum creatinine and abnormal urinalysis. This patient's normal serum creatinine makes this unlikely_

IS17 Notes Calculator. Lab Values

1

I 3

I 5

• 7

• 9

• 10

• 11

• 12

• 13

• 14

• 15

17

18

• 19 20 •

21

• 22

• 23

• 24

• 25

25 JI 27

I • 29

• 1

• 32 ■

• 33

• 34

• 36

• 37

• 39

• 40

41

42

11

(Choice B) Renal artery stenosis can present with severe hypertension (MK mm Hg systolic and/or 120

• 30

[ References:

1. Secondary hypertension: evaluation and treatment.

2. Parathyroidectomy decreases systolic and diastolic blood pressure in hypertensive patients with primary hyperparathyroidism.

Item: 18 of 44

r Mark

1P4

:iii

Previous

Next

Lab Values, Notes

Calculator,

elevated elevated serum creatinine and abnormal urinalysis_ This patient's normal serum creatinine makes this unlikely.

(Choice B) Renal artery stenosis can present with severe hypertension 0180 mm Hg systolic ands or 120 mm Hg diastolic) after age 55, abdominal bruit on auscultation, recurrent flash pulmonary edema or resistant heart failure. and elevated serum creatinine (especially after starting an angiotensin-converting-enzyme inhibitor or angiotensin receptor blocker)_

(Choice C) Patients with coarctation of the aorta typically present with headaches, epistaxis. blurred vision, elevated blood pressure in the upper extremities, possible "to-and-fro machinery murmur" from the aorta over the posterior chest, and brachial-femoral pulse delay on examination_

(Choice 0) Hypothyroidism can cause hypertension but typically also presents with other symptoms, including weight gain, fatigue, bradycardia, and dry skin_ It would not explain hypercalcemia and renal stones_

(Choice F) Patients with Cushing's syndrome tend to have other findings such as central obesity, muscle wasting. thin skin, hirsutism, and purple striae. Laboratory findings show hyperglycemia, leukocytosis, lymphocytopenia, and hypokalemia_

(Choice G) Amphetamine abuse usually presents with signs and symptoms of sympathetic stimulation (eg, tachycardia. hypertension. diaphoresis. hyperthermia, confusion)_ It would not explain hypercalcemia and renal stones.

Educational objective: Hyperparathyroidism is an uncommon cause of secondary hypertension and should be suspected in patients with hypertension, hypercalcemia, renal stones, and neuropsychiatric symptoms_ Significant hypertension with primary hyperparathyroidism suggests possible multiple endocrine neoplasia syndrome type 2 with pheochromocytoma and requires further evaluation_

Copyright © liWorld Last updated: [1111/2014]

1.1 Feedback, End Block ,

1 2

3

4

G

• 117 Lab Values Notes Calculator. Next

Item: 18 of 44 Previous

Mark

Media Exhibit

Multiple endocrine neoplasia type 2 syndrome

Classification of multiple endocrine neoplasia

e Primary hyperparathyroidism (>90%) Type I • Enteropancreatic tumors (601(0-70%)

. Pituitary tumors (1013(0-20%)

0 MTC (>90%) Type 2A • Pheochromocytorna (4D%-50%)

• Parathyroid hyperplasia (10%-20%)

• MTC • Pheochrornocytorna

Type 2B • Other • iVlucosal & intestinal neurornas • Marfanoid habitus

OUSIALEWorld,LLC

G 5

6

7

8

9

10 11

12

• 13

• 14

• 15

• 16

• 17

18 • 19

• 20 • 21

• 22

• 23

• 24

• 25

• 26

• 27

• 28

29

30 •1

• 32

• 33

•4

36

38

39

40

1.1 Feedback, End Block

CP,*

Notes Item: 19 of 44

Calculator Lab Values. Previous, Next r Mark

A 54-year-old man conies to the physician after having woken up in the middle of the night with substernal discomfort, which he describes as a burning sensation. The patient also has left-sided neck pain and feels sweaty and short of breath_ He has never had similar pain before. His past medical history is significant for type 2 diabetes mellitus and hypertension_ He has a 30-pack-year smoking history_ Which of the following physical findings is most commonly associated with this patient's clinical presentation?

A_ Ejection-type systolic murmur 5%]

11 Fixed splitting of S2 [556]

vd C. Fourth heart sound [5956]

D_ Friction rub [11136]

E Pulsus paradoxus [996]

Explanation:

End Block 1.1

Feedback,

• Children

• Young adults

• Pregnancy

• Age >40

• Heart failure

• Restrictive cardiomyopathy

• High-output states

Third heart sound

(S3)

Gallop heart sounds

Abnormallassociated conditions

• Ventricular gallop sound (after S2)

• Heard during rapid filling of ventricles in diastole

• Turbulent blood flow to the ventricles due to increased volume

Normal Features

1

3

5

• 7

• 9 • 10 • 11

• 12

• 13

• 14 1.

• 15

• 16

• 17

• 18

19 • 20.11

22

• 23

• 24

• 25

• 26

27

• 29

• 1

• 32 111

• 33

• 34 El iV

• 36

• 37

• 39

• 40

• 21

I

• 30

• Lab Values Notes . Calculator.

Item: 19 of 44 F. Mark Previous Next

111. Feedback, End Block

This patient's presentation with substernal discomfort. left-sided neck pain. diaphoresis, and dyspnea is consistent with acute coronani syndrome. Acute myocardial ischemia or infarction causes myocardial dysfunction and stiffening of the left ventricular myocardium and can lead to changes in normal heart sounds. An abnormal S4 (atrial gallop) can be heard in most patients during the acute phase of myocardial infarction due to ischemia induced myocardial dysfunction.

1

3

5

• 6

• 7 • 8

• 9

• 10

• 11

• 12

• 13

• 17

19

• 211

• 23

• 25

• 27

• 29

• 33

• 39

• 41

Gallop heart sounds

Feature s Normal Abnormal/associated

conditions

Third heart sound

(S3)

• VentricuJar gallop sound (after S2)

• Heard during rapid filling of ventricles in diastole

• Turbulent blood flow to the ventricles due to increased volume

• Children

• Young adults

• Pregnancy

• Age >40

• Heart failure

• Restrictive cardiomyopathy

• High-output states

Fourth heart sound

(S4)

• Atrial gallop sound (before 81)

• Heard immediately after atrial contraction phase as blood is forced into a stiff ventricle

• Healthy older adults

• Younger adults, children

• Ventricular hypertrophy

• Acute myocardial infarction

44.

©LIWorld

• 1

▪ 3

G 5

7

• 9

• 10 ▪ 11

• 12 • 13 • 14 • 15 • 16 • 17 ▪ 18 mi • 19 • 2 • 21

• 22 • 23 • 24 • 25 • 26 • 27

• 28 I

• 29 • 30

• 32 • 33 ▪ 34- • '1=.

• 36 • 37 •

• 39 • 40 • 41 • 42

Item: 19 of 44

r Mark 1>-

Previous Next Lab Values, Notes

Calculator

This patient's presentation with substernal discomfort. left-sided neck pain. diaphoresis. and dyspnea is consistent with acute coronary syndrome_ Acute myocardial ischemia or infarction causes myocardial dysfunction and stiffening of the left ventricular myocardium and can lead to changes in normal heart sounds_ An abnormal S4 (atrial gallop) can be heard in most patients during the acute phase of myocardial infarction due to ischemia induced myocardial dysfunction.

An S4 is also frequently heard in patients with decreased left ventricular compliance. including those with hypertensive heart disease, aortic stenosis. and hypertrophic cardiomyopathy. It can also sometimes be heard in healthy older adults without any other cardiac abnormality-.

(Choice A) Ejection-type systolic murmurs typically reflect the flow of blood across the aortic or pulmonic valves_ They are not usually associated with acute myocardial ischemia or infarction_

(Choice B) Wide and fixed splitting of second heart sound (S2) is typically heard in patients with an atrial septal defect Patients with myocardial ischemialinfarction can have paradoxical splitting of 52 due to delayed myocardial relaxation and delayed closure of the aortic valve_

(Choice 0) Pericardial friction rub is occasionally heard in patients with acute pericarditis: the chest pain is often sharp and pleuritic, worsens with inspiration or coughing, and diminishes when the patient sits up and leans forward. However, this patient's clinical features are more suggestive of acute coronary syndrome.

(Choice E) Pulsus paradoxus refers to an exaggerated drop in systolic blood pressure (11:[ mm Hg) during inspiration_ It is most commonly seen in patients with cardiac tamponade but can also occur in severe asthma and chronic obstructive pulmonary disease, hypovolemic shock, and infrequently with constrictive pericarditis.

Educational objective: An abnormal fourth heart sound (atrial gallop) can often be heard during the acute phase of myocardial infarction due to left ventricular stiffening and dysfunction induced by myocardial ischemia.

References:

1. Frequency of diastolic third and fourth heart sounds with myocardial ischemia induced during percutaneous coronary intervention.

2. Diastolic heart sounds as an adjunctive diagnostic tool with ST criteria for acute myocardial ischemia.

1.1 Feedback,

End Block

Item: 19 of 44

r Mark

I

' 4101.. Previous Next

Lab Values, Notes Calculator

dysfunction and stiffening of the left ventricular myocardium and can lead to changes in normal heart sounds. An abnormal S4 (atrial gallop) can be heard in most patients during the acute phase of myocardial infarction due to ischemia induced myocardial dysfunction.

An S4 is also frequently heard in patients with decreased left ventricular compliance. including those with hypertensive heart disease, aortic stenosis, and hypertrophic cardiomyopathy. It can also sometimes be heard in healthy older adults without any other cardiac abnormality.

(Choice Al Ejection-type systolic murmurs typically reflect the flow of blood across the aortic or pulmonic valves. They are not usually associated with acute myocardial ischemia or infarction_

(Choice B) Wide and fixed splitting of second heart sound (S2) is typically heard in patients with an atrial septal defect Patients with myocardial ischemia/infarction can have paradoxical splitting of 52 due to delayed myocardial relaxation and delayed closure of the aortic valve_

(Choice 0) Pericardial friction rub is occasionally heard in patients with acute pericarditis; the chest pain is often sharp and pleuritic, worsens with inspiration or coughing, and diminishes when the patient sits up and leans forward_ However, this patient's clinical features are more suggestive of acute coronary syndrome.

(Choice E) Pulsus paradoxus refers to an exaggerated drop in systolic blood pressure (10 mm Hg) during inspiration. It is most commonly seen in patients with cardiac tamponade but can also occur in severe asthma and chronic obstructive pulmonary disease, hypovolemic shock, and infrequently with constrictive pericarditis.

Educational objective: An abnormal fourth heart sound (atrial gallop) can often be heard during the acute phase of myocardial infarction due to left ventricular stiffening and dysfunction induced by myocardial ischemia_

References:

1. Frequency of diastolic third and fourth heart sounds with myocardial ischemia induced during percutaneous coronary intervention.

2. Diastolic heart sounds as an adjunctive diagnostic tool with ST criteria for acute myocardial ischemia.

Copyright © liWorld

Last updated: [8/13/2014]

1

1.1 Feedback, End Block

1

5

• 7

• 9

• 10 • 11 • 12 • 13 • 14 • 15 • 16 •

17

• 19 a 20

21 22

I, 23

• 24 • 25 • 26

27

I • 29

• 32 ■ • 33 • 34-

• 36 • 37

• 39 • 40

I

• 30

Item: 20 of 44

r Mark Previous Next Lab Values. Notes Calculator

44.

A 24-year-old man experiences syncope while shoveling snow_ He regains consciousness within 1 minute_ He has had some shortness of breath and chest pains recently, related primarily to exercise. He denies illicit drug use. His temperature is 37.2° C (98.9° F), blood pressure is 126/76 mm Hg, pulse is 76/min, and respirations are 14/min_ Physical examination shows a well-built man in no apparent distress_ Lungs are clear_ A crescendo-decrescendo systolic murmur is heard along the left sternal border without carotid radiation_ Chest x-ray is normal_ Which of the following is the most likely cause of his syncopal episode?

Atrioventricular conduction delay [2%]

11 Inter-ventricular septal hypertrophy [63%]

C_ Aortic dissection [1 lif[i]

• Aortic stenosis [30%]

E Mitral valve degeneration [4%]

Explanation:

This patient's crescendo-decrescendo systolic murmur along the left sternal border without carotid radiation is the description of the murmur present in hypertrophic obstructive cardiomyopathy (HOLM), which is inter-ventricular septal hypertrophy_ Common symptoms of HOLM include syncope, dyspnea, and chest pain, all of which this patient has experienced. Syncope in HOCM is multifactorial. In large part it is due to outflow obstruction from the hypertrophied myocardium_ However, syncope can also be secondary to arrhythmia, ischemia, and a ventricular baroreceptor response that inappropriately causes vasodilation_

(Choice A) Atrioventricular conduction delay occurs when conduction through the AV node is slowed, typically due to medications such as p-blockers or ischemic heart disease. This typically results in bradycardia, but this patient's heart rate is within the normal range.

(Choice C) Syncope can occur with aortic dissections, particularly those involving the ascending aorta. However, the primary complaint in patients with aortic dissection is chest pain, which this patient does not appear to have. Painless aortic dissection is rare. Most patients with aortic dissection are also older and hypertensive.

(Choice 0) Isolated aortic stenosis in a young adult without a familial syndrome would be unusual but is a rAimp nf p'%.prr iqp_rpWpri nnp Thp InrAtinn nf the rniirmiir (rinht qinr nnrl intprrntAl qnqrp fnr qnrtir

1.1 Feedback, End Block

G 5

Item: 20 of 44 1>- Mark Calculator Lab Values. Previous Notes Next

Copyright CI Morld Last updated: [83/201 .E1] a

End Block 1.1

Feedback,

i_nr.p 11;1 1 1.1;1 ILI UP 1 I .

This patient's crescendo-decrescendo systolic murmur along the left sternal border without carotid radiation is the description of the murmur present in hypertrophic obstructive cardiomyopathy (HOCM), which is inter-ventricular septal hypertrophy. Common symptoms of HOCM include syncope, dyspnea, and chest pain, all of which this patient has experienced_ Syncope in HOCM is multifactorial_ In large part it is due to outflow obstruction from the hypertrophied myocardium_ However, syncope can also be secondary to arrhythmia:. ischemia, and a ventricular baroreceptor response that inappropriately causes vasodilation_

(Choice Al Atrioventricular conduction delay occurs when conduction through the AV node is slowed, typically due to medications such as r3-blockers or ischemic heart disease_ This typically results in bradycardia. but this patient's heart rate is within the normal range_

(Choice C) Syncope can occur with aortic dissections, particularly those involving the ascending aorta_ However, the primary complaint in patients with aortic dissection is chest pain, which this patient does not appear to have_ Painless aortic dissection is rare_ Most patients with aortic dissection are also older and hypertensive_

(Choice 0) Isolated aortic stenosis in a young adult without a familial syndrome would be unusual but is a cause of exercise-related syncope. The location of the murmur (right second intercostal space for aortic stenosis) and lack of right carotid radiation help to differentiate it from aortic stenosis_

(Choice E) Mitral valve degeneration leading to regurgitation rarely causes syncope_ In addition, mitral regurgitation typically causes a holosystolic murmur as opposed to the crescendo-decrescendo type murmur heard in this patient

Educational objective: Syncope in a young patient with a crescendo-decrescendo murmur at the lower left sternal border is most likely due to hypertrophic obstructive cardiomyopathy (HOCM)_ Syncope in HOCM is often multifactorial and can be due to outflow obstruction, arrhythmia, ischemia, and a ventricular baroreceptor response that inappropriately causes vasodilation.

• 1

▪ 3

7

• 9

• 10

▪ 11

• 12

• 13

• 14

• 15

• 16

• 17

▪ 1 • 19

• 20 • 21

• 22 • 23

• 24

• 25

• 26

• 27

• 28 I

• 29

• 30

• 32

• 33

▪ 34- • '1=.

I 36

• 37 •

I 39

• 40

• 41

• 42

References:

1. The diagnosis and treatment of hypertrophic cardiomyopathy

1

5

• 7

• 9

• 10

• 11

• 12

• 13

• 14

• 15

• 16 • 17

• 19

• 20

a 21

• 22

• 23

• 24

• 25

• 26

• 27

I • 29

• 1

• 32 ■

• 33

• 34

• 36

• 37

• 39

• 40

I

• 30

Item: 21 of 44

r Mark 117

Previous Next

Lab Values. Notes Calculator.

44.

A 53-year-old man is admitted to the hospital with a 4-week history of fatigue and decreased exercise tolerance. Climbing 2 flights of stairs causes significant dyspnea_ He has had occasional palpitations for months but no chest pain_ Past medical history is unremarkable, and a routine check-up 6 months ago was normal. He had 2 binge-drinking episodes last month but usually only drinks in moderation_ The patient does not routinely take any medications_ His blood pressure is 150/90 mm Hg and pulse is 130/min and irregular_ Lungs are clear on auscultation_ Electrocardiogram (ECG) does not show clear P waves_ Echocardiography shows ejection fraction of 35%, moderate central mitral regurgitation. and left atrial and left ventricular dilatation with global hypokinesis_ Which of the following interventions is most likely to restore left ventricular function in this patient'?

A. Coronary revascularization [3%]

B_ Decreasing afterload [19%]

C_ Inotropic medications VI 0%]

D_ Preload optimization [6%]

E Rate or rhythm control [50%]

F. Valve surgery [11 IC]

Explanation:

This patient's presentation with progressive dyspnea, decreased exercise tolerance, atrial fibrillation (AF) with rapid ventricular response, and left ventricular (LV) systolic dysfunction, is consistent with tachycardia-mediated cardiomyopathy_ A variety of tachyarrhythmias with prolonged periods of rapid ventricular rates can lead to tachycardia-mediated cardiomyopathy; these include AF, atrial flutter, ventricular tachycardia, incessant atrial/junctional tachycardia, and atrioventricular nodal reentrant tachycardia. Chronic tachycardia causes structural changes in the heart including LV dilatation and myocardial dysfunction_

Most patients have palpitations and/or signs and symptoms of congestive heart failure_ Diagnosis requires electrocardiogram, echocardiography, and assessment for other causes of LV dysfunction including coronary artery disease (CAD), especially in patients with symptoms or risk factors suggestive of CAD_ Treatment includes aggressive rate control or restoration of normal sinus rhythm due to potential reversibility of tachycardia-mediated cardiomyopathy and normalization of LV systolic function_ Therapeutic options include AV nodal blocking agents. antiarrhvthmic drugs. and catheter ablation of arrhythmia_

1.1 Feedback, End Block

• 1

▪ 3

G 5

7

• 9

• 10

• 11

• 12

• 13

• 14

• 15

• 16

• 17 • 1a-■ • 19

• • 20

• 21 • 22 • 23

• 24

• 25

▪ 26

• 27

28 I • 29

• 30

• 32

• 33

▪ 34- • '1=.

• 36

• 37 •

I 39

• 40

• 41

• 42

Item: 21 of 44

r Mark Previous Next

Lab Values. Notes Calculator

This patient's presentation with progressive dyspnea, decreased exercise tolerance. atrial fibrillation (AF) with rapid ventricular response, and left ventricular (LV) systolic dysfunction, is consistent with tachycardia-mediated cardiomyopathy_ A variety of tachyarrhythmias with prolonged periods of rapid ventricular rates can lead to tachycardia-mediated cardiomyopathy; these include AF, atrial flutter, ventricular tachycardia, incessant atrial/junctional tachycardia, and atrioventricular nodal reentrant tachycardia. Chronic tachycardia causes structural changes in the heart including LV dilatation and myocardial dysfunction_

Most patients have palpitations and/or signs and symptoms of congestive heart failure_ Diagnosis requires electrocardiogram, echocardiography, and assessment for other causes of LV dysfunction including coronary artery disease (CAD), especially in patients with symptoms or risk factors suggestive of CAD_ Treatment includes aggressive rate control or restoration of normal sinus rhythm due to potential reversibility of tachycardia-mediated cardiomyopathy and normalization of LV systolic function_ Therapeutic options include AV nodal blocking agents, antiarrhythmic drugs. and catheter ablation of arrhythmia_

(Choice A) Coronary artery revascularization can lead to significant improvement in LV function in patients with left main or multivessel CAD_ This patient had a normal routine exam 6 months ago and has no risk factors or symptoms suggestive of CAD_

(Choices B, C, and D) Preload optimization with diuretics and inotropic medications (eg, digoxin) are useful for initial symptom control in patients with decompensated congestive heart failure. Beta-blockers and angiotensin converting enzyme inhibitors (afterload reduction) are also important components of long-term optimal medical therapy in patients with LV systolic dysfunction_ However, tachycardia-mediated cardiomyopathy is potentially reversible, and all attempts should be made to restore sinus rhythm and/or control ventricular heart rate_

(Choice F) Mitral regurgitation (MR) in this patient is due to mitral annular enlargement secondary to LV dilatation (functional MR). Mitral valve repair or replacement will not improve LV function and is not recommended_

Educational objective: Tachycardia-mediated cardiomyopathy can develop in patients who have persistent or recurrent tachyarrhythmia with prolonged periods of rapid ventricular rate_ Initial treatment is aimed at restoration of sinus rhythm or aggressive control of ventricular rate and can lead to significant improvement in left ventricular function_

TReferences:

1.1 Feedback, End Block

G 5

7

• 9

• 10

• 11

▪ 12

• 13

• 14

• 15

• 16

• 17

• 1 • 19

• 20

-c21

Previous Item: 21 of 44 1.401°-

Notes Calculator Lab Values. Mark

Next

Copyright © liWorld Last updated: [8/21/2014] a

End Block 1.1

Feedback,

Most patients have palpitations and/or signs and symptoms of congestive heart failure. Diagnosis requires electrocardiogram, echocardiography, and assessment for other causes of LV dysfunction including coronary artery disease (CAD), especially in patients with symptoms or risk factors suggestive of CAD_ Treatment includes aggressive rate control or restoration of normal sinus rhythm due to potential reversibility of tachycardia-mediated cardiomyopathy and normalization of LV systolic function_ Therapeutic options include AV nodal blocking agents. antiarrhythmic drugs. and catheter ablation of arrhythmia_

(Choice A) Coronary artery revascularization can lead to significant improvement in LV function in patients with left main or multivessel CAD_ This patient had a normal routine exam 6 months ago and has no risk factors or symptoms suggestive of CAD_

(Choices B, C, and D) Preload optimization with diuretics and inotropic medications (eg. digoxin) are useful for initial symptom control in patients with decompensated congestive heart failure. Beta-blockers and angiotensin converting enzyme inhibitors (afterload reduction) are also important components of long-term optimal medical therapy in patients with LV systolic dysfunction_ However, tachycardia-mediated cardiomyopathy is potentially reversible, and all attempts should be made to restore sinus rhythm and/or control ventricular heart rate.

(Choice F) Mitral regurgitation (MR) in this patient is due to mitral annular enlargement secondary to LV dilatation (functional MR). Mitral valve repair or replacement will not improve LV function and is not recommended.

Educational objective: Tachycardia-mediated cardiomyopathy can develop in patients who have persistent or recurrent tachyarrhythmia with prolonged periods of rapid ventricular rate_ Initial treatment is aimed at restoration of sinus rhythm or aggressive control of ventricular rate and can lead to significant improvement in left ventricular function.

• 1

▪ 3

• 21 • 22

• 23

• 24

• 25

▪ 26

• 27

28 I • 29

• 30

▪ 32

• 33

▪ 34- • '1=.

• 36

• 37 •

I 39

• 40

• 41

• 42

References:

1. Left ventricular dysfunction due to atrial fibrillation in patients initially believed to have idiopathic dilated cardiomyopathy.

2. Tachycardia-induced heart failure.

1

5

• 7

• 9

• 10

• 11

• 12

• 13

• 14

• 15

• 16 • 17

20

21

• 22 23

24

• 25

• 26

• 27

I • 29

• 1

• 32 ■

• 33

• 34-

• 36

• 37

• 39

• 40

I

• 30

Item: 22 of 44

rNlark

127

Previous

Next

Lab Values, Notes Calculator.

44.

A 36-year-old woman comes to the emergency department complaining of chest pain that started suddenly while she was shopping at the mall. She also reports shortness of breath, palpitations, and diaphoresis. The pain is retrosternal and radiates to the left arm. There are no aggravating or relieving factors. She appears to be in mild discomfort On review of systems, the patient reports having had a runny nose, sore throat, and dry cough for the past 3 days. Her other medical problems include panic attacks treated with paroxetine and dysfunctional uterine bleeding treated with estrogen. Family history is significant for her father's sudden death at age 44 from a heart attack_ The patient has a 15-pack-year smoking history. Her blood pressure is 144/90 mm Hg and pulse is 1041min and regular. Her body mass index is 29 kg/m2. Which of the following is the most appropriate initial therapy for this patient?

A. Acetaminophen [1%]

• 11 Aspirin [189q

C. Heparin [8%]

a Ibuprofen [39C]

E. Lorazepam [10N

F. Oxycodone [196]

Explanation:

Evaluation of chest pain in the emergency department

• Focused history &

physical examination

• Assess vital signs

• Obtain venous access

Stable patient

Unstable patient

1.1 Feedback, End Block

Mark Item: 22 of 44 Lab Values. Notes Calculator. Previous Next

Stable patient Unstable patient

• Obtain ECG & chest x-ray

• Administer aspirin lithe

risk for aortic dissection

is low

• Stabilize hemodynarnics

• Check for underlying causes

Treat cause

N$TEM.1

Treat with

appropriate

anticoagulation

STEN

Treat with

emergency

catherization or

thrombolysis

No

• AZe55 fOr puld-nOrlary embolism • Check cardiac markers & risk

1

3

5

• 6

▪ 7 ▪ 8

▪ 9

• 1Q

• 11

• 12

• 13

• -15

17

1 a—. 19

20 I 21

• 22

23

▪ 24

• 25

▪ 26

• 27

▪ 28 I

• 29

• 30 • 1

▪ 32 111 • 33

• 34

• 36

• 38

• 39

• 40

• 41 42

Evaluation of chest pain in the emergency department

• Focused history &

physical examination

• Assess vital signs

• Obtain venous access

ECG consi5tent with ACS?

NO

I Chest x-ray diagnostic?

44.

Yes

1

1 6 3

5

• 7

• 9

• 10 • 11 • 12 • 13 • 14 • 15 • 16 •

17 1 ■

• 19 20 21

• 22 23 24

• 25 • 26 • 27

I • 29

• 1 • 32 ■

• 33 • 34-

• 36 • 37

• 39 • 40

• 30

Item: 22 of 44 Mark -<1

IMO Previous Next Lab Values. Notes Calculator

LASNILEWN.Iii LEK

stratify for AC5

• Assess for pericarditis

• Assess for aortic dissection

44.

This patient's presentation and risk factors (smoking, family history, estrogen therapy) suggest acute coronary syndrome (ACS). Retrosternal chest pain radiating to the left arm with associated diaphoresis, nausea, and/or vomiting usually (but not always) suggests ACS. Regardless of the etiology, all patients who present to the emergency department with chest pain should have a focused history, physical examination, and assessment of vital signs. Unstable patients with abnormalities in airway, breathing, circulation, and/or life-threatening arrhythmias should be stabilized and assessed for underlying causes (eg, pulmonary embolism [PE], pericarditis, aortic dissection).

Stable patients should receive oxygen, intravenous access, electrocardiogram (ECG), and chest x-ray. Patients with possible ACS and low risk for aortic dissection should receive aspirin as soon as possible. Aspirin inhibits thromboxane A2 production to exert significant antiplatelet effects_ It significantly reduces the rate of myocardial infarction (Ml), stroke, and overall mortality in ACS_ Patients with ECG findings of ACS should be treated appropriately (eg, heparin). Those with nondiagnostic chest x-ray and ECG should have further workup for possible causes such as non-ST elevation Ml, PE, aortic dissection, and acute pericarditis_

(Choices A and F) Acetaminophen and oxycodone can be given for musculoskeletal chest pain. This patient has no history of recent trauma or mention of reproducible pinpoint tenderness on chest examination to suggest a musculoskeletal etiology_

(Choice C) Heparin is used for Ml (positive cardiac markers or ST elevation on ECG) or a high suspicion and/or confirmation of PE. This patient could have a PE but does not have a high probability by Well's criteria_ ACS is more likely given her presentation and risk factors and should be ruled out first In addition, aspirin is given initially before heparin in ACS_

(Choice 0) Ibuprofen is used in the treatment of pericarditis. Although pericarditis is possible with the patient's prior history of recent upper respiratory tract infection, ACS must be evaluated first given her high risk. Chest pain in acute pericarditis improves with leaning forward_

(Choice E) Lorazepam is a benzodiazepine used for chest pain stemming from an acute panic attack_ Although this patient has a history of panic attacks, her current history is more concerning for ACS, which should be ruled out first.

1.1 Feedback, End Block

WO' ■

Notes Calculator.

Item: 22 of 44 Lab Values.

r Mark Previous Next

Copyright © UWorld Last updated: [12/7/2014]

End Block 1.1

Feedback,

G 5

111 1._. -L111 L.L1L1._•1 III 19 L1111 iy LI 11111L1 •LL•14,1 111.1_ 1._•4,1 uI 1U 141 UI 1U1._•1 ly 1119 L•LIL.I. 1._•• IL,1 U11 114,1 11L41 y

embolism [PE], pericarditis, aortic dissection).

Stable patients should receive oxygen, intravenous access, electrocardiogram (ECG), and chest x-ray. Patients with possible ACS and low risk for aortic dissection should receive aspirin as soon as possible_ Aspirin inhibits thromboxane A2 production to exert significant antiplatelet effects_ It significantly reduces the rate of myocardial infarction (Ml), stroke, and overall mortality in ACS_ Patients with ECG findings of ACS should be treated appropriately (eg, heparin). Those with nondiagnostic chest x-ray and ECG should have further workup for possible causes such as non-ST elevation Ml, PE, aortic dissection. and acute pericarditis.

(Choices A and F) Acetaminophen and oxycodone can be given for musculoskeletal chest pain_ This patient has no history of recent trauma or mention of reproducible pinpoint tenderness on chest examination to suggest a musculoskeletal etiology.

(Choice C) Heparin is used for Ml (positive cardiac markers or ST elevation on ECG) or a high suspicion and/or confirmation of PE This patient could have a PE but does not have a high probability by Well's criteria_ ACS is more likely given her presentation and risk factors and should be ruled out first_ In addition. aspirin is given initially before heparin in ACS_

(Choice 0) Ibuprofen is used in the treatment of pericarditis_ Although pericarditis is possible with the patient's prior history of recent upper respiratory tract infection, ACS must be evaluated first given her high risk. Chest pain in acute pericarditis improves with leaning forward_

(Choice E) Lorazepam is a benzodiazepine used for chest pain stemming from an acute panic attack_ Although this patient has a history of panic attacks, her current history is more concerning for ACS; which should be ruled out first.

Educational objective: Patients presenting to the emergency department with chest pain and suspected acute coronary syndrome (ACS) should be administered aspirin as soon as possible_ Early antiplatelet therapy with aspirin reduces the rate of myocardial infarction and overall mortality in patients with ACS.

• 1

▪ 3

7

• 9

• 10

▪ 11

▪ 12

• 13

• 14

• 15

• 16

• 17

• 1 • 19

• 20 • 21

• 22 • 23

• 24

• 25

▪ 26

• 27

28 I • 29 • 30

▪ 32

• 33

▪ 34- • '1=.

▪ 36

• 37 •

I 39

• 40

• 41

• 42

References:

1. Emergency department and office-based evaluation of patients with chest pain.

2

3 ik• Next Previous II FP Mar k

Lab Values.

Item: 22 of 44 117 Notes Calculator.

4 •

22 •

• 6

▪ 8

• 10

• 12

• 14

• 16

• 18

• 20

• 24

• 26

• 28

• 30

▪ 5

• 7

▪ 9

• 11

• 13

• 15

• 17

• 19

• 21

• 23

• 25

• 27

• 29

• 32

• 33

• 34 • '1=.

• 36

• 38

• 39

• 40

• 41

42

Media Exhibit

Feedback, End Block

Modified Vliells criteria

Modified Wells criteria for

pretest probability of PE

Score +3 points

• Clinical signs of DVT

• Alternate diagnosis Fess likely than PE

Score +1.5 points

• Previous PE or DVT

• Heart rate >100

• Recent surgery (5 4 weeks) or

immobilization 3 days)

Score +1 point

• Hemoptysis

• Cancer

Total score for clinical probability

< 4 = PE unlikely

> 4 = PE likely

UShilLEINotld, iLIC

j

1

1 6 3

5

▪ 7

▪ 9

• 10

• 11 • 12

• 13

• 14 1.

• 15

• 16 •

• 17

19

ARM

• 21

▪ 22-11 23

• 24

• 25

• 26

• 27

I • 29

• 1

• 32 111

• 33

• 34- El

• 36

• 37

• 39

• 40

• 30

Item: 23 of 44

r Mark CP,*

Previous Next

Lab Values, Notes Calculator

An 82-year-old woman comes to the emergency department with sudden-onset epigastric pain. The pain started an hour ago and is 8/1 0 in severity. The patient has been nauseated and vomited twice in the past hour. Her past medical history includes type 2 diabetes mellitus, hyperlipidemia, hypertension, peptic ulcer disease 10 years ago, and cholelithiasis. She has a 40-pack-year smoking history and drinks alcohol occasionally_ Her temperature is 36_8 C (98.2 F). blood pressure is 140/80 mm Hg, pulse is 90/min, and respirations are 141min_ The lungs are clear to auscultation_ Abdominal examination shows a soft abdomen with no tenderness_ Murphy's sign is negative. Which of the following studies should be performed first in this patient?

A. Abdominal ultrasound [12%]

11 Electrocardiogram [61%]

C_ Serum amylase and lipase [1€%]

D_ Upper gastrointestinal endoscopy [2°A]

E Upright abdominal x-ray [9%]

Explanation:

Relative frequency of selected presenting symptoms in acute coronary syndrome

Chest pain 80%-85%

Dyspnea 1096-75%

Nausea 4096-55%

Vomiting 15%-20%

Epigastric pain 1096-15%

An electrocardiogram (ECG) is the essential first step in a patient with history and risk factors for coronary artery disease who has symptoms consistent with acute coronary syndrome (ACS). There are a number of well-known atypical presentations (ie. no overt chest pain) of ACS that should prompt a cardiac evaluation_ Unfortunately, patients with atypical symptoms are more like to have a delay in diagnosis and under-treatment, which contributes to worsened outcomes. Women, the elderly, and patients with diabetes in particular ices rrn rc. Illralir to k fr. i r I rd. rrn c. • 11n f r. 91104 rif 1kaca I-. fi rl-r• rIrr nnt• rar rr-F rtaco nei a times rtf

1.1 Feedback, End Block

G 5

CP,*

Notes

Item: 23 of 44 Calculator Lab Values.

Mark Previous Next

End Block 1.1

Feedback,

Relative frequency of selected presenting symptoms in acute coronary syndrome

Chest pain 8C6.6-85%

Dyspnea I0%-75%

Nausea 40%-55%

Vomiting 15%-20%

Epigastric pain 10%-15% _

An electrocardiogram (EGG) is the essential first step in a patient with history and risk factors for coronary artery disease who has symptoms consistent with acute coronary syndrome (ACS). There are a number of well-known atypical presentations (ie, no overt chest pain) of ACS that should prompt a cardiac evaluation_ Unfortunately, patients with atypical symptoms are more like to have a delay in diagnosis and under-treatment, which contributes to worsened outcomes_ Women, the elderly, and patients with diabetes in particular are more likely to have atypical symptoms; up to 20% of these patients do not report chest pain at the time of presentation_ Dyspnea, epigastric pain, and nausea/vomiting are common atypical presentations.

(Choices A, C, ID, and E) Abdominal ultrasound is useful for diagnosing acute cholecystitis_ Serum amylase and lipase are very helpful tests to support the diagnosis of pancreatitis_ Upper gastrointestinal endoscopy should be considered in patients with evidence of acute gastrointestinal blood loss or in the evaluation of possible peptic ulcer disease (PUD)_ An upright abdominal x-ray would be helpful in diagnosing PUD complicated by perforation as air may be seen under the diaphragm_ All may need to be considered in this patient but are less immediately life threatening: they should be evaluated after workup for a cardiac etiology has been initiated_

Educational objective: Abdominal pain and nausea/vomiting are atypical presentations of acute coronary syndrome_ In select patients, cardiac causes must be excluded prior to pursuing other etiologies_

• 1

▪ 3

7

• 9

• 10

• 11

• 12

• 13

• 14

• 15

• 16

• 17

• 19

• 2

• 21

• 22 23

• 24

• 25

• 26

• 27

28 I

• 29

• 30

• 32

• 33

▪ 34-

• 36

• 37 •

I 39

• 40

I

• 41

• 42

References:

1. Symptom presentation of acute myocardial infarction: influence of sex, age, and risk factors.

2. Sex differences in symptom presentation in acute myocardial infarction: a systematic review and meta-analysis.

• 1

▪ 3

G 5

7

• 9

• 10

i 11

i 12

• 13

• 14

• 15

• 16

• 17

• 19

• 2

• 21

• 22

.1. 23 • 24

• 25

• 26

• 27

28 I

• 29 • 30

i 32

• 33

▪ 34-

• 36

• 37 •

I 39

• 40

I Item: 23 of 44 Mark

SI

117 Previous Next Lab Values. Notes . Calculator.

Nausea

Vomiting

Epigastric pain

15%-20%

10%-15%

An electrocardiogram (ECG) is the essential first step in a patient with history and risk factors for coronary artery disease who has symptoms consistent with acute coronary syndrome (ACS). There are a number of well-known atypical presentations (ie, no overt chest pain) of ACS that should prompt a cardiac evaluation. Unfortunately, patients with atypical symptoms are more like to have a delay in diagnosis and under-treatment, which contributes to worsened outcomes. Women, the elderly, and patients with diabetes in particular are more likely to have atypical symptoms; up to 20% of these patients do not report chest pain at the time of presentation. Dyspnea, epigastric pain, and nausea/vomiting are common atypical presentations.

(Choices A, C, 1:1, and E) Abdominal ultrasound is useful for diagnosing acute cholecystitis. Serum amylase and lipase are very helpful tests to support the diagnosis of pancreatitis. Upper gastrointestinal endoscopy should be considered in patients with evidence of acute gastrointestinal blood loss or in the evaluation of possible peptic ulcer disease (PUD)_ An upright abdominal x-ray would be helpful in diagnosing PUD complicated by perforation as air may be seen under the diaphragm. All may need to be considered in this patient but are less immediately life threatening; they should be evaluated after workup for a cardiac etiology has been initiated.

Educational objective: Abdominal pain and nausea/vomiting are atypical presentations of acute coronary syndrome. In select patients, cardiac causes must be excluded prior to pursuing other etiologies.

References:

1 Symptom presentation of acute myocardial infarction: influence of sex, age, and risk factors.

2 Sex differences in symptom presentation in acute myocardial infarction: a systematic review and meta-analysis.

3. Presenting complaint among patients with myocardial infarction who present to an urban, public hospital emergency department.

Copyright @ UWorld Last updated: [10/31/2014]

• 41

• 42 1.1 Feedback, End Block

rNlark Item: 24 of 44 Lab Values. Notes Calculator. Previous Next

1.1 Feedback, End Block

44.

A 12-year-old male is admitted to the hospital with chest pain_ EKG and cardiac enzymes confirm a myocardial infarction and he is started on nitroglycerine. aspirin, simvastatin and low molecular weight heparin. Four days later, he complains of leg pain. Physical examination reveals a cold, pale leg. The angiogram is shown below.

What is the best next step in his management?

1

3

5

7 • a

• 9 • 10

• 11 • 12 • 13 ■ 14 • • 15 ■ 16 •

• 17

• 19 ■ ARM

21 —1

i 24 25 26 _I 27 28 I

▪ 29 • 30

• 32 ■

• 33 ▪ 34-

• 36 • 37

I 39 • 40

41 42

1

5

▪ 7

▪ 9

• 10 • 11 • 12 • 13 • 14 1. • 15 • 16 •

• 17

• 19 ▪ ARM

21

1 6 23 I 24

• 25 26 27

I • 29

• 32 111 • 33 • 34- El

• 36 • 37

• 39 • 40

I

• 30

Item: 24 of 44

r Mark -c21

117 Previous Next

Lab Values. Notes . Calculator .

C_ Cilostazol [3%]

a Leg elevation and warm compress [3%]

• E Embolectomy [13%]

Explanation:

This patient's angiogram is diagnostic of acute popliteal artery occlusion. Contrast can be seen filling the superior portion of the artery before the column of contrast abruptly ends in the popliteal fossa - a pattern suggestive of embolic occlusion. Clinically, ''5 P's" point toward this diagnosis: pain. pulselessness, paresthesia. poikilothermia (coldness). and pallor_ Associated morbidity and mortality are significant, so one must maintain a high index of suspicion_ The patient's recent myocardial infarction makes an embolus from the heart a likely culprit Heparin therapy should be initiated based on a suggestive history and physical exam, without delay for confirmatory testing_ For a limb that is still viable. the next treatment step is either percutaneous thrombolysis or surgical embolectomy. Since this patient has already had an angiogram, intra-arterial thrombolysis or mechanical embolectomy could be attempted. Surgical embolectomy is an option as well_ For a nonviable limb, surgery is mandatory.

(Choice A) Streptokinase is a fibrinolytic agent While intravenous infusion of a fibrinolytic agent is an option for STEM patients, patients with acute limb ischemia typically are treated with direct intra-arterial, not intravenous, administration of a fibrinolytic agent on the clot using an angiographic catheter.

(Choice B) Clopidogrel is an antiplatelet agent that promotes anticoagulation, but does not rapidly break up already existing clots.

(Choice C) Cilostazol inhibits platelet aggregation and causes vasodilation. It is sometimes used to treat chronic claudication, but it has no role in acute limb ischemia.

(Choice 0) Leg elevation and warm compresses would be inadequate therapy for this patient since his limb is likely to soon become nonviable.

Educational objective: The diagnosis of acute limb ischemia should be considered in patients who present with the "five P's'': pain. pulselessness, paresthesias. poikilothermia (coldness), and pallor_ Angiography will typically show an abrupt cutoff of arterial blood flow. IV heparin should be started immediately upon suspicion. The definitive treatment is surgical embolectomy or intra-arterial fibrinolysis/mechanical embolectomy via interventional radiology_

a 1.1

Feedback, End Block

• 1

▪ 3

▪ 5

• 7

• 9 • 10 • 11 • 12 • 13

• •

14 15 16

• 17 • 1E1. • 19 ▪ 2

21

I 24 • 25 • 26 • 27

28

• 29 • 30

• 32 • 33 • 34-

• 36• 37

• 39 • 40 • 41 • 42

Item: 24 of 44 r Mark -c21

Previous

UP Notes

Next Lab Values, Calculator

D_ Leg elevation and warm compress [3%]

• _ E Embolectomy [73%]

al

Explanation:

This patient's angiogram is diagnostic of acute popliteal artery occlusion. Contrast can be seen filling the superior portion of the artery before the column of contrast abruptly ends in the popliteal fossa - a pattern suggestive of embolic occlusion_ Clinically, 'r5 P's" point toward this diagnosis: pain, pulselessness. paresthesia, poikilothermia (coldness), and pallor_ Associated morbidity and mortality are significant, so one must maintain a high index of suspicion_ The patient's recent myocardial infarction makes an embolus from the heart a likely culprit Heparin therapy should be initiated based on a suggestive history and physical exam, without delay for confirmatory testing. For a limb that is still viable, the next treatment step is either percutaneous thrombolysis or surgical embolectomy. Since this patient has already had an angiogram, intra-arterial thrombolysis or mechanical embolectomy could be attempted_ Surgical embolectomy is an option as well. For a nonviable limb. surgery is mandatory_

(Choice A) Streptokinase is a fibrinolytic agent_ While intravenous infusion of a fibrinolytic agent is an option for STEM patients. patients with acute limb ischemia typically are treated with direct intra-arterial. not intravenous, administration of a fibrinolytic agent on the clot using an angiographic catheter.

(Choice B) Clopidogrel is an antiplatelet agent that promotes anticoagulation, but does not rapidly break up already existing clots.

(Choice C) Cilostazol inhibits platelet aggregation and causes vasodilation. It is sometimes used to treat chronic claudication, but it has no role in acute limb ischemia_

(Choice 0) Leg elevation and warm compresses would be inadequate therapy for this patient since his limb is likely to soon become nonviable_

Educational objective: The diagnosis of acute limb ischemia should be considered in patients who present with the "five P's'r-. pain. pulselessness, paresthesias, poikilothermia (coldness). and pallor. Angiography will typically show an abrupt cutoff of arterial blood flow_ IV heparin should be started immediately upon suspicion_ The definitive treatment is surgical embolectomy or intra-arterial fibrinolysis/mechanical embolectomy via interventional radiology.

Copyright (D IJWorld

Last updated: [8/22/2014]

1.1 Feedback, End Block

Autosomal dominant polycystic kidney disease

• Most patients are asymptomatic • Hematuria • Flank pain (nephrolithiasis, infection, cyst rupture,

hemorrhage)

Symptoms

1

I6 3

5

• 7

• 9

' 10

i 11

i 12

• 13

• 14

• 15

• 16 •

• 17

• 19

21

▪ 23

▪ 24

I 25 • 26

• 27

• 29

i 32 ■

• 33

' 34-

• 36

• 37

• 39

' 40

' 30

Item: 25 of 44 Mark 1270

Previous Next

Lab Values. Notes Calculator .

A 3€-year-old man comes to the physician for a routine pre-employment physical_ He has no complaints except for occasional morning headaches_ His father died suddenly at age 54_ The patient's blood pressure is 175/103 mm Hg in the right arm and 180/105 in the left, and pulse is 82 min_ The lungs are clear bilaterally and heart sounds are normal_ Bilateral. nontender. upper abdominal masses are palpated on examination_ His hemoglobin level is 15_2 g/dL and creatinine concentration is 0.8 mgicIL. Which of the following is the most appropriate next step in evaluating this patient's condition?

A_ 24—hour urine cortisol [3%] 41 B. Abdominal ultrasound [57%]

C_ Captopril-enhanced radionuclide renal scan [1%]

D_ Plasma aldosteroneirenin ratio [15%]

E Urine metanephrines [24%]

Explanation:

Clinical signs

• Hypertension • Palpable abdominal masses (usually bilateral) • Proteinuria • Chronic kidney disease

• Cerebral aneurysms • Hepatic & pancreatic cysts

I •

1.1 Feedback,

End Block

1270 Notes Calculator.

Item: 25 of 44 IlrMark Lab Values. Previous Next

1.1 Feedback, End Block

1

3

5

7

• 9

• 10

• 11

• 12 • 13 ▪ 14- • 15 ▪ 16 • 17

• 19 ▪ ARM

21

24

I 25 26

27

2I • 29 • 30

• 32 ■

▪ 33 • 34

• 36

• 38 • 39 • 40

41

42 ."5

44.

Autosornal dominant polycystic kidney disease

Symptoms

• Most patients are asymptomatic • Hematuna • Flank pain (nephrolithiasis, infection, cyst rupture,

hemorrhage)

Clinical signs

• Hypertension * Palpable abdominal masses (usually bilateral) i Proteinuha • Chronic kidney disease

Extrarenal features

• Cerebral aneurysms II Hepatic & pancreatic cysts

• Cardiac valve disorders (mitral valve prolapse, aortic regurgitation)

• Colonic diverticulosis

• Ventral & inguinal hernias

iagnosis ' Ultrasonography (alternate: computed tomography, magnetic resonance imaging) shows multiple renal cysts

Management

• Follow blood pressure & renal function • Aggressive control of cardiovascular risk factors, including

hypertension

• ACE inhibitors preferred for high Wood pressure

• End-stage renal disease: dialysis, renal transplant

0 LP5MLEVIorld, L LC

The nAtipnt' r linir iI nrpinntAtirin with pArlid_nnpt hvnprtpncinn nri hil ter I i inner qhrInminA F iq

End Block 1.1

Feedback,

LISPALEWorkl.LLC

The patient's clinical presentation with early-onset hypertension and bilateral upper abdominal masses is suggestive of polycystic kidney disease (PKD). The autosomal dominant form (ADPKD) is the most common genetic cause of chronic kidney disease. Common clinical features include hypertension, hematuria, proteinuria, palpable renal masses, and progressive renal insufficiency_ Patients may also develop flank pain due to renal calculi, cyst rupture or hemorrhage. or upper urinary tract infections_ Extrarenal complications of ADPKD include cerebral aneurysms. hepatic or pancreatic cysts, cardiac valvular abnormalities (mitral valve prolapse and aoitic regurgitation), colonic diverticula, and ventral and/or inguinal hernias_ The diagnosis of ADPKD is based mainly on abdominal ultrasound (or computed tomography) showing enlarged kidneys with numerous cysts_

Hypertension is a common early finding in patients with ADPKD and is often present before any significant decline in renal function_ Angiotensin-converting-enzyme (ACE) inhibitors are the preferred medications for hypertension associated with ADPKD_ Patients should be monitored closely to assure blood pressure control, with regular follow-up of potassium and renal function markers.

(Choice A) An increased 24-hour urine cortisol excretion is seen in Cushing's syndrome_ This patient has no clinical features to suggest Cushing's syndrome (eg, central obesity, facial plethora, proximal muscle weakness, abdominal striae, ecchymosis), and the potential adrenocortical disorders (eg, adenomas, hyperplasia) causing Cushing's syndrome are rarely palpable_

(Choice C) Captopril radionuclide renal scan is used occasionally to diagnose suspected renovascular disease or renal artery stenosis as a cause of refractory hypertension. These conditions are not associated with palpable renal enlargement

(Choice 0) A high plasma aldosterone/renin ratio is seen in patients with primary aldosteronism. These patients usually present with resistant hypertension or hypertension associated with unexplained hypokalemia_ However, primary aldosteronism is not associated with bilateral renal cysts and/or enlargement.

(Choice E) Urinary excretion of vanillylmandelic acid and metanephrines is increased in patients with pheochromocytoma. These patients generally present with pounding headaches. palpitations. and/or diaphoresis associated with paroxysmal elevation in blood pressure_ However, most pheochromocytomas are too small to be palpable on physical examination_

44.

1

3

5

7

• 9

• 10

▪ 11

▪ 12

• 13

▪ 14 1.

• 15

▪ 16 •

• 17

• 19

▪ ARM

21

24

I 25

26

27

• 29

30

32 ■

• 33

▪ 34- El

▪ 36

• 37

I 39

• 40

41

42

Item: 25 of 44

lirMark 117

Previous Next Lab Values, Notes Calculator .

• ACE inhibitors preferred for high bLood pressure

• End-stage renal disease: dialysis renal transplant

• 1

▪ 3

G 5

7

• 9

• 10

i 11

i 12

• 13

• 14

• 15

• 16

• 17

▪ 1 • 19

• 2

- 21

▪ 23

▪ 24

I 25 • 26

• 27 t 2g

• 29

• 30

i 32

• 33

▪ 34-

• 36

• 37

I 39

• 40

• 41

• 42

Item: 25 of 44

r Mark 117

Previous Next

Lab Values. Notes Calculator.

Hypertension is a common early finding in patients with ADPKD and is often present before any significant decline in renal function_ Angiotensin-converting-enzyme (ACE) inhibitors are the preferred medications for hypertension associated with ADPKD. Patients should be monitored closely to assure blood pressure control. with regular follow-up of potassium and renal function markers_

(Choice A) An increased 24-hour urine cortisol excretion is seen in Cushing's syndrome_ This patient has no clinical features to suggest Cushing's syndrome (eg, central obesity, facial plethora, proximal muscle weakness, abdominal striae, ecchymosis), and the potential adrenocortical disorders (eg, adenomas. hyperplasia) causing Cushing's syndrome are rarely palpable_

(Choice C) Captopril radionuclide renal scan is used occasionally to diagnose suspected renovascular disease or renal artery stenosis as a cause of refractory hypertension_ These conditions are not associated with palpable renal enlargement

(Choice 0) A high plasma aldosterone/renin ratio is seen in patients with primary aldosteronism_ These patients usually present with resistant hypertension or hypertension associated with unexplained hypokalemia_ However, primary aldosteronism is not associated with bilateral renal cysts and/or enlargement

(Choice E) Urinary excretion of vanillylmandelic acid and metanephrines is increased in patients with pheochromocytoma_ These patients generally present with pounding headaches. palpitations, and/or diaphoresis associated with paroxysmal elevation in blood pressure. However. most pheochromocytomas are too small to be palpable on physical examination.

Educational objective: Patients with autosomal dominant polycystic kidney disease (ADPKD) usually present with hypertension, hematuria, proteinuria, palpable renal masses, or progressive renal insufficiency_ They may also have flank pain due to renal calculi, cyst rupture or hemorrhage, or upper urinary tract infections_ Hypertension is a common early finding in patients with ADPKD and usually precedes the decline in renal function_

References:

1. Autosomal dominant polycystic kidney disease.

2. Hypertension in autosomal dominant polycystic kidney disease.

Copyright CI UWorld Last updated: [8/6/2G1 4]

1.1 Feedback, End Block

28

I

1

1 6 3

5

• 7 • • 9

• 10 • 11 • 12 • 13 • 14 • • 15 • 16 •

• 17

• 19

21 22

• 23 • 24

25 26

27

• 29 • 30

31

• 32 ■

• 33 ▪ 34-

• 36 • 37

I 39 • 40

Item: 26 of 44 rNlark

Previous Next Lab Values. Notes Calculator.

A 46-year-old man comes to the physician for a routine health maintenance visit He has no medical problems_ He smokes a pack of cigarettes daily and does not drink alcohol_ His mother has hypertension, and his father died of prostate cancer at age 70. The patient's blood pressure is 128/76 mm Hg and pulse is 8Gimin_ His body mass index is 29 kg/m2. Laboratory results are as follows:

Total cholesterol High-density lipoprotein Low-density lipoprotein Triglycerides Fasting glucose

262 mgfdL 50 mg/dL 162 mg/dL 150 mgfdL 100 mgfdL

His calculated 10-year risk of cardiovascular disease is 9%. Which of the following is the most appropriate next step in management of this patient's dyslipidemia?

A_ Fish oil supplementation [1°A]

E Lifestyle modification only [30%]

C_ Low-dose aspirin [2°A]

D_ Oral niacin therapy [1%]

• E Oral rosuvastatin [6794)]

Explanation:

Guidelines for lipid-lowering therapy

Indication Recommended therapy

Clinically significant atherosclerotic disease • ACS, MI • Stable or unstable angina • Age >75: Moderate-intensity • Coronary or other arteriaC revascularization statin

1.1 Feedback,

End Block

• Age 515: High-intensity statin

I 27

28

EVI*

Notes

Item: 26 of 44 Calculator Lab Values,

rNlark Previous Next

• LIWorld

Guidelines for lipid-lowering therapy

Indication Recommended therapy

Clinically significant atherosclerotic disease • ACS, MI • Stable or unstable angina • Coronary or other arterial revascularization • Stroke, TIA, PAD

• Age T5: High-intensity statin

• Age >15: Moderate-intensity statin

LDL ?190 mg/I:IL High-intensity statin

Age 40-75 with diabetes

• 10-year ASCVD risk ?7.5%: High-intensity statin

• 10-year ASCVD risk <7.5%: Moderate-intensity statin

Estimated 10-year ASCVD risk ?7_5% (Pooled Cohort Equations)

Moderate- to high-intensity statin"

ACS = Acute coronary syndrome; ASCVD = atherosclerotic cardiovascutar disease; MI = myocardial infarction; PAD = peripheral arterial disease; TIA = transient ischemic attack_

a High-intensity statins include atorvastatin 40-80 mg. rosuvastatin 20-40 mg-, moderate-intensity Matins include 2torvastatin 10-20 nog, rosuvastatin 5-10 mg, simvastallin 20-40 nig, pravastatin 40-80 mg, lovastaltin. 40 mg.

This patient has several risk factors for atherosclerotic cardiovascular disease (ASCVD), including smoking. hyperlipidemia, age, and male gender. He should be counseled regarding smoking cessation as well as dietary and exercise changes. His 10-year estimated ASCVD risk of 9% (risk calculator) is high enough to justify statin therapy in addition to lifestyle modification (Choice By Specifically, current guidelines recommend moderate-intensity (eg. atorvastatin 10-20 mg, rosuvastatin 5-1G mg, simvastatin 20-40 mg daily) rit- kink infaric•ihr in An rnri nr rrrc1 ltiractatin 911. An. rrin for •rici A 11 711.

1

3

5

7

• • 9

• 10

• 11

• 12

• 13

▪ 14

• 15

▪ 16

• 17

• 19

▪ ARM

21

22

• 23

24

25

26

• 29

• 30

' 32 ■

• 33

• 34 ' 36.

• 36

• 37

• .39

• 40

41

42 Feedback, End Block

Item: 26 of 44 r Mark

. 4 Notes

Previous Next Lab Values. Calculator

atorvastatin rng, rosuvastatin 5-10 mg, simvaskatin 20-40 mg, pravaStatin 40-80 mg, lovastatin 40 nig.

LIWorld

This patient has several risk factors for atherosclerotic cardiovascular disease (ASCVD). including smoking, hyperlipidemia, age, and male gender. He should be counseled regarding smoking cessation as well as dietary and exercise changes_ His 10-year estimated ASCVD risk of P/0 (risk calculator) is high enough to justify statin therapy in addition to lifestyle modification (Choice B). Specifically, current guidelines recommend moderate-intensity (eg, atorvastatin 10-20 mg, rosuvastatin 5-10 mg, simvastatin 20-40 mg daily) or high-intensity (atorvastatin 40-80 mg or rosuvastatin 20-40 mg daily) statin therapy for patients age 40-75 with an estimated 10-year risk L7.5%. Medication is also recommended for patients with clinical ASCVD, those age 40-75 with diabetes. and those with low-density lipoprotein levels L190 mgict_

(Choice Al High doses of fish oil with omega-3 polyunsaturated fatty acids can effectively reduce serum triglyceride levels_ However, a statin is the first-line therapy in patients with mild-to-moderate hypertriglyceridemia (150-500 mg/dL) for whom the goal of therapy is cardiovascular risk reduction_

(Choice C) Low-dose aspirin therapy (75-100 mg daily) has been shown to reduce the risk of cardiovascular events in patients age L50. However, this benefit is small and is at least partially offset by an increased risk of bleeding (eg. gastrointestinal bleeding, hemorrhagic stroke). In addition, benefits for younger patients are less well established.

(Choice 0) Niacin is effective in raising high-density lipoprotein and has a modest effect in lowering very low-density lipoprotein and low-density lipoprotein cholesterol levels_ However, in clinical trials, niacin has not been found to provide additional benefit in prevention of ASCVD beyond that of statin therapy alone_

Educational objective: Lipid-lowering therapy with statin medications is recommended for primary prevention in patients age 40-15 with a 10-year risk of atherosclerotic cardiovascular disease L7.5%.

References:

1. Comparative benefits of statins in the primary and secondary prevention of major coronary events and all-cause mortality: a network meta-analysis of placebo-controlled and active-comparator trials.

Copyright © Morld Last updated: [10/13/2014]

1.1 Feedback, End Block

▪ 38 ▪ 9

▪ 40

41

42

1

2

3

4

5

▪ 6

▪ 7

▪ 8

▪ 9

I 26 27

28

10

-1r1}1

-13

14

15

16

17

18

19

20

21

22

23

24

25

rNlark Item: 26 of 44 Lab Values. Notes Calculator. Previous Next

Media Exhapit

Pooled Cohort Equation for estimating 10-year risk

Sample screenshot of 10-year ASCVD risk calculator

Yes No

Yes No

Yes No

Gender

Male Female

Race Total Cholesterol (mgldL)

Systolic Blood Pressure

Diabetes

HDL - Cholesterol (rngidL)

Treatment for Hypertension

Smoker

Calculate

White

African American

Other

L

eilLISMLEworld,.LLC

End Block 1.1 Feedback,

• 29

▪ 30

• 32

• 33 4

i 5

• 36

• 30 ■

1

3

• 5 • 6 • 7 • 8 • g • 1^ • 11

• 12 • 13

• 15 • 16 • 17

• 19 • ARM

21 22

• 23 • 24 • 25 • 26 MM. • 28 • 29

• 1 • 32 ■

• 33 • 34

• 36

• 38 • 39 • 40

I

Hemodynamic measurements in shock

Hypovolemic shock

Cartliogenic Septic shock shock

Parameter Normal

Right atrial pressure (preload)

Normal to slight Mean of 4 nin-i Hg I I

Pulmonary capillary wedge

Mean of 9 nin-i Ha Normal to sliciht

Item: 27 of 44 Mark

Previous Next

Lab Values. Notes . Calculator.

44.

A 15:3-year-old woman is admitted to the intensive care unit with hypotension. She is receiving no vasoactive agents and is breathing room air. Blood pressure measured by an intra-arterial line is 72/46 mm Hg. Her pulse is 120/min and regular. Invasive hemodynamic monitoring is established and initial readings are as

i_Ilmonary capillary wedge pressure 6 mm Hg (N: 6-12 mm Hg) Mixed venous oxygen saturation 78% (N: 65%-75%)

of the following is the most likely cause of this patient's condition?

Cardiogenic shock [10°A]

B. Pericardial tamponade [49k]

C_ Right ventricular infarction [12%]

• D_ Septic shock []

E. Volume depletion [29%]

Explanation:

Item: 27 of 44 1>- rNlark Previous Next

End Block 1.1

Feedback,

Hemodynamic measurements in shock

Parameter Normal Hypovolemic shock

Cardiogenic shock

Septic shock

Right atrial pressure (preload)

Mean of 4 nrin-i Hg i '11/4

Normal to slight

Pulmonary capillary wedge

pre ore ss (preload)

Mean of 9 rum Hg i Norm al

I to slight

1

- I Cardiac index

(pump function) 2_8-4.2 Urninim2 1 .1L.

Systemic vascular

resistance (afterload)

Mean of 1150 dynes*sec/c rri5 I I

I

,

Mixed venous oxygen

saturation 60%-80% i .1. 1

uworld.00m

This patient's presentation with low pulmonary capillary wedge pressure (PCWP) and high mixed venous oxygen saturation (Mv02) is most consistent with septic shock. Septic shock is a form of distributive shock rii IP to an i inripirlyinn infinrtinn invcrilvinn thin r Qi..ftinr-n Thin Qi.mtinr-nir

1

3

5

7

• 9

4 10

▪ 11

▪ 12

• 13

▪ 14

• 15

▪ 16

• 17

• 19

▪ ARM 21

22

• 23

24

25

26

27 28

29

4 30

• 32 ■

• 33

4 34 • 36.

• 36

• 37

4 39

4 40

41

42

CP,•

*

Notes Calculator a

Lab Value

• 1

▪ 3

G 5

7

• 9 • 10 ▪ 11 • 12 • 13 • 14 • 15 • 16 • 17 ▪ 1 • 19 • 2 - 21 • 22 • 23 • 24 • 25 • 26 • 27 • 28 • 29 • 30

• 32 • 33 ▪ 34- • '1=.

• 36 • 37 •

I 39 • 40

Item: 27 of 44

-441 Previous

W. 4. Notes

r Mark Next Lab Values. Calculator.

uvrono.uurri

This patient's presentation with low pulmonary capillary wedge pressure (FCWP) and high mixed venous oxygen saturation (Mv02) is most consistent with septic shock. Septic shock is a form of distributive shock

due to an underlying systemic infection involving the circulatory system_ The systemic inflammatory response causes peripheral vasodilatation and decreased systemic vascular resistance (SVR). The decreased blood flow returning to the heart also lowers PCWP. Cardiac output is often increased to compensate and maintain adequate tissue perfusion_ Mv0.2 is high due to hyperdynamic circulation, improper distribution of cardiac

output. and inability of the tissues to adequately extract oxygen_ Hypotension, warm (early) cool (late) extremities. and elevated lactate levels are common clinical findings_

(Choice A) Cardiogenic shock is usually due to significant left ventricular dysfunction and reduced pump function_ There is often low cardiac output. elevated PCWP, and increased SVR. The decreased cardiac output decreases tissue perfusion. which signals tissues to extract more oxygen from the blood and decrease Mv02.

(Choice B) Pericardial tamponade causes increased right atrial and ventricular pressures in addition to characteristic equalization of right atrial, right ventricular end diastolic. and PCW pressures_ A decrease in cardiac output would also cause low mixed venous oxygen saturation.

(Choice C) Patients with right ventricular infarction have reduced blood delivery to the left ventricle, which in turn decreases cardiac output despite normal left ventricular systolic function_ It also leads to increased right atrial andior ventricular pressures along with low or normal PCWP. Mv02 would be expected to be low due to

reduced cardiac output and tissue hypoperfusion.

(Choice E) Hypovolemic shock has reduced preload (PC'...VP) and reduced cardiac output. SVR increases in an attempt to maintain adequate perfusion to the vital organs. IMv0 is low due to reduced tissue perfusion

and increased oxygen extraction by hypoperfused tissue.

Educational objective: The basic underlying pathophysiology in septic shock is the decrease in systemic vascular resistance due to overall peripheral vasodilation_ Hemodynamic monitoring in these patients shows a low pulmonary capillary wedge pressure. low systemic vascular resistance, increased cardiac output. and high mixed venous oxygen saturation.

References:

• 41 42

Item: 27 of 44 r Mark 1111

Previous Next Lab Values. Notes Calculator. puriprlUrcll ildbUUlldLdLIUF1 dflU UUL;rUizIUL.3 bybLurF111; rUbIbLarILU kvrt..). I FIU UUL;rUdSUL3 Ulan.]

flow returning to the heart also lowers PCWE Cardiac output is often increased to compensate and maintain adequate tissue perfusion_ Mv02 is high due to hyperdynamic circulation, improper distribution of cardiac

output, and inability of the tissues to adequately extract oxygen. Hypotension, warm (early) cool (late) extremities, and elevated lactate levels are common clinical findings_

(Choice A) Cardiogenic shock is usually due to significant left ventricular dysfunction and reduced pump function. There is often low cardiac output. elevated PCWP. and increased R. The decreased cardiac output decreases tissue perfusion. which signals tissues to extract more oxygen from the blood and decrease Mv02.

(Choice 6) Pericardial tamponade causes increased right atrial and ventricular pressures in addition to characteristic equalization of right atrial, right ventricular end diastolic. and PON pressures A decrease in cardiac output would also cause low mixed venous oxygen saturation.

(Choice C) Patients with right ventricular infarction have reduced blood delivery to the left ventricle, which in turn decreases cardiac output despite normal left ventricular systolic function_ It also leads to increased right atrial and/or ventricular pressures along with low or normal PCWR Mv02 would be expected to be low due to

reduced cardiac output and tissue hypoperfusion.

(Choice E) Hypovolemic shock has reduced preload (P P) and reduced cardiac output SVR increases in an attempt to maintain adequate perfusion to the vital organs.. Mv0.2 is low due to reduced tissue perfusion and increased oxygen extraction by hypoperfused tissue.

Educational objective: The basic underlying pathophysiology in septic shock is the decrease in systemic vascular resistance due to overall peripheral vasodilation_ Hemodynamic monitoring in these patients shows a low pulmonary capillary wedge pressure, low systemic vascular resistance, increased cardiac output, and high mixed venous oxygen saturation_

References:

1. Systemic hemodynamic abnormalities and vasopressor therapy in sepsis and septic shock.

]

Copyright © UWorld Last updated: [12/19/2014]

1.1 Feedback, End Block

MVO

Notes Item: 28 of 44

Calculator

i 41

Lab Values, rNlark

Previous Next

End Block 1.1 Feedback,

44.

A 34-year-old male who recently immigrated from Brazil presents with a 5 month history of exertional dyspnea without associated chest pain. palpitations, dizziness, or syncope_ His past medical history is significant for an episode of megacolon. which was treated 2 years ago_ On physical examination, there is 1+ pedal edema and mild jugular venous distention. Cardiac exam is significant for the presence of an S3, but no murmurs are heard_ Chest x-ray reveals prominent cardiomegaly_ Based on these findings_ which of the following is mast likely causing his symptoms?

• Diphtheric myocarditis [6°A]

E Coronary artery disease [2°A]

C_ Protozoal disease [80%]

a Giant cell myocarditis [5%]

E Rickettsial myocarditis [6%]

Explanation:

This patient has Chagas disease, a chronic protozoal disease caused by Trypanosoma cruzi_ The organism is common throughout Latin America, and should be considered in a symptomatic patient who has recently immigrated from that area_ The two primary manifestations of Chagas disease are megacolonimegaesophagus and cardiac disease_ Megacolon or megaesophagus (focal GI dilatation) occur secondary to destruction of the nerves controlling the GI smooth muscle_ This patient's pedal edema, jugular venous distention, S3, and cardiomegaly all point toward congestive heart failure_ The pathophysiology of Chagas heart disease is not well understood, but probably represents a prolonged myocarditis secondary to the protozoal infection_

(Choice A) Diphtheria is unusual in present times because of widespread immunizations_ Furthermore, diphtheria myocarditis is exceedingly rare. Upper respiratory tract symptoms would be expected_

(Choice B) Coronary artery disease would be fairly unlikely in this young patient without risk factors for atherosclerosis.

An .1,,voir r PII Arriitiq is A rirF And frpni ipntivftA farm ref mvnr.rrlitiq Thp i inriprIvinri

1

3

5

▪ 7

▪ 9

• 10

• 11

• 12

• 13

• 14

• 15

• 16 •

• 17

• 19

21

22

• 23

• 24

I 25 26

I' 27 I 28

29

• 32 ■

• 33

• 34-

• 36

• 37

• 39

• 40

41

42 V

I

I • 30

Next 1>- CP,*

Notes rNlark

Previous Calculator Lab Values Item: 28 of 44

Copyright (D UWorld Last updated: [8/22/2014]

End Block 1.1 Feedback,

44.

i 11

i 12

• 13

• 14

• 15

• 16 • • 17

• 19

21

22

• 23

• 24

25 26

I' 27 I 28

29

i 32 ■

• 33

' 34-

• 36

• 37

• 39

' 40

41

Explanation:

This patient has Chagas disease, a chronic protozoal disease caused by Trypanosoma cruzi. The organism is common throughout Latin America, and should be considered in a symptomatic patient who has recently immigrated from that area. The two primary manifestations of Chagas disease are megacolon/megaesophagus and cardiac disease_ Megacolon or megaesophagus (focal GI dilatation) occur secondary to destruction of the nerves controlling the GI smooth muscle_ This patient's pedal edema, jugular venous distention, S3, and cardiomegaly all point toward congestive heart failure. The pathophysiology of Chagas heart disease is not well understood. but probably represents a prolonged myocarditis secondary to the protozoal infection_

(Choice A) Diphtheria is unusual in present times because of widespread immunizations_ Furthermore. diphtheria myocarditis is exceedingly rare. Upper respiratory tract symptoms would be expected_

(Choice B) Coronary artery disease would be fairly unlikely in this young patient without risk factors for atherosclerosis.

(Answer D) Giant cell myocarditis is a rare and frequently fatal form of myocarditis. The underlying pathophysiology is not well understood but likely involves autoimmunity_ Patients with giant cell myocarditis tend to present with symptoms that are both more severe and more acute than those affecting the patient described here.

(Choice E) Rickettsial infection can cause myocarditis, but megacolon would not be expected_

Educational objective: Chagas disease is a chronic disease that can cause megaesophagus. megacolon, and/or cardiac dysfunction_ The protozoan Trypanosoma cruzi. endemic to Latin America. is responsible_

1

5

• 7

• 9

' 10

I

11 Coronary artery disease [2%]

C_ Protozoal disease [80%]

D_ Giant cell myocarditis [5%]

E Rickettsial myocarditis [696]

I• 30

r Mark Item: 29 of 44 Lab Values. Notes Calculator. Previous Next

Hemoglobin Platelets Creatinine

14_2 WI:IL 230,000/mm3 1.0 mg/dL

End Block 1.1

Feedback,

1

5

7

9

' 10

i 11

i 12

• 13

14

• 15

16 • • 17

• 19

▪ ARM

21

22

• 23

• 24

• 25

I' 26

• 27

• 28

'

'

30

31 i 32 ■

• 33

' 34-

• 36

• 37

• 39

' 40

41

42

44.

A 29-year-old man comes to the clinic for a follow-up 2 weeks after an emergency department visit for epistaxis requiring anterior nasal packing. In the emergency department, his blood pressure was 170/110 mm Hg. He complains of occasional headaches and fatigue but has no chest pain, palpitations. or syncope_ His past medical history is unremarkable and he does not use tobacco, alcohol. or illicit drugs_ His current blood pressure is 180/112 mm Hg and his pulse is 78/min and regular_ Cardiac auscultation in the supine position shows no murmurs or additional sounds_ Abdominal examination shows no periumbilical bruits_ Electrocardiogram shows normal sinus rhythm, high-voltage QRS complexes, downsloping ST-segment depression. and T-wave inversion in leads V and V6.

Laboratory results are as follows:.

Which of the following is the best next step in evaluation of this patient?

A_ Ambulatory blood pressure monitoring [99t]

• 11 Bilateral arm and leg blood pressure measurements [5z1-%]

C_ Cardiac auscultation in squatting and standing positions [14%]

D_ Carotid sinus massage [7%]

E Exercise stress testing [1.6%]

Explanation:

Late presentation of aortic coarctation

• Asymptomatic hypertension most common

• Chest pain, claudication, headache, epistaxis, heart failure, aortic dissection in severe cases

Presentation

=.11.11 . rNlark Item: 29 of 44

Calculator Lab Values Previous Notes Next

End Block 1.1 Feedback,

Late presentation of aortic coarctation

Presentation • Asymptomatic hypertension most common

• Chest pain, claudication, headache, epistaxis, heart failure, aortic dissection in severe cases

Examination

• Brachial-femoral delay

• tipper-extremity hypertension, lower-extremity hypotension

• Continuous cardiac murmur from large collaterals

Diagnostic studies

• ECG: Left ventricular hypertrophy

• Chest x-ray:

• Notching of the 3rd-8th ribs from enlarged intercostal arteries

• "3 sign" from aortic indentation

• Echocardiography — diagnostic confirmation

Treatment Balloon angioplasty +1- stent

Associated conditions

• Bicuspid aortic valve

• Ventricular septa) defect

• Turner syndrome

44.

OUSWILEINorkl, LLC

The patient has severe symptomatic hypertension with headaches, epistaxis, and evidence of left ventricular hypertrophy on electrocardiogram (ECG). According to the guidelines, all patients (especially young individuals) with systemic hypertension should be evaluated for the presence of coarctation of the aorta with simultaneous palpation of the brachial and femoral pulses to assess for "brachial-femoral' delay. They should

1

3

5

7

• 9

• 10

• 11

• 12

• 13

• 15

▪ 16

• 17

• 19

▪ ARM 21

22

• 23

• 24

• 25

1. 26

27

28

• 30

• 32 1. • 33

▪ 34 El

• 36

• 37

• 39

• 40

41 42

9

4 11

1 12 • 13 • 14 • 15 • 16 • 17

1E1. • 19

22 23 24

• 25 26

• 27 .28

• 30

4 33 34-

• 32

• 36 .311.17

• 39 • 40

41 42

Item: 29 of 44 r Mark

Previous Next Lab Values. Notes Calculator UUNV1Lt4NOTITI, Lii.

The patient has severe symptomatic hypertension with headaches. epistaxis. and evidence of left ventricular hypertrophy on electrocardiogram (ECG)_ According to the guidelines. all patients (especially young individuals) with systemic hypertension should be evaluated for the presence of coarctation of the aorta with simultaneous palpation of the brachial and femoral pulses to assess for "brachial-femoral" delay. They should also have supine bilateral arm (brachial) and prone right and/or left leg (popliteal) blood pressures measured to assess for differential pressures.

Coarctation of the aorta is the narrowing of the descending thoracic aorta, usually just distal to the origin of the left subclavian artery at the site of the ligamentum arteriosum. The condition may be congenital or acquired (eg, Takayasu arteritis).

ECG may show changes of left ventricular hypertrophy, with increased voltage of QRS complexes and ST-and T-wave changes in the left precordial leads_ Chest x-ray in patients with longstanding coarctation can show notching of the 3rd-8th ribs due to erosion by enlarged intercostal arteries. and a classic ''3" sign caused by indentation of aorta at the site of coarctation with pre- and post-stenotic dilatation.

(Choice Al Ambulatory blood pressure monitoring is occasionally used for diagnosis or to assess adequate blood pressure control in patients with hypertension. This patient has episodes of severe symptomatic hypertension along with evidence of end-organ damage (left ventricular hypertrophy on ECG) and does not reed ambulatory blood pressure monitoring.

(Choice C) Cardiac auscultation with squatting and standing positions is a useful physiological maneuver to differentiate between various cardiac murmurs_ However, this patient has no murmur on auscultation_

(Choice 0) Carotid sinus massage is a vagal maneuver that transiently slows sinoatrial nodal activity and atrioventricular node conduction_ It is used to terminate paroxysmal supraventricular tachycardia, which this patient does not have_

(Choice E) Exercise stress testing is a useful screening test to assess for obstructive coronary artery disease_ However, this patient does not have chest pain or dyspnea to suggest coronary artery disease. In addition, the ECG findings are more likely due to left ventricular hypertrophy with strain in this young patient.

Educational objective: Patients with coarctation of the aorta have 'brachial-femoral' delay (due to differential blood pressure in the upper and lower extremities) or hypertension in the upper extremity_

1.1 Feedback, End Block

r

References:

1. Coarctation of the aorta.

Item: 29 of 44 r Mark

Previous Next

Lab Values. Notes . Calculator. SLip111U UlldLUF ai 'GIFU! 15 Urcl(FIrc11) drill prUFIU riga i3FILISUr iei l ley tpupneall Ula.] far US! OF 1F1UdUrUL.1

to assess for differential pressures.

Coaretation of the aorta is the narrowing of the descending thoracic aorta, usually just distal to the origin of the left subclavian artery at the site of the ligamentum arteriosum. The condition may be congenital or acquired (eg, Takayasu arteritis)_

ECG may show changes of left ventricular hypertrophy. with increased voltage of QRS complexes and ST-and T-wave changes in the left precordial leads. Chest x-ray in patients with longstanding coarctation can show notching of the 3rd-8th ribs due to erosion by enlarged intercostal arteries, and a classic "3" sign caused by indentation of aorta at the site of coarctation with pre- and post-stenotic dilatation_

(Choice Al Ambulatory blood pressure monitoring is occasionally used for diagnosis or to assess adequate blood pressure control in patients with hypertension_ This patient has episodes of severe symptomatic hypertension along with evidence of end-organ damage (left ventricular hypertrophy on ECG) and does not need ambulatory blood pressure monitoring_

(Choice C) Cardiac auscultation with squatting and standing positions is a useful physiological maneuver to differentiate between various cardiac murmurs_ However, this patient has no murmur on auscultation.

(Choice 0) Carotid sinus massage is a vagal maneuver that transiently slows sinoatrial nodal activity and atrioventricular node conduction. It is used to terminate paroxysmal supraventricular tachycardia. which this patient does not have_

(Choice E) Exercise stress testing is a useful screening test to assess for obstructive coronary artery disease_ However, this patient does not have chest pain or dyspnea to suggest coronary artery disease_ In addition, the ECG findings are more likely due to left ventricular hypertrophy with strain in this young patient

Educational objective: Patients with coarctation of the aorta have 'brachial-femoral' delay (due to differential blood pressure in the upper and lower extremities) or hypertension in the upper extremity_

Copyright © Mond

Last updated: [121261201.4]

1

1.1 Feedback, End Block

Next

1270 Notes

rNlark Previous Calculator. Lab Values.

Item: 29 of 44

Left common — carotid

Brachiocephalic artery

Left subclavian artery

Aortic coarctation

Patent ductus arteriosus

2 ik• 3 G 4 G 5 ik• 6 ik• 7

8

9

• 10

• 11

• 12

• 13

• 14

• 15

• 16

• 17

• 18

• 19

• 20

• 21

• 22

• 23

• 24

• 25

• 26

• 27

• 28

29

• 30

Coardation of the aorta

Patent ductus Closed ductus

V

Media Exhibit

Aortic coarctation

Ligamentum arteriosum

(ductus closed)

• 32

• 33

• 34 iK•

• 36

• 38

• 39

• 40

End Block

• 41

Feedback, 42

127

Notes Calculator. Item: 30 of 44

Lab Values. rNlark

Previous Next

44.

A 25-year-old woman, gravida 2, pars 1, at 28 weeks gestation is brought to the emergency department by her

boyfriend who found her lying in a pool of blood on the bed_ She is very drowsy but has no pain or uterine

contractions_ She has no other medical conditions and has had an uneventful pregnancy so far. Her only

medication is a prenatal vitamin_ The patient's temperature is 36_7 C (98 F), pulse is 120/min, and

respirations are 181min_ Physical examination shows that the fetus is in a transverse lie_ Inspection of the

perineum shows gross blood and active bleeding per vagina. Which of the following set of parameters would

most likely be seen in this patient?

Cardiac Pulmonary Capillary Systemic Blood Output Wedge Pressure Vascular Resistance Pressure

Explanation:

End Block 1.1 Feedback,

Hemodynamic measurements in shock

Hypovolemic shock

Cardiogenic Septic shock shock

Parameter Normal

Right atrial pressure Mean of 4 mm Hg

Normal to slight 1

1

3

5

▪ 7

▪ 9

' 10

i 11 i 12

• 13

• 14 1.

• 15

• 16

• 17

• 19

• ARM 21 22

• 23

• 24

• 25

• 26

27 I

I. 29

31 ▪ ARM

• 33

' 34 El

' 36

• 37

• .39

' 40

41

• 42

A. Decreased Increased Increased Decreased [8%]

B. Increased Decreased Decreased Decreased [10%]

of C. Decreased Decreased Increased Decreased [75%]

D. Increased Normal Increased Increased [5%]

E. Normal Normal Increased Normal [2%]

I

Calculator Lab Values, Notes Item: 30 of 44 rNlark

Previous Next

End Block 1.1 Feedback,

Hemodynamic measurements in shock

Parameter Normal Hypovolernic shock

Cardiogenic shock

Septic shock

Right atrial pressure (preload)

Mean of 4 nrini Hg i 11/4 Normal to slight

Pulmonary capillary wedge

pressure (preload)

Mean of 9 rum Hg i Florm a I

I to slight

Cardiac index (pump function) 2_8-42 Urninim2

i I • _

Systemic vascular

resistance (afterl oa d)

Mean of 1150 dynes*secicrn5 I I Jr

Mixed venous oxygen

saturation 60%-80% i i 1

44.

uworld.com

This patient with anteparkum hemorrhage arid intravascular volume loss has likely progressed to hypovolemic shock (decreased cerebral perfusion resulting in drowsiness). The loss of intravascular volume decreases left ventricular preload. resulting in decreased cardiac output (CO) and systemic blood pressure (BP). This in

1

3

5

7

• 9 • 10 • 11 • 12

• 13

▪ 14 1. • 15

▪ 16

• 17

• 19

▪ ARM 21 22

• 23

• 24

• 25

▪ 26

27 28 I 29 30 31

• 33 ▪ 34 El

• 36 • 37

• 39 • 40

41 42

Item: 30 of 44 r Mark

Previous

Next

Lab Values. Notes Calculator.

This patient with antepartum hemorrhage and intravascular volume loss has likely progressed to hypovolemic shock (decreased cerebral perfusion resulting in drowsiness). The loss of intravascular volume decreases left ventricular preload. resulting in decreased cardiac output (CO) and systemic blood pressure (BP). This in turn stimulates the sympathetic nervous system, leading to increased heart rate and peripheral vasoconstriction (increased systemic vascular resistance [SVR]) in an attempt to maintain cardiac output and perfusion to vital organs. Pulmonary capillary wedge pressure (PCWP) is a measure of left atrial pressure (and left ventricular end-diastolic pressure): it is decreased due to intravascular volume loss in patients with hypovolemic shock.

(Choice Al This set of readings is consistent with cardiogenic shock due to cardiac pump failure_ Severe heart failure leads to decreased cardiac output and blood pressure_ Heart rate and SVR are increased to compensate for diminished cardiac output and maintain organ perfusion_ PCWP is increased, reflecting an elevated left atrial pressure in patients with cardiogenic shock_

(Choice B) This set of values is suggestive of vasodilatory or distributive shock, which may be seen in a variety of clinical settings, such as sepsis, anaphylaxis, systemic inflammatory response syndrome or central nervous system injury_ In the hyperdynamic phase of septic shock. there is a significant decrease in SVR, with reduced blood pressure and a compensatory rise in heart rate and cardiac output PCWP may be low or normal_ As sepsis progresses to the hypodynamic phase. there may be vasoconstriction with a rise in SVR and a significant decline in CO, usually with a grave deterioration in the patient's overall clinical status.

(Choice 0) These findings are suggestive of a hyperadrenergic state due to catecholamine surge with tachycardia, systemic hypertension. and increased CO and SVR_ PCWP may be normal or elevated_

(Choice E) This set of values is consistent with pre-shock or compensated shock_ as may be seen in patients with mild hypovolemia or early shock from other causes_ There is an increase in heart rate and peripheral vasoconstriction (SVR) to maintain adequate cardiac output and tissue perfusion_ Blood pressure may be mildly low. normal, or occasionally slightly elevated_ However, as the underlying condition worsens further, the increase in heart rate cannot maintain adequate cardiac output and shock ensues_

Educational objective: Hypovolemic shock is characterized by hypotension, tachycardia, decreased cardiac output, increased systemic vascular resistance, and decreased central venous pressure and pulmonary capillary wedge pressure_

References:

1 Hamodynamic menitorina.

1.1 Feedback, End Block

Item: 30 of 44

r Mark 127

Previous Next

Lab Values, Notes Calculator.

(and left ventricular end-diastolic pressure): it is decreased due to intravascular volume loss in patients with hypovolemic shock_

(Choice Al This set of readings is consistent with cardiogenic shock due to cardiac pump failure_ Severe heart failure leads to decreased cardiac output and blood pressure_ Heart rate and SVR are increased to compensate for diminished cardiac output and maintain organ perfusion_ PCWP is increased, reflecting an elevated left atrial pressure in patients with cardiogenic shock_

(Choice B) This set of values is suggestive of vasodilatory or distributive shock, which may be seen in a variety of clinical settings, such as sepsis, anaphylaxis, systemic inflammatory response syndrome or central nervous system injury_ In the hyperdynamic phase of septic shock. there is a significant decrease in SVR, with reduced blood pressure and a compensatory rise in heart rate and cardiac output PCWP may be low or normal_ As sepsis progresses to the hypodynamic phase. there may be vasoconstriction with a rise in SYR and a significant decline in CO, usually with a grave deterioration in the patient's overall clinical status..

(Choice 0) These findings are suggestive of a hyperadrenergic state due to catecholamine surge with tachycardia, systemic hypertension, and increased CO and SVF_ PCWP may be normal or elevated_

(Choice E) This set of values is consistent with pre-shock or compensated shock, as may be seen in patients with mild hypovolemia or early shock from other causes_ There is an increase in heart rate and peripheral vasoconstriction (SVR) to maintain adequate cardiac output and tissue perfusion_ Blood pressure may be mildly low, normal, or occasionally slightly elevated_ However, as the underlying condition worsens further, the increase in heart rate cannot maintain adequate cardiac output and shock ensues_

Educational objective: Hypovolemic shock is characterized by hypotension, tachycardia, decreased cardiac output. increased systemic vascular resistance, and decreased central venous pressure and pulmonary capillary wedge pressure_

References:

1. Hemodynamic monitoring.

2. Effects of fluid resuscitation with colloids vs crystalloids on mortality in critically ill patients presenting with hypovolemic shock: the CRISTAL randomized trial.

Copyright 0 LiWorld Last updated: [1 0/7/201 .E1]

1.1 Feedback, End Block

I

• 9 • 10 • 11 • 12 • 13

• 15 • 16

▪ 14

• 17 ▪ 1E1. • 19

21 1.1r.

21— • 23 • 24 • 25 • 26 • 27

28 29

31

• 33 4 34-. • 36 • 37

4 39 4 40

• 42

Item: 31 of 44 r Mark

Previous Next

Lab Values. Notes Calculator.

44.

A 64-year-old man complains of palpitations and progressive shortness of breath over the past several hours_ He says that he also develops a choking sensation every time he tries to lie down. His medical history is significant for hypertension for the past 20 years and medication non-compliance. He also has a 35 year smoking history_ He reports that his father died of a heart attack at age 10 and his mother suffered from asthma On physical examination. his blood pressure is 170/100 mmHg and his heart rate is 130/min and irregularly irregular_ Lung exam reveals bibasilar crackles_ There is 2+ pitting edema of the lower extremities_ Bedside echocardiography shows a left ventricular ejection fraction of 55%_ Which of the following is most likely responsible for his symptoms?

A_ Cardiogenic shock [5%]

11 Diastolic dysfunction [62%]

C_ High-output heart failure [26%]

D. Small airway bronchoconstriction [3%]

E Increased lung compliance [3%]

Explanation:

This man has heart failure due to diastolic dysfunction, also called heart failure with preserved left ventricular ejection fraction. Diastolic dysfunction refers to impaired filling of the left or right ventricle, either because of impaired myocardial relaxation or a stiff, non-compliant ventricle_ Contractility (i_e_ ejection fraction) may remain normal but diastolic pressures are elevated, consequently reducing cardiac output This leads to typical findings of congestive heart failure_

Systemic hypertension is the classic cause of diastolic dysfunction_ Chronically elevated left ventricular diastolic pressures cause left atrial dilatation, which in turn can lead to atrial fibrillation, as in this patient Treatment is with diuretics and blood pressure control_

(Choice Al Cardiogenic shock usually results from compromised left ventricular function due to an acute myocardial infarction or chronic heart failure. Affected patients show signs of heart failure but would not have the left ventricular function to produce a blood pressure of 170/100. An ejection fraction of 55% would be very unusual for a patient with cardiogenic shock_

1.1 Feedback, End Block

9 10

4 11

1 12 • 13

14 • 15 • 16 • 17 ▪ 1E1. • 19

21 1 .1r.

21— • 23 • 24 • 25 • 26 • 27

28

29

31

• 33 4 34-

• 36 • 37

• 39 • 40

• 42

I

Item: 31 of 44

r Mark -c21

117 Previous Next

Lab Values. Notes . Calculator. I—F 4-0 I I ILA II 1,1 II 11 Li y I 4J1 I 4, I I 4, ha. I I •—• L 1 14r L14J1 1 VJ

E. Increased lung compliance [3%]

Explanation:

This man has heart failure due to diastolic dysfunction, also called heart failure with preserved left ventricular ejection fraction_ Diastolic dysfunction refers to impaired filling of the left or right ventricle, either because of impaired myocardial relaxation or a stiff, non-compliant ventricle_ Contractility (i.e. ejection fraction) may remain normal but diastolic pressures are elevated, consequently reducing cardiac output This leads to typical findings of congestive heart failure

Systemic hypertension is the classic cause of diastolic dysfunction_ Chronically elevated left ventricular diastolic pressures cause left atrial dilatation, which in turn can lead to atrial fibrillation, as in this patient_ Treatment is with diuretics and blood pressure control.

(Choice A) Cardiogenic shock usually results from compromised left ventricular function due to an acute myocardial infarction or chronic heart failure. Affected patients show signs of heart failure but would not have the left ventricular function to produce a blood pressure of 170/100. An ejection fraction of 55% would be very unusual for a patient with cardiogenic shock_

(Choice C) High-output heart failure refers to supranormal ventricular function that still cannot meet the body's metabolic demands. Causes include anemia. hyperthyroidism, beriberi, Paget's disease. and arteriovenous fistulas. The patient above has a normal ejection fraction_

(Choice 0) Asthma is due to small airway bronchoconstriction and will not cause the signs and symptoms of heart failure described here.

(Choice E) Lung compliance increases in chronic obstructive lung disease_ CQPD would not cause this patient's pulmonary edema or signs of right heart failure. Lung compliance is decreased in pulmonary edema.

Educational objective: Diastolic dysfunction refers to impaired ventricular filling due to poor myocardial relaxation or diminished ventricular compliance_ It is usually due to hypertension and is treated with diuretics and antihypertensives_ In severe cases, diastolic dysfunction can cause decompensated heart failure.

Copyright (D IJWorld

Last updated: [8/22/201 4] a 1.1

Feedback, End Block I

1

5

• 7

• 9

' 10 i 11 i 12 • 13 • 14- • 15 • 16 •

• 17

• 19

21 22

• 23 • 24 • 25 • 26

27

I • 29

• 31 32 33 34

' 36 • 37

• 39 ' 40

I

' 30

Item: 32 of 44

127 Notes

r Mark Previous Next Lab Values, Calculator.

44.

A 31-year-old healthy Caucasian male is seen in your office for a routine physical examination. He denies any symptoms or illness_ He says he smokes a pack a day and drinks one to two beers every weekend. He has no allergies. Examination is unremarkable. The EKG reveals normal sinus rhythm with a heart rate of 72: there are frequent premature atrial beats present The blood pressure is 120/65mm Hg. The next step in his management is:

A. Digoxin [11Y0]

B. Lidocaine

C. Order potassium levels [1n]

D_ Complete electrophysiological study 18%]

E Observation [78%]

Explanation:

The diagnosis of atrial premature beats depends upon the recognition of 'P' waves that are premature relative to sinus cycle length and which differ in morphology from sinus 'P' waves_ Atrial premature beats frequently reset the sinus node. producing pauses, which are only partially compensatory_ The QRS width is normal. Premature atrial beats may be completely normal or due to anxiety, CHF, hypoxia, caffeine or electrolyte abnormalities. Premature atrial beats never require any treatment and are completely benign (Option E).

(Option A): Digoxin is a classic inotrope, which is frequently used in the treatment of atrial arrhythmias. especially fibrillation_ It increases the AV nodal refractoriness and thereby slows the ventricular rate in atrial fibrillation and flutter_ Digoxin is particularly used in patients with heart failure (systolic dysfunction) and atrial fibrillationiflutter_ Digoxin has no role in the management of premature atrial beats.

(Option B): Lidocaine is a class 1 anti arrhythmic agent used in the treatment of ventricular arrhythmias. It has no role in the treatment of atrial arrhythmias_ The drug is usually given intravenously.

(Option C): Rarely premature atrial beats may be due to electrolyte abnormalities_ In such cases. levels of potassium, magnesium and calcium may need to be evaluated_ Removing the causative agent or replacing the deficient electrolyte can treat premature atrial beats_ Electrolyte abnormalities are unlikely in a healthy patient without comorbid illnesses.

1.1 Feedback, End Block

1

1 6 3

5

• 7

• 9

' 10

i 11

i 12

• 13

• 14-

• 15

• 16 •

• 17

• 19

21

22

• 23

• 24

• 25

• 26

27

I • 29

• 31

32

33

34

' 36

• 37

• 39

' 40

' 30

Item: 32 of 44

-c21 Previous

127 Notes

Mark Next Lab Values. Calculator.

Digoxin [1%]

Lidocaine [1%]

C_ Order potassium levels [10%]

D_ Complete electrophysiological study [8%]

44.1 E. Observation [78%]

44.

Explanation:

The diagnosis of atrial premature beats depends upon the recognition of 'P' waves that are premature relative to sinus cycle length and which differ in morphology from sinus 'P' waves_ Atrial premature beats frequently reset the sinus node, producing pauses, which are only partially compensatory. The QRS width is normal. Premature atrial beats may be completely normal or due to anxiety, CHF_ hypoxia, caffeine or electrolyte abnormalities_ Premature atrial beats never require any treatment and are completely benign (Option Ey

(Option Al: Digoxin is a classic inotrope, which is frequently used in the treatment of atrial arrhythmias. especially fibrillation. It increases the AV nodal refractoriness and thereby slows the ventricular rate in atrial fibrillation and flutter. Digoxin is particularly used in patients with heart failure (systolic dysfunction) and atrial fibrillation/flutter_ Digoxin has no role in the management of premature atrial beats.

(Option B): Lidocaine is a class 1 anti arrhythmic agent used in the treatment of ventricular arrhythmias. It has no role in the treatment of atrial arrhythmias. The drug is usually given intravenously.

(Option C): Rarely premature atrial beats may be due to electrolyte abnormalities. In such cases, levels of potassium. magnesium and calcium may need to be evaluated. Removing the causative agent or replacing the deficient electrolyte can treat premature atrial beats_ Electrolyte abnormalities are unlikely in a healthy patient without comorbid illnesses.

(Option 0): Electrophysiological study can establish whether an anomalous pathway is present or absent and allow its localization_ Premature atrial beats do not have an anatomical disturbance and EP studies are not helpful at all_

Educational objective: Premature atrial beats are benign and neither require any follow up nor treatment

Copyright CI Morld Last updated: [8/22/2014]

a 1.1

Feedback, End Block

1

1 6 3

5

• 7

• 9

' 10

i 11

i 12

• 13

• 14 • -

• 15

• 16 • • 17

• 19

21

22

• 23

• 24

• 25

• 26

27

I • 29

• 1

32

I 33

' 36

• 37

• 39

' 40

' 30

Item: 33 of 44 Mark -c:=1

CP,*

Previous Next Lab Values Notes Calculator

A 58-year-old woman is evaluated for chest discomfort that occurs while climbing stairs and walking uphill_ She also complains of associated shortness of breath_ She has no known history of coronary artery disease or stroke_ Her past medical history is significant for anxiety disorder, hypertension, hyperlipidemia, type 2 diabetes mellitus. and acid reflux disease_ Her body mass index is 3G kgim2_ Baseline electrocardiogram shows normal sinus rhythm_ Chemistry panel shows normal electrolyte levels and creatinine of 1_1 mgicIL. An exercise (treadmill) stress test is scheduled to diagnose coronary artery disease_ Which of the following daily medications should be held 24-48 hours before the test?

Long-acting nifedipine

A_ Yes Yes No No No [59%]

E. Yes Yes Yes Yes No [1 1%]

C_ Yes No No No No [11%]

D_ No Yes Yes No No [5%]

E No No Yes No Yes [2%]

Explanation:

Metoprolol Ramipril Chlorthalidone Simvastatin

Medications to withhold prior to cardiac stress testing

Beta blockers, calcium channel dockers, nitrates Hold for 48 hours

L

Hold for 48 hours prior to

vasodilator stress test apyridamale

1.1 Feedback,

End Block

rNlark Item: 33 of 44 Lab Values. Notes . Calculator . Previous Next

End Block 1.1

Feedback,

44.

Medications to withhold prior to cardiac stress testing

.

Mold for 48 hours Beta blockers, calcium channel blockers, nitrates

Hold for 48 hours prior to

vasodilator stress test

ti

Dpyridamole

Hold for 12 hours prior to

vasodilator stress test faffeine-contaning food or drinks

Continue

Angiotensin-converting enzyme inhibitorso

anghDtensin receptor blockers, crhgoxin, statins,

diuretics

US.M.LEW.:34d, LLC

Stress testing is an important modality for initial evaluation and diagnosis of patients with symptoms suggesting coronary artery disease (CAD). Coronary angiography is considered the gold standard for diagnosis, but it is invasive and expensive_ Therefore, it is not a suitable initial diagnostic test for those with low-to-intermediate pretest probability of CAD_ Stress testing (exercise or pharmacologic stress) can detect underlying reversible ischemia or prior myocardial infarction through electrocardiogram changes, perfusion defects. or wall-motion abnormalities.

A detailed medication and dietary history should be obtained prior to stress testing as a variety of medications and dietary supplements can interfere with its accuracy (Table).. Beta blockers, calcium channel blockers, and nitrates are antianginal agents that reduce the extent and severity of ischemia during exercise stress testing_ These medications should be withheld for at least 48 hours prior to stress testing (Choice CY However, they should be continued in patients with known CAD undergoing stress testing to assess the efficacy of antianginal therapy_ The decision to withhold medications should also be carefully tailored to the

1

3

5

7

• 9

• 10

• 11

• 12

• 13

▪ 14-

• 15

▪ 16 •

• 17

• 19

▪ ARM

21

22

• 23

• 24

• 25

▪ 26

27

28 I • 29

• 30

32

I 33

34

• 36

• 37

I 39

• 40

41

42

ail

1

1 6 3

5

▪ 7

▪ 9

• 10 • 11 • 12 • 13 • 14- 1. • 15 • 16 •

• 17

• 19 ▪ ARM

21 22

• 23 • 24 • 25 • 26

27

I • 29

• 1 32

I 33

• 36 • 37

• 39 • 40

• 30

Item: 33 of 44

—441

Previous

r Mark Next Lab Values. Notes . Calculator .

Continue

Angiotensin-converting enzyme inhibitors, angiotensin receptor blockers, digoxin, statins, diuretics

kti uSIALEWarld, LLC

Stress testing is an important modality for initial evaluation and diagnosis of patients with symptoms suggesting coronary artery disease (CAD). Coronary angiography is considered the gold standard for diagnosis, but it is invasive and expensive_ Therefore, it is not a suitable initial diagnostic test for those with low-to-intermediate pretest probability of CAD. Stress testing (exercise or pharmacologic stress) can detect underlying reversible ischemia or prior myocardial infarction through electrocardiogram changes. perfusion defects, or wall-motion abnormalities_

A detailed medication and dietary history should be obtained prior to stress testing as a variety of medications and dietary supplements can interfere with its accuracy (Table). Beta blockers, calcium channel blockers. and nitrates are antianginal agents that reduce the extent and severity of ischemia during exercise stress testing_ These medications should be withheld for at least 48 hours prior to stress testing (Choice Cy However, they should be continued in patients with known CAD undergoing stress testing to assess the efficacy of antianginal therapy_ The decision to withhold medications should also be carefully tailored to the individual to minimize the risk of rebound hypertension or arrhythmias_

(Choice B) Diuretics do not directly affect the accuracy of stress testing_ However_ diuretic- induced hypokalemia (potassium <3 mEq.1) can cause ST-segment depression and false- positive results on exercise stress testing_ Testing should be delayed until potassium has been repleted in those patients_

(Choices 0 and E) Angiotensin-converting enzyme inhibitors,. angiotensin receptor blockers, and statins do not affect the diagnostic accuracy of stress testing for CAD_ These medications should not be withheld prior to stress testing_

Educational objective: Beta blockers, calcium channel blockers. and nitrates are antianginal agents that should be withheld for at least 48 hours prior to cardiac stress testing_ However, these medications should be continued in patients with known coronary artery disease undergoing stress testing to assess the efficacy of antianginal therapy_

Copyright © Morld Last updated: [12/20/2014]

1.1 Feedback, End Block

• 4

• 1 2

3

• 5

• 6

• 7

• 10

• 11

• 12

▪ 13

▪ 14-

▪ 15

▪ 16

▪ 17

▪ 18

▪ 19

▪ 20

▪ 21

▪ 22

▪ 23

▪ 24

▪ 25

▪ 26

▪ 27

▪ 28

• 29

• 30

▪ 32

I 33

• 4

• 36

• 38

• 39

• 40

rNlark Item: 33 of 44 Next Previous Lab Values. Notes Calculator .

Media Exhibit

Pretest probability of coronary artery disease

Pretest probability of coronary artery disease

Low

(<10%)

• Asymptomatic people of all ages

• Atypical chest pain in women age <50

tntermediate

(20% -80%)

• Atypical angina in men of all ages

. Atypical angina in women age a-50

. Typical angina in women age 30-50

High

(>90%)

. Typical angina in men age X40

- Typical angina in women age X60

1.115MLE11orld.

1

5

• 7

• 9

• 10

• 11

• 12

• 13

• 14-

• 15

• 16 •

• 17

• 19

21

22

• 23

• 24

• 25

• 26

27

I • 29

1

32

33

24

• 36

• 37

I 39

• 40

I

• 30

Item: 34 of 44

117 Notes

r Mark Previous Next Lab Values. Calculator .

44.

A 34-year-old man rushes into the ER complaining of severe substernal chest pain that began abruptly 30 minutes ago. He says that he also feels as though his heart 'is racing,' but denies any shortness of breath. cough or fever. He has never experienced pain like this before. His past medical history is significant for an appendectomy one year ago_ The patient reports that his father died at age 64 due to "some heart problem" and his mother died of ovarian cancer_ On physical examination, the patient is agitated and sweating profusely_ His pulse is 110 min. blood pressure is160/100 mm Hg, and respirations are 14/min. Physical examination is normal except for dilated pupils and a small amount of blood at the external nares. EKG shows ST elevations in leadsV1_v4. liAlhat is the most likely explanation for his symptoms?

A_ Atherosclerotic vascular disease [69/0]

B_ Acute pericarditis [3%]

C. Pleurodynia [1%]

D_ Drug-induced vasospasm [84%]

E_ Aortic dissection [4'36]

F. Pulmonary infarction [116]

Explanation:

This patient's dilated pupils and blood-crusted nose suggest cocaine abuse_ Cocaine inhibits catecholamine reuptake from neuronal synapses, and is therefore sympathomimetic_ In some, cocaine-induced vasospasm can be so severe as to cause myocardial ischemia and/or infarction, which, given this patient's chest pain and ST elevations. is likely the case here_ Patients may not always be forthcoming about drug use, so it is important to maintain a high index of suspicion in the right clinical setting_ Cocaine-induced ST elevation myocardial infarctions (STEMIs) are treated the same as classic STEMIs, with PICA or thrombolysis. Aspirin and nitrates are also appropriate. but beta-blockers should be avoided. Beta-blocker therapy administered to cocaine-using individuals allows unopposed alpha agonist activity that can worsen vasospasm_ Calcium channel blockers and alpha blockers like phentolamine can also help reduce vasospasm in these patients_

1.1 Feedback, End Block

1>- Next

Mark Notes

-c21 Previous Calculator Lab Values.

Item: 34 of 44

Copyright CI UWorld Last updated: [8/22/2014] a

End Block 1.1 Feedback,

Drug-induced vasospasm [84%]

E Aortic dissection [4%]

• Pulmonary infarction [1%]

Explanation:

This patient's dilated pupils and blood-crusted nose suggest cocaine abuse_ Cocaine inhibits catecholamine reuptake from neuronal synapses, and is therefore sympathomimetic_ In some, cocaine-induced vasospasm can be so severe as to cause myocardial ischemia and/or infarction, which, given this patient's chest pain and ST elevations. is likely the case here. Patients may not always be forthcoming about drug use, so it is important to maintain a high index of suspicion in the right clinical setting. Cocaine-induced ST elevation myocardial infarctions (STErylls) are treated the same as classic STErylls, with PICA or thrombolysis. Aspirin and nitrates are also appropriate, but beta-blockers should be avoided_ Beta-blocker therapy administered to cocaine-using individuals allows unopposed alpha agonist activity that can worsen vasospasm_ Calcium channel blockers and alpha blockers like phentolamine can also help reduce vasospasm in these patients.

(Choice A) Atherosclerotic heart disease would be rare in a patient this young without documented additional risk factors_

(Choice 6) Acute pericarditis can cause ST elevations on EKG, which are often accompanied by PR depressions. Pericarditis is not a common complication of cocaine use.

(Choice C) Pleurodynia is chest pain typically of pulmonary etiology that is worse with deep breathing_ While cocaine can cause pleuritic chest pain through a variety of different mechanisms. ST elevations would not be expected on EKG_

(Choice E) While cocaine can cause aortic dissection, this patient's ST elevations are more suggestive of myocardial infarction_

Educational objective: Pupillary dilation and blood at the external nares in a patient with chest pain can be a clue to cocaine-induced vasospasm_ ST elevation myocardial infarctions can occur as a consequence of cocaine use.

1

5

• 7

• 9

' 10

i 11

i 12

• 13

• 14-

• 15

• 16 •

• 17

• 19

21

22

• 23

• 24

• 25

• 26

27

I • 29

• 1

32

33

24

' 36

• 37

• 39

' 40

I

' 30

44.

-441

Previous Item: 35 of 44

Calculator Lab Values. 1>- Next

r Mark Notes

It

(Choice B) Digoxin inhibits the ATPase-dependent sodium-potassium pump and increases intracellular qnrliiim Thic rprliirpq qnriiiim_rAlriiim pvrhAnripr rpqiiltinn in n inrrinqp in intmrplliilr-

End Block 1.1

Feedback,

44.

A 42-year-old man comes to the physician for recurrent palpitations. After initial evaluation. he is diagnosed with paroxysmal atrial fibrillation_ Echocardiogram shows normal left ventricular function and no valvular disease_ He is initiated on medication therapy. Two weeks later, the patient undergoes a treadmill exercise test During the test, his heart rate increases from 75/min to 1€ /min and the QRS complex duration increases from 109 seconds to 1113 seconds_ Which of the following medications is most likely responsible for the observed findings?

Explanation:

Class I antiarrhythmic drugs act by blocking sodium channels and inhibiting the initial depolarization phase (phase 0) of the action potential_ Flecainide and propafenone are class IC antiarrhythmic drugs that are occasionally used in treatment for atrial fibrillation (for maintenance of sinus rhythm) in patients with structurally normal hearts. They have the slowest rate of drug binding and dissociation from the sodium channel receptor_ Under normal circumstances, flecainide does not cause any significant prolongation of the QRS or QT interval_

In patients with faster heart rates. the drug has less time to dissociate from the sodium channels, leading to a higher number of blocked channels, which in turn leads to a progressive decrease in impulse conduction and a widening of the ORS complex. This phenomenon is termed "use dependence," and is seen most frequently with class IC antiarrhythmic agents (less with class IA drugs and rarely with class IB drugs), and is the mechanism behind their efficacy against supraventricular arrhvthmias.

(Choice A) Amlodipine is a dihydropyridine calcium channel blacker (CCB) and does not exhibit use dependence or prolong QRS duration.

1

1 6 3

5

▪ 7

▪ 9

• 10

• 11

• 12

• 13

▪ 14 • -

• 15

▪ 16 • • 17

• 19

▪ ARM

21

22

• 23

• 24

• 25

▪ 26

27

I • 29

• 30

• 32

• 33

• 34

35 • 36

• 37

• 38

• 39

• 40

41

42

Amlodipine [111°A]

B_ Digoxin [24%]

Flecainide [39%]

• Metoprolol [10%]

E Verapamil [16%]

1

1 6 3

5

▪ 7

▪ 9

• 10

• 11

• 12

• 13

▪ 14- 1.

• 15

▪ 16 •

• 17

• 19

21

22

• 23

• 24

• 25

▪ 26

27

I • 29

• 1

• 32 111

• 33

• 34

35 • 36

• 37

• 38

• 39

• 40

• 30

Item: 35 of 44 Mark Previous Next

Lab Values. Notes Calculator

44. Class I antiarrhythmic drugs act by blocking sodium channels and inhibiting the initial depolarization phase (phase 0) of the action potential_ Flecainide and propafenone are class IC antiarrhythmic drugs that are occasionally used in treatment for atrial fibrillation (for maintenance of sinus rhythm) in patients with structurally normal hearts_ They have the slowest rate of drug binding and dissociation from the sodium channel receptor_ Under normal circumstances, flecainide does not cause any significant prolongation of the ORS or QT interval_

In patients with faster heart rates. the drug has less time to dissociate from the sodium channels, leading to a higher number of blocked channels. which in turn leads to a progressive decrease in impulse conduction and a widening of the QRS complex_ This phenomenon is termed "use dependence," and is seen most frequently with class IC antiarrhythmic agents (less with class IA drugs and rarely with class IB drugs), and is the mechanism behind their efficacy against supraventricular arrhythmias_

(Choice A) Amlodipine is a dihydropyridine calcium channel blocker (CCB) and does not exhibit use dependence or prolong ORS duration_

(Choice B) Digoxin inhibits the ATPase-dependent sodium-potassium pump and increases intracellular sodium_ This reduces sodium-calcium exchanger activity, resulting in an increase in intracellular calcium_ It also enhances vagal tone and slows conduction through the atrioventricular (AV) node_ These effects can cause bradyarrhythmias (younger, healthier patients) or enhance automaticity and delayed after-depolarizations. leading to ventricular ectopy and tachyarrhythmias (more common in older patients with underlying cardiac disease)_

(Choice 0) l'yletoprolol is a beta blocker and inhibits sympathetic activity, leading to a decrease in the rate of impulse generation and an increase in the refractory period of the AV node_ It does not prolong the ORS complex duration_

(Choice E) Calcium channel blockers (GCBs) with antiarrhythmic properties (verapamil and diltiazem) also display use dependence, with an increase in calcium channel blockade with increasing ventricular activation_ They cause a prolongation of the refractory period of the AV node, leading to an increased PR interval_ There is no change in the ORS complex duration.

Educational objective: Use dependence refers to enhanced pharmacologic effects of a drug during faster heart rates and is seen with class I (especially IC) and class IV (calcium channel blockers) antiarrhythmic agents_ Class IC agents cause a progressive decrease in impulse conduction with faster heart rates, leading to an increase in the ORS complex duration.

1.1 Feedback, End Block

Item: 35 of 44

r Mark Previous Next

Lab Values. Notes Calculator

and and a widening of the ORS complex_ This phenomenon is termed "use dependence," and is seen most frequently with class IC antiarrhythmic agents (less with class IA drugs and rarely with class IB drugs), and is the mechanism behind their efficacy against supraventricular arrhythmias.

(Choice A) Amlodipine is a dihydropyridine calcium channel blocker (COB) and does not exhibit use dependence or prolong ORS duration.

(Choice B) Digoxin inhibits the ATPase-dependent sodium-potassium pump and increases intracellular sodium_ This reduces sodium-calcium exchanger activity, resulting in an increase in intracellular calcium_ It also enhances vagal tone and slows conduction through the atrioventricular (AV) node_ These effects can cause bradyarrhythmias (younger, healthier patients) or enhance automaticity and delayed after-depolarizations, leading to ventricular ectopy and tachyarrhythmias (more common in older patients with underlying cardiac disease)_

(Choice ID) Metoprolol is a beta blocker and inhibits sympathetic activity, leading to a decrease in the rate of impulse generation and an increase in the refractory period of the AV node_ It does not prolong the ORS complex duration.

(Choice E) Calcium channel blockers (CCEs) with antiarrhy-thmic properties (verapamil and diltiazem) also display use dependence, with an increase in calcium channel blockade with increasing ventricular activation_ They cause a prolongation of the refractory period of the AV node, leading to an increased PR interval_ There is no change in the QRS complex duration_

Educational objective: Use dependence refers to enhanced pharmacologic effects of a drug during faster heart rates and is seen with class I (especially IC) and class IV (calcium channel blockers) antiarrhvthmic agents_ Class IC agents cause a progressive decrease in impulse conduction with faster heart rates. leading to an increase in the ORS complex duration_

References:

1. Old and new antiarrhythmic drugs for converting and maintaining sinus rhythm in atrial fibrillation: comparative efficacy and results of trials.

2. Flecainide associated torsade de painter: A potential case of reverse use dependence.

Copyright @ Morld

Last updated: [11/11/2014]

a 1.1

Feedback, End Block

• 9

4 11

1 12 . 13 . 14 . 15: . 16 . 17 ▪ 1 E . 19

21!Ir.

22 2 . 23 . 24 . 25 . 261 • 27

• 2B . 29 • 30

1 32 . 33 . 34 • 35 • 36

37 38 39 40

42

(Choices A and El Pulseless electrical activity is defined as the presence of electrocardiographic rhythm in

1.1 Feedback, End Block

Item: 36 of 44 r Mark

Previous Next

Lab Values. Notes . Calculator.

44.

A 41-year-old man develops sudden onset of mid-sternal chest pain and diaphoresis during a meeting in his office. While waiting for emergency medical personnel to arrive, he complains of dizziness and becomes unresponsive. His coworkers perform cardiopulmonary resuscitation and he regains consciousness after 60 seconds_ The patient has a history of diet-controlled type 2 diabetes mellitus, hypertension. and hyperlipidemia_ In the emergency department, his blood pressure is 142/88 mm Hg and pulse is 92/min. Electrocardiogram shows normal sinus rhythm, ventricular premature complexes, and a 3-mm ST-segment elevation in leads V1-V3_ What is the most likely primary pathophysiologic mechanism responsible for this patient's syncopal episode?

A_ Asystole [12%]

13_ Atrial fibrillation [Flo]

C_ Atrio-ventricular conduction block [19%]

Paroxysmal supraventricular tachycardia [10%]

• E Pulseless electrical activity [1€%]

: • F. Reentrant ventricular arrhythmias [319q

Explanation:

Ventricular arrhythmias. including ventricular premature beats. nonsustained or sustained ventricular tachycardia, and ventricular fibrillation, are quite common in the immediate post-myocardial infarction (Ml) period_ Ventricular fibrillation is the most frequent underlying arrhythmia responsible for sudden cardiac arrest in the setting of acute MI: more than 50% occur within the first hour of symptom onset Reentry is the predominant mechanism responsible for ventricular arrhythmias in the immediate post-infarction period_

The underlying mechanism responsible for periinfarction ventricular arrhythmias varies according to the time elapsed since the onset of MI. Arrhythmias occurring within 10 minutes of coronary occlusion are known as "immediate" or phase la ventricular arrhythmias_ Acute ischemia causes heterogeneity of conduction with areas of marked conduction slowing and delayed activation, which in turn predisposes to reentrant arrhythmias_ On the contrary, "delayed" or phase lb arrhythmias occur about 10-6G minutes after acute infarction and are thought to result from abnormal automaticity_

G 5

WO' Notes

Item: 36 of 44 Calculator Lab Values.

r Mark Previous Next

End Block 1.1

Feedback,

Ventricular arrhythmias, including ventricular premature beats. nonsustained or sustained ventricular tachycardia, and ventricular fibrillation, are quite common in the immediate post-myocardial infarction (MI) period_ Ventricular fibrillation is the most frequent underlying arrhythmia responsible for sudden cardiac arrest in the setting of acute Ml: more than 5G% occur within the first hour of symptom onset Reentry is the predominant mechanism responsible for ventricular arrhythmias in the immediate post-infarction period_

The underlying mechanism responsible for periinfarction ventricular arrhythmias varies according to the time elapsed since the onset of ML Arrhythmias occurring within 10 minutes of coronary occlusion are known as "immediate" or phase la ventricular arrhythmias. Acute ischemia causes heterogeneity of conduction with areas of marked conduction slowing and delayed activation. which in turn predisposes to reentrant arrhythmias_ On the contrary, "delayed" or phase lb arrhythmias occur about 1G-6G minutes after acute infarction and are thought to result from abnormal automaticity.

(Choices A and El Pulseless electrical activity is defined as the presence of electrocardiographic rhythm in the absence of adequate cardiac mechanical contraction to generate a palpable pulse_ Asystole is the complete absence of electrical and mechanical cardiac activity_ Patients with prolonged duration of cardiac arrest can degenerate into pulseless electrical activity or asystole, which usually signifies the presence of severe and irreversible myocardial damage.

(Choice Cl Atrio-ventricular conduction block can occur in patients with acute Ml, especially those with inferior wall ML It is usually transient and resolves after successful reperfusion with thrombolytics or primary percutaneous coronary intervention_

(Choices B and Ell Supraventricular tachycardiac other than atrial fibrillation or flutter are uncommon in the periinfarction period_ Even atrial fibrillation occurs rarely (about 3%) during the first 3 hours of acute Ml; if it does occur, it rarely presents with syncope_

Educational objective: Reentrant ventricular arrhythmias (eg, ventricular fibrillation) are the most common cause of sudden cardiac arrest in the immediate post-infarction period in patients with acute myocardial infarction_

• 1

▪ 3

7

• 9

• 10

▪ 11

▪ 12

• 13

• 14

• 15

• 16

• 17

• 19

▪ 2

- 21

▪ 22

• 23

• 24

• 25

▪ 26

• 27

• 28 I • 29

• 30

▪ 32

• 33

• 34

• 35

I 36

• 37

▪ 38

I 39

• 40

• 41

• 42

References:

1. Sustained ventricular arrhythmias in patients receiving thrombolytic therapy: incidence and outcomes. The GUSTO investigators.

117 Notes . Calculator .

Item: 36 of 44 Lab Values.

r Mark Previous Next

Copyright © liWorld Last updated: [12/1/2014] a

End Block 1.1

Feedback,

5 G

7

• 9 • 10

▪ 11 ▪ 12 • 13 • 14

• 1

▪ 3

• 15 • 16 • 17 ▪ 1 • 19 ▪ 2 - 21 ▪ 22 • 23 • 24 • 25 ▪ 26 • 27

• 28 I • 29 • 30

▪ 32 • 33 • 34 • 35 I 36

• 37 • 38 I 39 • 40 • 41 • 42

tachycardia. and ventricular fibrillation, are quite common in the immediate post-myocardial infarction (Ml) period. Ventricular fibrillation is the most frequent underlying arrhythmia responsible for sudden cardiac arrest in the setting of acute Ml; more than 50% occur within the first hour of symptom onset Reentry is the predominant mechanism responsible for ventricular arrhythmias in the immediate post-infarction period.

The underlying mechanism responsible for periinfarction ventricular arrhythmias varies according to the time elapsed since the onset of ML Arrhythmias occurring within 10 minutes of coronary occlusion are known as "immediate" or phase la ventricular arrhythmias_ Acute ischemia causes heterogeneity of conduction with areas of marked conduction slowing and delayed activation, which in turn predisposes to reentrant arrhythmias. On the contrary, "delayed" or phase lb arrhythmias occur about 10-6G minutes after acute infarction and are thought to result from abnormal automaticity_

(Choices A and E) Pulseless electrical activity is defined as the presence of electrocardiographic rhythm in the absence of adequate cardiac mechanical contraction to generate a palpable pulse_ Asystole is the complete absence of electrical and mechanical cardiac activity_ Patients with prolonged duration of cardiac arrest can degenerate into pulseless electrical activity or asystole, which usually signifies the presence of severe and irreversible myocardial damage_

(Choice C) Atrio-ventricular conduction block can occur in patients with acute Ml, especially those with inferior wall ML It is usually transient and resolves after successful reperfusion with thrombolytics or primary percutaneous coronary intervention_

(Choices B and D) Supraventricular tachycardias other than atrial fibrillation or flutter are uncommon in the periinfarction period_ Even atrial fibrillation occurs rarely (about 3%) during the first 3 hours of acute Ml; if it does occur. it rarely presents with syncope_

Educational objective: Reentrant ventricular arrhythmias (eg, ventricular fibrillation) are the most common cause of sudden cardiac arrest in the immediate post-infarction period in patients with acute myocardial infarction_

r

References:

1. Sustained ventricular arrhythmias in patients receiving thrombolytic therapy: incidence and outcomes. The GUSTO investigators.

1

1 6 3

5

▪ 7

▪ 9

• 10 • 11 • 12 • 13 • 14 1. • 15 • 16 •

• 17

• 19 ▪ ARM

21 22

• 23 • 24 • 25 • 26

27

I • 29

• 1 • 32 111 • 33 • 34- El

• 36 37

• 38 • 39 • 40

• 30

Item: 37 of 44 r Mark Previous Next

SI Lab Values!

OPio

Notes Calculator

A 64-year-old nondiabetic man complains of cramping pain in his right thigh after walking 2 blocks. The pain goes away once he stops and rests for several minutes_ He is an ex-smoker with 35-pack-year history. On physical examination, the distal pulses are diminished on the right leg. His ankle brachial index is 0.98 on the left and 075 on the right Which of the following is this patient most likely to suffer over the next 5 years?

A. Abdominal aortic aneurysm rupture [7%]

B. Above-knee amputation [7c,vo]

C. Below-knee amputation [22%]

0_ Intracranial hemorrhage [1°/0]

E Myocardial infarction [63%]

Explanation:

The patient's clinical presentation, with symptoms of intermittent claudication, diminished pulses, and abnormal (<1) ankle brachial index, is suggestive of peripheral artery disease (PAD) involving the right lower extremity-. Multiple studies assessing the natural history of patients with PAID have found intermittent claudication as a strong predictor of the future risk of cardiovascular morbidity and mortality_ Patients with PAD and intermittent claudication have an estimated 20% 5-year risk of nonfatal myocardial infarction and stroke and a 15%-30% 5-year risk of death due to cardiovascular causes. The risk rises exponentially with progression of PAID, with an estimated 25% 1-year risk of cardiovascular mortality in patients with critical limb ischemia. The presence of PAID is a coronary artery disease risk equivalent, and these patients should be managed with aggressive risk factor modification for prevention of cardiovascular morbidity and mortality_

(Choice A) Abdominal aortic aneurysm (W) can be detected in approximately 10% of patients with PAD_ The risk of AAA rupture is increased in patients with large aneurysm diameter (>5.5 cm) aortic expansion rate >15 cm/year. female gender, current ongoing smoking, and hypertension. This patient has no symptoms or signs suggestive of AAA and therefore is not at increased risk of rupture.

(Choices B and C) The majority of patients (70%-80%) with intermittent claudication continue to have stable claudication symptoms at 5 years. Approximately 10%-20% of patients progress to worsening claudication, and only 1%-2% progress to develop critical limb ischemia with rest pain. nonhealing ulcer, and tissue gangrene that may require limb amputation_

1.1 Feedback, End Block

G 5

-cZ1

Previous

Item: 37 of 44 Calculator Lab Values,

r Mark Notes Next

Copyright @ Mond Last updated: [12/30/2014] a

End Block 1.1

Feedback,

ne patient's clinical presentation, witri symptoms or intermittent claudication, diminisried pulses, and abnormal (<1) ankle brachial index, is suggestive of peripheral artery disease (PAD) involving the right lower extremity. Multiple studies assessing the natural history of patients with PAD have found intermittent claudication as a strong predictor of the future risk of cardiovascular morbidity and mortality_ Patients with PAD and intermittent claudication have an estimated 20% 5-year risk of nonfatal myocardial infarction and stroke and a 15%-30% 5-year risk of death due to cardiovascular causes_ The risk rises exponentially with progression of PAD, with an estimated 25% 1-year risk of cardiovascular mortality in patients with critical limb ischemia. The presence of PAD is a coronary artery disease risk equivalent. and these patients should be managed with aggressive risk factor modification for prevention of cardiovascular morbidity and mortality.

(Choice A) Abdominal aortic aneurysm (AAA) can be detected in approximately 10% of patients with PAD. The risk of AM rupture is increased in patients with large aneurysm diameter (>5_5 cm), aortic expansion rate >115 cmiyear, female gender, current ongoing smoking, and hypertension_ This patient has no symptoms or signs suggestive of AAA and therefore is not at increased risk of rupture.

(Choices B and C) The majority of patients (70%-80%) with intermittent claudication continue to have stable claudication symptoms at 5 years_ Approximately 10%-20% of patients progress to worsening claudication, and only 1%-2% progress to develop critical limb ischemia with rest pain, nonhealing ulcer, and tissue gangrene that may require limb amputation_

(Choice 0) The presence of PAD is not associated with an increased risk of intracranial hemorrhage_

Educational objective: Cardiovascular disease is the major cause of morbidity and mortality in patients with peripheral artery disease (PAD). Patients with PAD and intermittent claudication have an estimated 20% 5-year risk of nonfatal myocardial infarction and stroke and a 15%-30% risk of death due to cardiovascular causes_ Only 1%-2% of patients with PAD progress to develop critical limb ischemia with risk of limb amputation_

• 1

▪ 3

7

• 9

• 10

▪ 11

▪ 12

• 13

• 14

• 15

• 16

• 17

▪ 1 • 19

• 2

- 21

▪ 22

• 23

• 24

• 25

▪ 26

• 27

• 28 I • 29

• 30

▪ 32

• 33

▪ 34

▪ 36

37 • 38

▪ 39

• 40

• 41

42

References:

1. Mortality and cardiovascular risk across the ankle-arm index spectrum: results from the Cardiovascular Health Study.

2. ACCIAHA 2006 Practice guidelines for the management of patients with peripheral arterial disease.

1

5

• 7

• 9

• 10

• 11

• 12

• 13

▪ 14 1.

• 15

▪ 16 •

• 17

• 19

▪ ARM

21

22

• 23

• 24

• 25

▪ 26

27

I • 29

31

• 32 111

• 33

▪ 34- El

• 36

I 37 0 0

u

39

40

I

• 30

Item: 38 of 44

r Mark CP,*

Previous Next

Lab Values. Notes Calculator

A 37-year-old Cambodian woman presents to the emergency room with acute onset of left-sided weakness_ She has been experiencing progressive exertional dyspnea, nocturnal cough and occasional hemoptysis over the past six months_ She also describes frequent episodes of palpitations and irregular heart beats_ Which of the following is the most likely diagnosis?

• A_ rylitral stenosis [6n]

B_ Hypertrophic cardiomyopathy [4913]

C. Aortic insufficiency [7%]

.Wolf-Parkinson-White syndrome [129C]

E Primary pulmonary hypertension [1.6%]

Explanation:

The most likely underlying diagnosis for all of this patient's symptoms is mitral stenosis secondary to rheumatic fever. The information that the patient is from Cambodia is helpful since rheumatic fever is more common in developing countries_ The stenotic mitral valve causes an increase in the left atrial pressure which in turn gets transmitted to the pulmonary vascular bed. This increase in pulmonary vascular pressure causes pulmonary congestion which results in symptoms such as exertional dyspnea, nocturnal cough. and hemoptysis. Hemoptysis in particular should raise suspicion for mitral stenosis_ Patients with mitral stenosis are at a high risk of developing atrial fibrillation due to left atrial dilation. The patient's episodes of palpitations and irregular heartbeats likely represent untreated atrial fibrillation. The patient in this scenario probably developed a thrombus in her left atrium secondary to untreated atrial fibrillation_ The thrombus then embolized to her cerebral circulation causing a stroke and left-sided weakness.

(Choice B) Hypertrophic cardiomyopathy (H C1'..,1) is less likely to be the correct diagnosis_ Syncope. angina, and palpitations are most typical of HCM_ Hemoptysis and nocturnal cough are less typical of HCM. Atrial fibrillation and stroke may result from HCM, but this patient's other symptoms and the fact that she is from a developing country point towards rheumatic mitral stenosis_

(Choice C) Aortic insufficiency can be associated with syphilis or rheumatic fever, both of which have a higher incidence in developing countries. Aortic insufficiency could also possibly cause dyspnea and cough from pulmonary congestion. However. left atrial pressure is typically less elevated in aortic insufficiency than in mitrAl qtpnnqiq Thprpfnrp in inrtir inzi iffiripnry thprp iq miirh Intivpr likplihnnrl ref hpmnnhiqiq triA

1.1 Feedback, End Block

1

1 6 3

5

• 7

• 9

• 10

• 11

• 12

• 13

• 14

• 15

• 16 • • 17

• 19

21

22

• 23

• 24

• 25

• 26

27

I • 29

1

• 32 ■

• 33

▪ 34-

• 36

I 37

39

40

• 30

Item: 38 of 44

117 Notes

Mark Previous Next Lab Values. Calculator.

44. Explanation:

The most likely underlying diagnosis for all of this patient's symptoms is mitral stenosis secondary to rheumatic fever_ The information that the patient is from Cambodia is helpful since rheumatic fever is more common in developing countries_ The stenotic mitral valve causes an increase in the left atrial pressure which in turn gets transmitted to the pulmonary vascular bed_ This increase in pulmonary vascular pressure causes pulmonary congestion which results in symptoms such as exertional dyspnea, nocturnal cough, and hemoptysis. Hemoptysis in particular should raise suspicion for mitral stenosis_ Patients with mitral stenosis are at a high risk of developing atrial fibrillation due to left atrial dilation. The patient's episodes of palpitations and irregular heartbeats likely represent untreated atrial fibrillation. The patient in this scenario probably developed a thrombus in her left atrium secondary to untreated atrial fibrillation. The thrombus then embolized to her cerebral circulation causing a stroke and left-sided weakness.

(Choice B) Hypertrophic cardiomyopathy (H C1'..,1) is less likely to be the correct diagnosis. Syncope. angina. and palpitations are most typical of HCM. Hemoptysis and nocturnal cough are less typical of HC1'....1. Atrial fibrillation and stroke may result from HCM, but this patient's other symptoms and the fact that she is from a developing country point towards rheumatic mitral stenosis_

(Choice C) Aortic insufficiency can be associated with syphilis or rheumatic fever. both of which have a higher incidence in developing countries_ Aortic insufficiency could also possibly cause dyspnea and cough from pulmonary congestion. However, left atrial pressure is typically less elevated in aortic insufficiency than in mitral stenosis. Therefore. in aortic insufficiency there is a much lower likelihood of hemoptysis, atrial fibrillation and cardioembolic stroke.

(Choice 0) The patient's palpitations and irregular heart rate could possibly be explained by %2VP.A.T. but since WPW causes a transient tachycardia it would be unlikely to cause progressive dyspnea, cough. and hemoptysis over 6 months.

(Choice E) Primary pulmonary hypertension is a rare disorder that is more common in females in the fourth and fifth decades, so this patient is in the appropriate demographic_ However. it involves the pulmonary arteries. so the left atrium is unaffected_ Therefore, left atrial dilation and left atrial thrombus formation are unlikely in primary pulmonary hypertension.

Educational objective: stenosis results in left atrial dilation and a risk of atrial fibrillation and cardiac emboli_ The pressure is

also transmitted to the pulmonary vasculature, which can result in dyspnea, cough and hemoptysis_

1.1 Feedback, End Block

41

1

1 6 3

5

▪ 7

▪ 9

• 10 • 11 • 12 • 13 ▪ 14 • • 15 ▪ 16 • • 17

• 19 ▪ ARM

21 22

• 23 • 24 • 25 ▪ 26

27 I

• 29

• 1 • 32 ■

• 33 • 34 El

• 36 • 37

0 0 u

39 40

• 30

Item: 38 of 44 Mark

W. 4. Previous Next

Lab Values. Notes Calculator

44.

The most likely underlying diagnosis for all of this patient's symptoms is mitral stenosis seconoary to rheumatic fever. The information that the patient is from Cambodia is helpful since rheumatic fever is more common in developing countries_ The stenotic mitral valve causes an increase in the left atrial pressure which in turn gets transmitted to the pulmonary vascular bed_ This increase in pulmonary vascular pressure causes pulmonary congestion which results in symptoms such as exertional dyspnea, nocturnal cough, and hemoptysis. Hemoptysis in particular should raise suspicion for mitral stenosis_ Patients with mitral stenosis are at a high risk of developing atrial fibrillation due to left atrial dilation. The patient's episodes of palpitations and irregular heartbeats likely represent untreated atrial fibrillation_ The patient in this scenario probably developed a thrombus in her left atrium secondary to untreated atrial fibrillation_ The thrombus then embolized to her cerebral circulation causing a stroke and left-sided weakness_

(Choice B) Hypertrophic cardiomyopathy (HCM) is less likely to be the correct diagnosis_ Syncope. angina. and palpitations are most typical of HCM_ Hemoptysis and nocturnal cough are less typical of HCM. Atrial fibrillation and stroke may result from HCM, but this patient's other symptoms and the fact that she is from a developing country point towards rheumatic mitral stenosis_

(Choice C) Aortic insufficiency can be associated with syphilis or rheumatic fever. both of which have a higher incidence in developing countries_ Aortic insufficiency could also possibly cause dyspnea and cough from pulmonary congestion_ However, left atrial pressure is typically less elevated in aortic insufficiency than in mitral stenosis_ Therefore, in aortic insufficiency there is a much lower likelihood of hemoptysis. atrial fibrillation and cardioembolic stroke_

(Choice 0) The patient's palpitations and irregular heart rate could possibly be explained by WP W. but since WPW causes a transient tachycardia it would be unlikely to cause progressive dyspnea, cough, and hemoptysis over 6 months_

(Choice E) Primary pulmonary hypertension is a rare disorder that is more common in females in the fourth and fifth decades, so this patient is in the appropriate demographic_ However, it involves the pulmonary arteries, so the left atrium is unaffected_ Therefore, left atrial dilation and left atrial thrombus formation are unlikely in primary pulmonary hypertension_

Educational objective: Mitral stenosis results in left atrial dilation and a risk of atrial fibrillation and cardiac emboli_ The pressure is also transmitted to the pulmonary vasculature. which can result in dyspnea, cough and hemoptysis_

Copyright © UWorld Last updated: [8/2212014]

1.1 Feedback, End Block

• 42

41

1

5

▪ 7

▪ 9

• 10 • 11

• 12 • 13 ▪ 14 1. • 15 ▪ 16 •

• 17

• 19

21

22

• 23 • 24 • 25 ▪ 26

27

I • 29

• 1 • 32 111 • 33 • 34- El

• 36 • 37

38

• 39

I

• 30

Item: 39 of 44 Mark

Previous Next

Lab Values. Notes . Calculator.

44.

A 65-year-old female is admitted to the hospital with increasing shortness of breath, weight gain and lower extremity edema_ She has a history of hypertension, nonischemic cardiomyopathy with an ejection fraction of 30%, and hyperlipidemia_ Her home medications include oral aspirin, digoxin. furosemide, metoprolol, lisinopril and atorvastatin. She is started on intravenous furosemide_ On day three of hospitalization telemetry reveals six beats of wide complex. ventricular tachycardia_ Physical examination now shows decreased leg edema and clear lungs_ Which of the following is the most appropriate next step in the management of this patient's tachycardia?

A. Add spironolactone [FA]

11 Add metolazone [2%]

• C_ Measure serum electrolytes [84%]

D_ Discontinue atorvastatin VIN

E. Discontinue metoprolol [4%]

Explanation:

When a patient experiences recurrent VT (ventricular tachycardia), the first thing to do after stabilizing the patient is to search for an underlying cause_ This patient most likely has an electrolyte imbalance due to diuretics. Furosemide commonly causes hypokalemia and hypomagnesemia_ If uncorrected, both hypokalemia and hypomagnesemia can lead to ventricular tachycardia_ Furthermore, hypokalemia potentiates side effects of digoxin, which include arrhythmias such as ventricular tachycardia_ Therefore, ordering serum electrolytes and serum digoxin level is the most reasonable initial approach.

(Choice Al Spironolactone is a potassium sparing diuretic, and has proven mortality benefits in patients with severe CHF. It would be of benefit in the long-term management of this patient, and could balance out the potassium wasting caused by furosemide_ However, immediate identification and reversal of the underlying cause of this patient's VT is the priority_

(Choice 6) Metolazone is a thiazide diuretic_ Adding another diuretic is unnecessary because the patient's fluid overload has resolved_ In addition, metolazone could further worsen her electrolyte imbalances and increase her risk of developing recurrent ventricular tachycardia_

(Choice 131 Stowing the patient's atorvastatin is unnecessary because it does not cause ventricular

1.1 Feedback, End Block

• 42

41

1

5

• 7

• 9

• 10 • 11

• 12 • 13 • 14 • 15 • 16 • • 17

• 19

21

22

• 23 • 24 • 25 • 26

27

I

• 29

• 1 • 32 ■

• 33 • 34-

• 36 • 37

38

• 39

I

• 30

Item: 39 of 44 Mark Previous Next Lab Values.

117 Notes . Calculator.

44. B_ Add metolazone [2%]

• C_ Measure serum electrolytes [84%]

D_ Discontinue atorvastatin [1%]

E. Discontinue metoprolol [4%]

Explanation:

When a patient experiences recurrent (ventricular tachycardia), the first thing to do after stabilizing the patient is to search for an underlying cause. This patient most likely has an electrolyte imbalance due to diuretics. Furosemide commonly causes hypokalemia and hypomagnesemia_ If uncorrected. both hypokalemia and hypomagnesemia can lead to ventricular tachycardia. Furthermore, hypokalemia potentiates side effects of digoxin, which include arrhythmias such as ventricular tachycardia. Therefore, ordering serum electrolytes and serum digoxin level is the most reasonable initial approach_

(Choice A) Spironolactone is a potassium sparing diuretic, and has proven mortality benefits in patients with severe CHF. It would be of benefit in the long-term management of this patient. and could balance out the potassium wasting caused by furosemide. However, immediate identification and reversal of the underlying cause of this patient's VT is the priority_

(Choice B) Metolazone is a thiazide diuretic_ Adding another diuretic is unnecessary because the patient's fluid overload has resolved. In addition, metolazone could further worsen her electrolyte imbalances and increase her risk of developing recurrent ventricular tachycardia.

(Choice 0) Stopping the patient's atorvastatin is unnecessary because it does not cause ventricular arrhythmias and the patient needs it to treat her hyperlipidemia_

(Choice E) Beta-blockers should be avoided in acute decompensated heart failure due to systolic dysfunction_ However, the patient's congestive symptoms have now improved so holding the metoprolol is currently unnecessary_ Metoprolol does not cause ventricular arrhythmias_

Educational objective: Loop diuretics cause hypokalemia and hypomagnesemia_ These electrolyte abnormalities can cause ventricular tachycardia, and also potentiate the side effects of digoxin.

Copyright CI Morld Last updated: [8/2212014]

1.1 Feedback, End Block

CP,*

Notes

Item: 40 of 44 Calculator Lab Values

rNlark Previous Next

A 74-year-old woman comes to the physician because of diarrhea, nausea, and decreased appetite. She has had these symptoms for 'I week. She also complains of increasing fatigue and occasional palpitations over the past few days_ Her other medical problems include chronic atrial fibrillation and cardiomyopathy_ Her cardiac ejection fraction measured 3 months ago showed 4096_ Her medications include furosemide, metoprolol, digoxin, and warfarin. She has smoked one pack of cigarettes daily for the past 5G years_ Her blood pressure is 140190 mm Hg and pulse is 70/min and irregularly irregular_ Lung auscultation reveals scattered wheezes. Her abdomen is soft and nontender, with a liver span of 8 cm and nonpalpable spleen. There is no ankle edema. Her latest INR measured 2 weeks ago was 2_1 Which of the following is the most appropriate initial test in this patient?

Explanation:

This patient is receiving treatment for chronic atrial fibrillation and presents with diarrhea, nausea, and fatigue consistent with possible digoxin toxicity_ Digoxin is a cardiac glycoside used to treat atrial fibrillation and heart failure and is renally cleared with a narrow therapeutic index. Digoxin toxicity typically presents with nausea, vomiting, decreased appetite, confusion. and weakness. The patient can also have visual symptoms of scotomata, blurry vision with changes in color, or blindness_ An inciting event, such as a viral illness or excessive diuretic use, can lead to volume depletion or renal injury that acutely elevates the digoxin level_ This leads to symptoms of toxicity in patients chronically taking digoxin_ Hypokalemia, often associated with loop diuretic use, increases the patient's susceptibility to the toxic effects of digoxin_

This patient should first have a digoxin level to rule out toxicity as well as ECG and a PTIINR to rule out life-threatening arrhythmias and coagulopathy (high INR from low oral intake) before evaluating for other causes of her symptoms.

1

3

5

7 ▪ a

• 9

▪ 10

a 11

▪ 12 • 13 ▪ 14 • • 15 ▪ 16 • • 17

• 19

22

• 23 • 24 • 25 ▪ 26

27 28 I

▪ 29 ▪ 30

▪ 32 ■

• 33 ▪ 34-

▪ 36 • 37

39 a 40

41

42

A_ Echocardiography [10%)]

B_ Chest x-ray [1€56]

C_ Pulmonary function testing [1%]

D_ Thyroid function testing [15?...6]

E Blood drug level [54%]

• Upper gastrointestinal endoscopy [2%]

G. ECG stress test [2%]

1

1 6 3

5

▪ 7

▪ 9

• 10 • 11 • 12 • 13 ▪ 14 1. • 15 ▪ 16 •

• 17

• 19

21 22

• 23 • 24 • 25 ▪ 26

27

I • 29

• 1 • 32 111 • 33 • 34- El

• 36 • 37

39 a 40

• 30

Item: 40 of 44 Mark Previous Next

Lab Values. Notes Calculator

This patient is receiving treatment for chronic atrial fibrillation and presents with diarrhea, nausea and fatigue consistent with possible digoxin toxicity_ Digoxin is a cardiac glycoside used to treat atrial fibrillation and heart failure and is renally cleared with a narrow therapeutic index_ Digoxin toxicity typically presents with nausea, vomiting, decreased appetite, confusion, and weakness_ The patient can also have visual symptoms of scotomata, blurry vision with changes in color, or blindness_ An inciting event, such as a viral illness or excessive diuretic use, can lead to volume depletion or renal injury that acutely elevates the digoxin level_ This leads to symptoms of toxicity in patients chronically taking digoxin_ Hypokalemia, often associated with loop diuretic use, increases the patient's susceptibility to the toxic effects of digoxin.

This patient should first have a digoxin level to rule out toxicity as well as ECG and a PT/INR to rule out life-threatening arrhythmias and coagulopathy (high INR from low oral intake) before evaluating for other causes of her symptoms.

(Choice A) Echocardiography can be beneficial in evaluating systolic and diastolic function, ejection fraction, wall motion abnormalities, and valve disorders_ This patient does not appear to have a decompensated heart failure (no jugular venous distension or edema) at this time and does not require echocardiography.

(Choice B) Aside from wheezes on examination, this patient does not have clinical signs suggestive of a primary pulmonary process_ None of her medications cause significant pulmonary toxicity, and lung disease probably would not cause her gastrointestinal (GI) symptoms_

(Choice C) Pulmonary function tests are used to diagnose restrictive and obstructive pulmonary diseases_ Although this patient has wheezing on physical examination, suggesting a component of bronchoconstriction, neither COPD nor asthma would account for her GI symptoms_

(Choice 0) Levels of TSH, T3, and T4 are measured to evaluate thyroid function. Hyperthyroidism can cause diarrhea, palpitations, and changes in appetite. However, it is more likely that this patient's presentation is related to an adverse medication effect rather than a new disease process_

(Choice F) Upper GI endoscopy is invasive and unlikely to explain this patient's symptoms_ If the serum digoxin level and renal function panel are within normal limits, further workup for GI symptoms is appropriate.

(Choice G) Stress testing is a reasonable investigation in an older woman complaining of palpitations, but coronary artery disease would not explain this patient's GI symptoms_ IN:loreover. an ECG stress test can be very difficult to interpret in someone with baseline ECG abnormalities from digoxin use.

Educational objective: irrrrvi rri r rrlirr r ciri }.with arl7rarca af-Fa I n trnrrnifinrr rii irioirrn r hinnac

1.1 Feedback, End Block

• 1

▪ 3

G 5

7

• 9

• 10

• 11

• 12

• 13

• 14

• 15

• 16

• 17

▪ 1 • 19

• 2

- 21

▪ 22

• 23

• 24

• 25

▪ 26

• 27

• 28 I

• 29

• 30

• 32

• 33

▪ 34-

• 36

• 37

• 39

• 40 • 41

• 42

Item: 40 of 44

r Mark

IF" Previous Next

Lab Values. Notes Calculator

III c-LI II edLe1 III iy dl I I iyu II I 11dS dl IU Ludyulupciu Iy kI iiyi I HU\ II UI I I ILIWY UI dl II ILdrke, Uel UI C eVdlUdLII ly I UI ULI ICI

causes of her symptoms_

(Choice A) Echocardiography can be beneficial in evaluating systolic and diastolic function, ejection fraction, wall motion abnormalities, and valve disorders. This patient does not appear to have a decompensated heart failure (no jugular venous distension or edema) at this time and does not require echocardiography_

(Choice B) Aside from wheezes on examination, this patient does not have clinical signs suggestive of a primary pulmonary process_ None of her medications cause significant pulmonary toxicity, and lung disease probably would not cause her gastrointestinal (GI) symptoms.

(Choice C) Pulmonary function tests are used to diagnose restrictive and obstructive pulmonary diseases. Although this patient has wheezing on physical examination. suggesting a component of bronchoconstriction, neither COPD nor asthma would account for her GI symptoms.

(Choice Cl) Levels of TSH, T3, and T4 are measured to evaluate thyroid function. Hyperthyroidism can cause diarrhea, palpitations, and changes in appetite_ However, it is more likely that this patient's presentation is related to an adverse medication effect rather than a new disease process.

(Choice F) Upper GI endoscopy is invasive and unlikely to explain this patient's symptoms. If the serum digoxin level and renal function panel are within normal limits, further workup for GI symptoms is appropriate_

(Choice G) Stress testing is a reasonable investigation in an older woman complaining of palpitations, but coronary artery disease would not explain this patient's GI symptoms_ Moreover, an ECG stress test can be very difficult to interpret in someone with baseline ECG abnormalities from digoxin use_

Educational objective: Digoxin is a cardiac glycoside with adverse effects that include nausea. vomiting, diarrhea, vision changes. and arrhythmias_ Patients chronically taking digoxin should have close and routine monitoring of their digoxin levels_

References:

1. Recognition and management of digitalis intoxication: implications for emergency medicine

Copyright © UWorld Last updated: [1 G/20/201 4]

44.

1.1 Feedback, End Block

Ipm._

Previous Next Item: 41 of 44

Calculator Lab Values, rNlark

Notes

End Block 1.1 Feedback,

44.

A 59-year-old man comes to visit a friend in the hospital and collapses in the parking lot. He had been feeling unwell all day due to vague chest discomfort. A bystander witnesses his collapse. finds no pulse. and immediately calls for help. Which of the following is the most important factor for suniival in this patient?

Explanation:

1

3

5

• 7

• 9

• 10 • 11 • 12 • 13 • 14 • • 15 • 16 • 17

• 19 • ARM

21 22

• 23 • 24 • 25 • 26

27

• 29

• 1 • 32 ■

• 33 • 34

• 36 • 37

• 39 I • 40

a 41

A Time to rescue breathing [18%]

• 11 Time to rhythm analysis and defibrillation [58%]

C_ Time to endotracheal intubation [2%]

D_ Time to epinephrine injection [3%]

E Time to cardiac catheterization lab [1996]

Factors associated with Ipoor outcome after witnessed

out-of-hospital sudden cardiac arrest

• Time elapsed prior to effective resuscitation (delayed bystander

CPR, delayed defibrillation)

• Initial rhythm of pulseless electrical activity or as stole

• Prolonged CPR (> 5 minutes)

• Absence of vital signs

▪ Advanced age

• Prior history of cardiac disease

• > 2 Chronic illnesses

• Persistent coma after CPR

• Need for intubation or vasopresscirs

• Pneumonia or renal failure after CPR

• cpncic. rprphrrivaccillar clacc Ill nr IV heart fail! ire

I

• 30

1

• 7

• 22

3

5

• 9 • 10 • 11

• 12 • 13 • 14 • 15 • 16 • • 17

• 19

21

• 23 • 24 • 25 • 26

27

28 I

• 29 • 30 • 1 • 32 ■

• 33 • 34-

• 36 • 37

• 39 I • 40

• 41 • 42

Item: 41 of 44

r Mark 117

Previous Next

Lab Values. Notes . Calculator .

Factors associated with poor outcome after witnessed

out-of-hospital sudden cardiac arrest

Time elapsed prior to effective resuscitation (delayed bystander

CPR, delayed defibrillation)

Initial rhythm of puIseles electrical activity or a systole

Prolonged CPR (> 5 minutes)

Absence of vital signs

Advanced age

Prior history of cardiac disease

2 Chronic illnesses

Persistent coma after CPR

Need for intubation or vasopressors

Pneumonia or renal failure after CPR

Sepsis, cerebrovascular accident, or class III or IV heart failure

el LISMEWorld. LLC

This patient had a witnessed out-of-hospital sudden cardiac arrest (SCA). The most common cause of out-of-hospital SCA in adults is sustained ventricular tachycardia or fibrillation due to acute myocardial ischemia or infarction. Despite best efforts, resuscitation for out-of-hospital SCA is successful in only one-third of patients. and only about 10% of all patients are eventually discharged from the hospital.

Multiple factors affect overall survival in patients with witnessed SCA (see table). The most critical factor determining overall patient survival is elapsed time to effective resuscitation_ This includes effective bystander CPR, prompt rhythm analysis, and early defibrillation for patients found to be in ventricular fibrillation. Ventricular fibrillation almost never terminates spontaneously, and early rhythm analysis and defibrillation are the only effective means to reestablish perfusing cardiac rhythm and improve patient survival.

(Choice Al Early administration of effective bystander CPR is an important factor that determines patient cq inri7raI aftar r i it rif QrA Qt I Irliafi in QrA ni-c• r ni 11-r r• and imnrntra rl

1.1 Feedback, End Block

' 30

1

1 6 3

5

• 7

• 9

' 10

i 11

i 12

• 13

• 14

• 15

• 16 • • 17

• 19

21

22

• 23

• 24

• 25

• 26

27

I • 29

i 32 ■

• 33

' 34-

• 36

• 37

• 39

I • 40

a 41

Item: 41 of 44

r Mark

1121:1P, Previous Next

Lab Values. Notes Calculator

This patient had a witnessed out-of-hospital sudden cardiac arrest (SCA). The most common cause of out-of-hospital SCA in adults is sustained ventricular tachycardia or fibrillation due to acute myocardial ischemia or infarction. Despite best efforts, resuscitation for out-of-hospital SCA is successful in only one-third of patients, and only about 10% of all patients are eventually discharged from the hospital.

Multiple factors affect overall survival in patients with witnessed SCA (see table). The most critical factor determining overall patient survival is elapsed time to effective resuscitation_ This includes effective bystander CPR, prompt rhythm analysis, and early defibrillation for patients found to be in ventricular fibrillation. Ventricular fibrillation almost never terminates spontaneously, and early rhythm analysis and defibrillation are the only effective means to reestablish perfusing cardiac rhythm and improve patient survival_

(Choice A) Early administration of effective bystander CPR is an important factor that determines patient survival after out-of-hospital SCA. Studies in SCA patients have shown better outcomes and improved survival to discharge with compression-only CPR as compared to standard CPR that includes rescue breathing_ The 2010 American Heart Association guidelines for CPR recommend that bystanders should perform compression-only CPR prior to arrival of emergency personnel_

(Choice C) Endotracheal intubation should be performed to secure the airway as a part of overall management only after the patient has been resuscitated successfully with restoration of circulation. Placement of an endotracheal tube should not delay the initial efforts of rhythm analysis and early defibrillation_

(Choice 0) The initial approach in patients with out-of-hospital SCA should be prompt and effective CPR, prompt rhythm analysis. and early defibrillation if indicated_ Advanced cardiac life support guidelines recommend epinephrine only in patients with asystole, pulseless electrical activity, or refractory ventricular arrhythmias unresponsive to defibrillation_

(Choice E) Acute myocardial ischemia or infarction is the leading cause of SCA due to ventricular tachycardia/fibrillation_ Patients who survive the initial resuscitation efforts should undergo coronary angiography to evaluate for coronary artery disease.

Educational objective: Despite advances in therapy, the overall survival rate for patients with sudden cardiac arrest remains quite poor. The most important factors in improving patient survival are prompt effective resuscitation with adequate bystander CPR, prompt rhythm analysis, and defibrillation in patients found to be in ventricular fibrillation.

Copyright CI Morld Last updated: [12/19/2014] a 1.1

Feedback, End Block

1

1 6 3

5

▪ 7

▪ 9

' 10 i 11 i 12 • 13 ▪ 14 • • 15 ▪ 16 •

• 17

• 19 ▪ ARM

21 22

• 23 • 24 • 25 ▪ 26

27 I

• 29

• 1 i 32 ■

• 33 ' 34- El

' 36 • 37

• 39 • 40

41

' 30

End Block 1.1 Feedback,

Features of constrictive pericarditis

• Idiopathic or viral pericarditis • Cardiac surgery or radiation therapy • Tuberculous pericarditis (in endemic areas)

Etiology

Item: 42 of 44 r Mark

Previous Next Lab Values. Notes Calculator.

A 64-year-old man comes to the physician due to shortness of breath and abdominal distension_ He was treated for Hodgkin lymphoma with radiation and chemotherapy 18 years ago and was told that he was cured. The patient drinks alcohol on a regular basis. His temperature is 36_7 C (98 F); blood pressure is 120/76 mm Hg. pulse is 92/min. and respirations are 20/min. Neck examination shows jugular venous pulsations 9 cm above the sternal angle. The abdomen is distended with a positive fluid wave_ The liver edge is palpated 5 cm below the right costal edge. There is bilateral lower-extremity pitting edema_ Initial laboratory results are as follows:

Serum creatinine 0_8 mg/I:IL Albumin 4.0 g/dL Total bilirubin 1.0 mg/I:IL Prothrombin time 11 sec

Which of the following is the most likely cause of this patient's condition?

4.0 A_ Inelastic pericardium [55%]

H. Portal vein compression [18%]

C_ Portal vein thrombosis [15%]

D_ Thoracic duct obstruction [8%]

E. Urinary protein loss [1%]

Venous valve incompetence [2%]

Explanation:

EVI*

Notes Item: 42 of 44

Calculator Lab Values. rNlark

Previous Next

1.1 Feedback, End Block

Features of constrictive pericarditis

Etiology • Idiopathic or viral pericarditis • Cardiac surgery or radiation therapy • Tuberculous pericarditis (in endemic areas)

Clinical presentation

• Fatigue & dyspnea on exertion • Peripheral edema & ascites • ' Jugular venous pressure • Pericardial knock may be heard • Pulsus paradoxus • Kussmaul's sign

, .

Diagnostic findings

• ECG may be nonspecific or show atrial fibrillation or low-voltage ORS complex

• Imaging shows pericardial thickening & calcification

• Jugular venous pulse tracing shows prominent x & y descents

44.

'U World

This patient's presentation is suggestive of right heart failure, most likely due to constrictive pericarditis. a potential late complication of radiation therapy_ Survivors of Hodgkin lymphoma are at increased risk for cardiac disease, which can present as much as 10-20 years or more after mediastinal irradiation and' or anthracycline therapy_ Potential cardiac complications of Hodgkin lymphoma include acute or delayed pericardial disease, myocardial ischemialinfarction, restrictive cardiomyopathy, congestive heart failure. valvular abnormalities. and conduction defects_

Constrictive pericarditis occurs as a result of scarring and subsequent loss of normal elasticity of the pericardial sac. The inelastic pericardium prevents venous return to the right heart during inspiration and

1

3

5

7

• 9 • 10 ▪ 11 ▪ 12 • 13 ▪ 14 1. • 15 ▪ 16 • 17

• 19 ▪ ARM

21 22

• 23 • 24 • 25 ▪ 26

27 28 I

• 29 • 30

• 32 ■

• 33 ▪ 34 El

• 36 • 37

.• 39 I. 40

41 I 42

rReferences:

1. Pericardial constriction: diagnosis and management.

Item: 42 of 44 r Mark Previous Next Lab Values.

001°1' Notes Calculator.

Constrictive pericarditis occurs as a result of scarring and subsequent loss of normal elasticity of the pericardial sac_ The inelastic pericardium prevents venous return to the right heart during inspiration and leads to right heart failure_ Patients typically present with peripheral edema, ascites, and hepatic congestion with hepatomegaly. which can progress to cirrhosis (cardiac cirrhosis). Physical examination shows elevated jugular venous pressure (JVP) with prominent x and y descents and hepatojugular reflux, Kussmaul's sign (lack of decrease or increase in JVP on inspiration). or pericardial knock (mid-diastolic sound).

Patients may have pericardial calcifications on chest radiograph_ Echocardiography confirms the diagnosis and typically shows increased pericardial thickness. abnormal septal motion, and biatrial enlargement. Diuretics can provide temporary relief, and pericardiectomy is the definitive treatment for patients with refractory symptoms_

(Choices B and C) Patients with portal vein thrombosis/compression are usually asymptomatic or develop signs and symptoms related to portal hypertension_ Predisposing conditions for portal vein thrombosis include decompensated cirrhosis or hypercoagulable states_ JVP is typically not elevated_

(Choice 0) Obstruction of the thoracic duct (or its tributaries) leads to leakage of lymphatic fluid (chyle) into the pleural space, resulting in unilateral or bilateral chylothorax_ Ascites and hepatomegaly are not associated with thoracic duct obstruction_

(Choice E) Heavy proteinuria and hypoalbuminemia can cause peripheral and/or generalized edema (ie, anasarca). Patients with severe hypoalbuminemia can develop intravascular volume depletion. and JVP is often reduced or normal_ This patient's serum albumin is within normal limits.

(Choice F) Lower-extremity venous valve incompetence can lead to dependent edema, skin pigmentation. dermatitis/eczema, and venous ulcerations. Jugular venous distension. ascites, and hepatomegaly are typically not present

Educational objective: Constrictive pericarditis is a complication of mediastinal irradiation and an important cause of right heart failure_ It should be suspected in patients with progressive peripheral edema, elevated jugular venous pressure, hepatomegaly, and ascites_ Other findings include the presence of hepatojugular reflux. Kussmaul's sign (lack of decrease or increase in jugular venous pressure on inspiration). and a pericardial knock (mid-diastolic sound).

1.1 Feedback, End Block

G 5

127 Notes Calculator .

Item: 42 of 44 Lab Values.

r Mark Previous Next

1 Copyright CI Morld Last updated: [8/7/2014]

1.1 Feedback, End Block

TV I LI I I I 4-11.) ULU! I I 4-• y Lil y I I I 'LA. I I Li. Lil I 1.-1, I 4.) 14s y7J 1-4) 4. II I I ILIIJ Lr Lil LI I Li Lr Li. II I I ILIJ- I I ly J14.U1 1._. A.L11 I 1111L4L14,1 11 JI 14J YYJ 4s 14Y uLI Los 4J

jugular venous pressure (JVP) with prominent x and y descents and hepatojugular reflux, Kussmaul's sign (lack of decrease or increase in JVP on inspiration), or pericardial knock (mid-diastolic sound).

Patients may have pericardial calcifications on chest radiograph_ Echocardiography confirms the diagnosis and typically shows increased pericardial thickness, abnormal septal motion, and biatrial enlargement. Diuretics can provide temporary relief, and pericardiectomy is the definitive treatment for patients with refractory symptoms_

(Choices B and C) Patients with portal vein thrombosis/compression are usually asymptomatic or develop signs and symptoms related to portal hypertension_ Predisposing conditions for portal vein thrombosis include decompensated cirrhosis or hypercoagulable states. JVP is typically not elevated_

(Choice 0) Obstruction of the thoracic duct (or its tributaries) leads to leakage of lymphatic fluid (chyle) into the pleural space, resulting in unilateral or bilateral chylothorax_ Ascites and hepatomegaly are not associated with thoracic duct obstruction_

(Choice E) Heavy proteinuria and hypoalbuminemia can cause peripheral andlor generalized edema (ie, anasarca). Patients with severe hypoalbuminemia can develop intravascular volume depletion. and JVP is often reduced or normal_ This patient's serum albumin is within normal limits.

(Choice F) Lower-extremity venous valve incompetence can lead to dependent edema, skin pigmentation. dermatitis/eczema, and venous ulcerations_ Jugular venous distension. ascites, and hepatomegaly are typically not present

Educational objective: Constrictive pericarditis is a complication of mediastinal irradiation and an important cause of right heart failure_ It should be suspected in patients with progressive peripheral edema, elevated jugular venous pressure, hepatomegaly, and ascites_ Other findings include the presence of hepatojugular reflux. Kussmaul's sign (lack of decrease or increase in jugular venous pressure on inspiration). and a pericardial knock (mid-diastolic sound).

• 1

▪ 3

7

• 9

• 10

▪ 11

• 12

• 13

• 14

• 15

• 16

• 17

▪ 1 • 19

• 2

- 21

▪ 22

• 23

• 24

• 25

• 26

• 27

• 28 I

• 29

• 30

• 32

• 33

▪ 34- • '1=.

• 36

• 37 •

I 39

• 40

• 41

I 42

References:

1. Pericardial constriction: diagnosis and management.

117 Notes . Calculator.

Item: 43 of 44 Lab Values.

rNlark Previous Next

1.1 Feedback, End Block

1

3

5

7

• 9 • 10

▪ 11 ▪ 12 • 13 ▪ 14 1. • 15 ▪ 16 •

• 17

• 19 ▪ ARM

21 22

• 23 • 24 • 25 ▪ 26

27 28 I

• 29 • 30

▪ 32 111 • 33 ▪ 34- El

▪ 36 • 37

I 39 • 40 • 41 ▪ 42

44.

A 43-year-old woman with a past medical history of bronchial asthma comes to the physician for evaluation of high blood pressure. On her previous 2 visits, her blood pressure has been 154/88 and 150/92 mm Hg. She has no headaches or complaints. The patient uses fluticasone and as-needed albuterol for her chronic asthma She does not smoke or use alcohol and has no known drug allergies_ Her family history is significant for myocardial infarction in her father at age 65_ She exercises regularly and has followed a low-salt diet over the last 2 months_ The patient's blood pressure during this visit is 157/93 mm Hg and pulse is 69/min. Her body mass index is 27 kgim2_ The heart and lung examinations are unremarkable_ No vascular bruits are heard. and peripheral pulses are 2+ and symmetrical_ Electrocardiogram shows normal sinus rhythm. Complete blood count and chemistry panel are normal_ Which of the following is the most appropriate next step in management of this patient?

A_ 24—hour urine cortisol excretion [1096]

4.0 11 Lipid panel and urinalysis [43%]

C. Plasma renin activity [23%]

D_ Renal ultrasound [14%]

E Thyroid-stimulating hormone and urine metanephrines [10%]

Explanation:

Although most patients with hypertension have essential hypertension, initial evaluation should assess possible secondary causes_ This basic workup includes investigating the duration of hypertensioft presence of precipitating/aggravating factors, extent of target organ damage, and cardiovascular risk factors_ Basic laboratory analysis should include urinalysis (for occult hematuria and proteinicreatinine ratio), chemistry panel, lipid profile (risk stratification for coronary artery disease) and baseline electrocardiogram (to evaluate for coronary artery disease or left ventricular hypertrophy).

Further diagnostic evaluation should be performed in patients with signs or symptoms suggesting possible secondary hypertension_ These include severe or malignant hypertension, resistant hypertension requiring

drugs, sudden blood pressure rise in a patient with previously controlled blood pressure, and age of onset <30 without family history of hypertension.

(Choice A) A 24-hour urine cortisol excretion is used to evaluate for Cushing's syndrome_ However, this + r.+ r1 r• r..+ + + r.+ •-• +rteI •-• r I rti I rtitk

117 Notes . Calculator .

Item: 43 of 44 Lab Values

Mark Previous Next

44.

a

laboratory analysis should include urinalysis (for occult hematuria and protein/creatinine ratio). chemistry panel, lipid profile (risk stratification for coronary artery disease) and baseline electrocardiogram 'Jo evaluate for coronary artery disease or left ventricular hypertrophy).

Further diagnostic evaluation should be performed in patients with signs or symptoms suggesting possible secondary hypertension_ These include severe or malignant hypertension. resistant hypertension requiring

drugs, sudden blood pressure rise in a patient with previously controlled blood pressure, and age of onset <30 without family history of hypertension.

(Choice Al A 24-hour urine cortisol excretion is used to evaluate for Cushing's syndrome_ However, this patient does not have the characteristic findings (eg, central obesity, abdominal striae, facial plethora) of Cushing's syndrome_

(Choice C) Primary hyperaldosteronism typically presents with low plasma renin, hypokalemia due to renal potassium wasting, and hypertension_ Routine plasma renin activity is not recommended in patients with mild hypertension and normal serum potassium levels.

(Choice ID) Renal ultrasound can identify asymmetrical kidney size or small atrophic kidneys, which suggests primary renal disease_ Duplex Doppler ultrasound is also useful in screening for renal artery stenosis in patients with severe or resistant hypertension. However, these imaging studies are done only if there is an elevated serum creatinine or abnormal urinalysis_

(Choice E) Thyroid-stimulating hormone levels screen for occult hypothyroidism_ Twenty-four-hour urine metanephrines and catecholamines are elevated in patients with pheochromocytoma_ However, this patient does not have the typical findings of hypothyroidism (eg, fatigue, dry skin, constipation) or pheochromocytoma (eg, palpitations, headaches).

Educational objective: Patients initially diagnosed with hypertension should have a detailed history and physical examination. In addition, the following basic testing should be performed:

1. Urinalysis for occult hematuria and urine protein/creatinine ratio 2. Chemistry panel 3. Lipid profile 4. Baseline electrocardiogram

Copyright © UWorld

Last updated: [12/20/2014]

End Block 1.1

Feedback,

1

1 6 3

5

▪ 7

▪ 9

' 10 i 11 i 12 • 13 • 14 1. • 15 • 16 •

• 17

• 19 ▪ ARM

21 22

• 23 • 24 • 25 • 26

27

I • 29

• 1 i 32 ■

• 33 ' 34- El

' 36 • 37

• 39 ' 40

' 30

4 G

5 G

End Block 1.1 Feedback,

• 2

• 1

3

6

7

• 8

• 10

• 11

• 12

• 13

• 14-

• 15

• 16

• 17

• 18

• 19

• 20

• 21

▪ 22 • 23

• 24

• 25

• 26

• 27

• 28

• 29

• 30

• 32

• 33

• 34

• 36

• 38

• 39

• 40

117 Lab Values Notes Calculator .

Item: 43 of 44 r Mar k

Medlia Exhibit

Previous Next

Secondary causes of hypertension

v • Central obesity, facial plethora

Secondary causes of hypertension

Condition

Renal parenchymal disease

Renovascular disease

44.

Clinical cluesileatures

• Elevated serum creatinine

• Abnormal urinalysis (protein aria, red blood cell casts)

• Severe hypertension (MO ITIM Hg systolic andlor 120 mm Hg diastolic) after age 55

• Possible recurrent flash pulmonary edema or resistant heart failure

• Unexplained rise in serum creatinine

• Abdominal bruit

• Easily provoked hypokalemia

• Slight hypematremia • Hypertension with adrenal incidenthloma

Primary aldosteronism

• Paroxysmal elevated blood pressure with tachycardia • Pounding headaches, palpitations, diaphoresis • Hypertension with an adrenal incidentaloma

Pheochromocytoma

• 41

• 42

• 2

• 4

• 1

3

5

• 6

• 7

• 1^

• 11

• 12

• 13

• 14-

• 15

• 16

• 17

• 1

• 19

• 20

• 21

• 22

• 23

• 24

• 25

• 26

• 27

• 28

• 29

• 30

• 32

• 33

• 34

• 36

• 38

• 39

• 40

• 41

• 42

117 Lab Values Notes Calculator.

End Block 1.1

Feedback,

Secondary causes of hypertension

44.

• Easily provoked hypokalemia • Slight hypematremia • Hypertension with adrenal incidentaloma

Primary aldosteronism

• Paroxysmal elevated blood pressure with tachycardia • Pounding headaches, palpitations, diaphoresis • Hypertension with an adrenal incidentaloma

• Central obesity, facial plethora • Proximal muscle weakness, abdominal striae • Ecchymosis, amenorrhealerectile dysfunction • Hypertension with adrenal incidentaloma

• Fatigue, dry skin, cold intolerance • Constipation, weight gain, bradycardia

Pheochromocytoma

Cushing's syndrome

Hypothyroidism

• Hypercalcenlia (polyuria, polydipsia) • Kidney stones • Neuropsychiatric presentations (confusion, depression,

psychosis)

Primary hyperparathyroidism

• Differential hypertension with brachial-femoral pulse delay

Coarctation of the aorta

CUSMLEWorld, LLC v

Item: 43 of 44 Mark

Media Exhibit

Previous Next

1

5

• 7

• 9

• 10

• 11

• 12

• 13

• 14

• 15

• 16 • • 17

• 19

21

22

• 23

• 24

• 25

• 26

27

I • 29

• 1

• 32 ■

• 33

• 34-

• 36

• 37

• 39

• 40

I

• 30

Item: 44 of 44

rNlark 127

Previous Next Lab Values, Notes Calculator.

44.

A 64-year-old nondiabetic man complains of cramping pain in his right thigh after walking 2 blocks. The pain goes away once he stops and rests for several minutes. He is an ex-smoker with 35-pack-year history. He has no history of heart disease or stroke. On physical examination, the femoral pulses are normal. but the popliteal and dorsalis pedis pulses are diminished on the right leg. His ankle brachial index is 0.98 on the left and 172 on the right. He is started on low-dose aspirin and lipid-lowering therapy. liAbirhich of the following is the best initial management for this patient?

A. Arteriography followed by surgical bypass procedure [12°A]

B. Computed tomography angiography followed by stenting [21%]

• C_ Exercise therapy [52%]

• Pentoxifylline [1%]

E Warfarin [79...6]

Explanation:

The patient's clinical presentation is consistent with the presence of peripheral artery disease (PAD) involving the right lower extremity. PAD is a coronary artery disease risk equivalent, and the medical therapy for such patients should include aggressive risk factor modification with counseling for smoking cessation, lipid-lowering therapy. and evaluation and treatment for hypertension and diabetes mellitus.

This patient is started appropriately on low-dose aspirin and statin therapy. The next step is to enroll the patient in a supervised exercise program_ Exercise therapy for a minimum of 1 weeks, with 30-45 minutes of exercise at least 3 times a week, is recommended for all patients with claudication. The goal is to reproduce the claudication symptoms during each session. Supervised exercise programs have been shown in multiple clinical trials to reduce symptoms and improve the maximum walking distance in these patients.

(Choices A and B) Percutaneous or surgical revascularization is generally reserved for patients who have persistent symptoms despite initial exercise and/or pharmacologic therapy.

(Choice 0) Pentoxifylline has not been shown to provide any significant symptomatic improvement and is not recommended for use in patients with intermittent claudication. Cilostazol should be considered in patients who have persistent symptoms despite antiplatelet therapy and adequate supervised exercise programs.

1.1 Feedback, End Block

Item: 44 of 44

r Mark I>- 001°1'

Previous Next Lab Values. Notes Calculator pouci 1L3 3I ILIU1U 1114,11JUC oyyi C331VC 1131‘ 104,L1-11 1111-1U1111-0L11-111 WWILI I 1-1-1U113C111iy 11.-11 Z111,1-.11%.111 ,I.-CZMCILIMI I,

lipid-lowering therapy, and evaluation and treatment for hypertension and diabetes mellitus.

This patient is started appropriately on low-dose aspirin and statin therapy_ The next step is to enroll the patient in a supervised exercise program_ Exercise therapy for a minimum of 1 weeks, with 30-45 minutes of exercise at least 3 times a week, is recommended for all patients with claudication. The goal is to reproduce the claudication symptoms during each session. Supervised exercise programs have been shown in multiple clinical trials to reduce symptoms and improve the maximum walking distance in these patients.

(Choices A and B) Percutaneous or surgical revascularization is generally reserved for patients who have persistent symptoms despite initial exercise andfor pharmacologic therapy.

(Choice ID) Pentoxifylline has not been shown to provide any significant symptomatic improvement and is not recommended for use in patients with intermittent claudication. Cilostazol should be considered in patients who have persistent symptoms despite antiplatelet therapy and adequate supervised exercise programs.

(Choice E) Warfarin has not been shown to improve claudication symptoms or cardiovascular outcomes in patients with PAD and is not recommended.

Educational objective: A supervised exercise program should be recommended as a part of an initial treatment regimen in all patients with intermittent claudication. Pharmacologic therapy with cilostazol and percutaneous or surgical revascularization should be reserved for those with persistent symptoms despite adequate supervised exercise therapy.

References:

1. Supervised exercise versus primary stenting for claudication resulting from aortoiliac peripheral artery disease: six month outcomes from the claudication exercise versus endoluminal revascularization (CLEVER) study.

2. Systematic review of exercise training or percutaneous transluminal angioplasty for intermittent claudication.

3. 2011 ACCFIAHA focused update of the guideline for the management of patients with peripheral artery disease (updating the 2005 guideline).

Copyright @ Ii World

Last updated: [12/30/2014]

a 1.1

Feedback, End Block